You are on page 1of 271

2015 Orthopaedic In-Training Examination

Copyright  2015
by the American Academy of Orthopaedic Surgeons

All Rights Reserved. No part of the electronic media program may be reproduced, stored in a
retrieval system, or transmitted in any form or by any means, electronic, mechanical, photocopying,
recording, or otherwise without the prior written permission from the American Academy of
Orthopaedic Surgeons. Made in the USA.

Produced 2015 by the


American Academy of Orthopaedic Surgeons
9400 West Higgins Road
Rosemont, IL 60018-4976

Requests for permission to reproduce any part of the work should be mailed to:
Attention Examinations Department
American Academy of Orthopaedic Surgeons
9400 West Higgins Road
Rosemont, IL 60018-4976

© 2015 American Academy of Orthopaedic Surgeons 2015 Orthopaedic In-Training Examination


November 14, 2015

Dear Residents and Colleagues,

On behalf of the Central Evaluation Committee, I would like to thank you for participating in this
year’s examination.

Being the first such examination among all the medical specialties, this year marks the 53rd
administration of the Orthopaedic In-training Examination™.

Every year, the OITE™ is created by a group of orthopaedic surgeons who are deeply committed to
education through a rigorous peer-review process. While many topics in orthopaedic surgery lack
conclusive evidence, many of our practices are based on sound medical principles and a consensus of
experts. Sometimes, there may appear several correct answers to a test item, but each question has
been vetted through this peer-review process to select the one best preferred response.

With this and each subsequent administration, we strive to make the OITE™ better. Thank you for
being part of this year’s examination, and I hope that you find this to be an enriching educational
experience.

© 2015 American Academy of Orthopaedic Surgeons 2015 Orthopaedic In-Training Examination


AAOS Evaluation Programs
CME credit is awarded to a single respondent whose Academy Identification Number is associated with the
purchase of the examination to which credit is desired; no CME credit will be awarded to an individual who
has not purchased the examination from AAOS.

PDF Copy of the 2015 Orthopaedic In-Training Examination Study Guide


The PDF copy of the 2015 OITE Study Guide will be made available after the test-item analysis has been
completed which will be in January 2016.

Accreditation
The American Academy of Orthopaedic Surgeons is accredited by the Accreditation Council for Continuing
Medical Education to provide continuing medical education for physicians.

AMA PRA Credit


U.S. Physicians: The American Academy of Orthopaedic Surgeons designates this enduring material for a
maximum of 20 AMA PRA Category 1 Credits™. Physicians should claim only the credit commensurate with
the extent of their participation in the activity.

International Physicians: AMA PRA credit may only be claimed by, and awarded to, physicians, defined by
the AMA as individuals who have completed an allopathic (MD), osteopathic (DO) or an equivalent medical
degree from another country.

Allied Health Professionals: The AAOS is not accredited to offer credit for nurses and other allied health
professionals. To determine if activities offering AMA PRA Category 1 Credits™ are acceptable for your
licensing or certification needs please contact the relevant organizations directly.

Educational Objectives

Orthopaedic In-Training
As a result of taking the 2015 Orthopaedic In-Training Examination,
Examination, IIam
am able
ableto
to

 making clinical
interpret patient data for making clinical decision.
decision.
 determine the appropriate patient
patient management
management procedures
procedures in
in orthopaedic
orthopaedicsurgery.
surgery.
 evidence-based information
integrate relevant evidence-based information regarding
regarding treatment
treatment procedures
procedures into
into my
mypatient
patientpractice.
practice.
 principles in
apply basic science principles in diagnosing
diagnosing and
and developing
developing patient
patient management
managementplans
plansfor
formusculoskeletal
musculoskeletal
diseases.

Inquiries
Questions regarding completing this
this CME
CME activity
activity or
or other
other comments
comments can
canbe
besent
sentto
to exams@aaos.org
exams@aaos.orgor
orwrite
write
to: Attention Examinations Department,
Department, American
American Academy
Academy of of Orthopaedic
OrthopaedicSurgeons,
Surgeons,9400
9400West
WestHiggins
Higgins
Road, Rosemont, IL 60018-4976.
60018-4976.

Release Date:
Expiration November
Date: December 14,31,
2015
2015; 11:59:59 PM EST
Expiration Date:will
No CME credit Junebe1,awarded
2016 for this activity after December 31, 2015; 11:59:59 PM EST.
No CME credit will be awarded for this activity after June 1, 2016.

© 2015 American Academy of Orthopaedic Surgeons 2015 Orthopaedic In-Training Examination


© 2015 American Academy of Orthopaedic Surgeons 2015 Orthopaedic In-Training Examination
Produced by the American Academy of Orthopaedic Surgeons
Central Evaluation Committee

M. Daniel Wongworawat, MD, Editor in Chief


Professor, Program Director, Orthopaedic Surgery Residency, Loma Linda University Medical Center,
Department of Orthopaedic Surgery, Loma Linda, California
Contributors
Jaimo Ahn, MD
Assistant Professor, Director Medical Student Clerkship, Assistant Director, Residency Program, Co-
Director, Orthopaedic Trauma and Fracture Service, Department of Orthopaedic Surgery, Admissions
and MD-PhD Program Steering Committees, Perelman School of Medicine, University of Pennsylvania,
Philadelphia, Pennsylvania
Joseph Benevenia, MD
Chair, Department of Orthopaedics, Professor, Department of Orthopaedics, Division of Musculoskeletal
Oncology, Rutgers-New Jersey Medical School, Newark, New Jersey
Brandon D. Bushnell, MD, MBA
Vice-Chairman, Department of Orthopaedics and Sports Medicine, Harbin Clinic, LLC, Rome, Georgia;
Assistant Clinical Professor, Department of Orthopaedic Surgery, Medical College of Georgia, Georgia
Regents University, Augusta, Georgia
Samuel J. Chmell, MD
Professor, Department of Orthopaedic Surgery, University of Illinois School of Medicine, Chicago,
Illinois
Robert Hyun Cho, MD
UCLA Clinical Assistant Professor of Orthopaedic Surgery, Interim Chief of Staff, Shriners Hospital for
Children, Los Angeles, California
Michael P. Clare, MD
Director of Fellowship Education, Foot and Ankle Fellowship, Florida Orthopaedic Institute; Affiliate
Associate Professor, Department of Orthopaedic Surgery, University of South Florida, Tampa, Florida
Bruce E. Cohen, MD
Foot and Ankle Fellowship, OrthoCarolina Foot and Ankle Institute, Charlotte, North Carolina
Sheila A. Conway, MD
Program Director, Residency Program and Musculoskeletal Oncology Fellowship, Associate Professor,
Department of Orthopaedic Surgery, University of Miami Miller School of Medicine/Jackson Memorial
Hospital, Miami, Florida
Gregory John Della Rocca, MD, PhD
Associate Professor, Department of Orthopaedic Surgery, Co-director, Orthopaedic Trauma Service,
Department of Orthopaedic Surgery, University of Missouri, Columbia, Missouri
David G. Dennison, MD
Assistant Professor of Orthopaedics, Mayo Clinical College of Medicine, Department of Orthopedic
Surgery, Rochester, Minnesota
Thomas C. Dowd, MD
Associate Residency Director, Orthopaedic Surgery Residency Program, San Antonio Military Medical
Center, Ft. Sam Houston, Texas
Stephen T. Duncan, MD
Assistant Professor, Assistant Program Director, Department of Orthopaedic Surgery, University of
Kentucky School of Medicine, Lexington, Kentucky
Patrick B. Ebeling, MD
Assistant Professor, Department of Orthopedic Surgery, University of Minnesota Medical School,
Minneapolis, Minnesota; Orthopaedic Foot and Ankle Surgeon, Twin Cities Orthopedics, Burnsville,
Minnesota
Michael J. Elliott, MD
Assistant Clinical Professor, University of California, San Francisco School of Medicine, Department of
Orthopedics, Valley Children’s Hospital, Madera, California
Robert J. Esther, MD
Program Director, Department of Orthopaedics, University of North Carolina, Chapel Hill, North
Carolina

© 2015 American Academy of Orthopaedic Surgeons 2015 Orthopaedic In-Training Examination


John C. France, MD
Professor Orthopaedic Surgery, Vice Chairman and Chief of Spine Service, West Virginia University,
Morgantown, West Virginia
Keith R. Gabriel, MD
Pediatric Surgeon-in-Chief, St. John's Children's Hospital; Orthopaedic Residency Program Director,
Southern Illinois University School of Medicine, Springfield, Illinois
Albert O. Gee, MD
Assistant Professor, University of Washington, Department of Orthopaedics & Sports Medicine; Chief,
Sports Medicine & Shoulder Surgery, Puget Sound Veterans Administration, Seattle, Washington
Charles L. Getz, MD
Associate Professor, Department of Orthopaedic Surgery, Thomas Jefferson University Hospital, Division
of Shoulder and Elbow Surgery, Rothman Institute, Philadelphia, Pennsylvania
Shawn R. Gilbert, MD
Associate Professor, Division of Orthopaedic Surgery, The Children’s Hospital of Alabama, University of
Alabama at Birmingham, Birmingham, Alabama
Daniel W. Green, MS, MD, FACS
Professor of Orthopaedic Surgery, Division of Pediatric Orthopaedic Surgery, Weill Cornell Medical
College, Hospital for Special Surgery, New York, New York
Konrad I. Gruson, MD
Assistant Professor, Department of Orthopaedic Surgery, Albert Einstein College of Medicine, Bronx,
New York
Syed Ashfaq Hasan, MD
Associate Professor of Orthopaedic Surgery, Department of Orthopaedics, University of Maryland School
of Medicine, Baltimore, Maryland
William L. Hennrikus, Jr, MD
Professor of Orthopaedics and Rehabilitation, Associate Dean of Education, Medical Director Pediatric
Bone and Joint Center, Penn State College of Medicine, Hershey, Pennsylvania
Kathleen A. Hogan, MD
Orthopaedic Surgeon, New Hampshire Orthopaedic Center, Nashua, New Hampshire
Joshua Langford, MD
Director, Limb Deformity Service; Attending Orthopaedic Traumatologist, Orlando Health Orthopaedic
Residency Program; Instructor of Orthopaedic Surgery, University of Central Florida College of
Medicine, Orlando, Florida; Affiliate Assistant Professor, University of South Florida College of
Medicine, Tampa, Florida
Alfonso Mejia, MD, MPH
Associate Professor of Clinical Orthopedic Surgery, Program Director, Orthopedic Surgery Residency
Program, Vice Head, Department of Orthopedic Surgery, University of Illinois at Chicago College of
Medicine, Chicago, Illinois
Jose A. Morcuende, MD
Marvin and Rose Lee Pomerantz Chair in Orthopaedic Surgery, Professor of Orthopaedic Surgery and
Rehabilitation, Professor of Pediatrics, University of Iowa, Iowa City, Iowa
Varatharaj Mounasamy, MD
Associate Professor, Director of Orthopedic Trauma, Department of Orthopedic Surgery, VCU Health
System, Richmond, Virginia
Fred R. T. Nelson, MD
Emeritus, Department of Orthopaedics, Henry Ford Hospital, Detroit, Michigan
Joseph R. O’Brien, MD, MPH
Associate Professor of Orthopaedic and Neurological Surgery, GWUMC; Director of Minimally Invasive
Spine Surgery, GWUH, Washington, DC
Matthew Oetgen, MD
Assistant Professor, Department of Orthopaedic Surgery and Sports Medicine, Children’s National
Medical Center, Washington, DC
Shervin V. Oskouei, MD
Assistant Professor, Department of Orthopaedic Surgery, Emory University School of Medicine, Atlanta,
Georgia

© 2015 American Academy of Orthopaedic Surgeons 2015 Orthopaedic In-Training Examination


Norman Y. Otsuka, MD
Chief, Pediatric Orthopaedic Surgery, The Children's Hospital at Montefiore; Professor of Clinical
Orthopaedic Surgery Albert Einstein College of Medicine, Bronx, New York
Alpesh A. Patel, MD, FACS
Associate Professor, Department of Orthopaedic Surgery, Northwestern University Feinberg School of
Medicine, Chicago, Illinois
Christopher W. Peer, MD, MS

Richard W. Pope, MD
Orthopaedic Surgeon, Private Practice, Augusta, Georgia
Sheeraz Qureshi, MD
Associate Professor, Orthopaedic Surgery, Minimally Invasive Spinal Surgery, Mount Sinai Hospital
Icahn School of Medicine; Co-Director, Spinal Surgery Fellowship, New York, New York
Timothy Rapp, MD
Associate Professor of Orthopaedic Surgery, Chief, Division of Orthopaedic Oncology NYU Langone
Medical Center, Department of Orthopaedic Surgery, New York, New York
Mark C. Reilly, MD
Chief Orthopaedic Trauma Service, University Hospital, Newark, New Jersey; Associate Professor,
Department of Orthopaedic Surgery, Rutgers-New Jersey Medical School, Newark, New Jersey
Tom E. Reinsel, MD
Assistant Professor of Clinical Orthopaedic Surgery, Department of Orthopaedic Surgery, University of
Missouri-Columbia, Columbia, Missouri
Christopher J. Roach, MD
LTC, MC, Asst. Chief, Orthopaedics, San Antonio Military Medical Center; Asst. Professor, Orthopaedic
Surgery, Uniformed Services University of the Health Sciences, San Antonio, Texas
J. R. Rudzki, MD
Clinical Assistant Professor, Department of Orthopaedic Surgery, The George Washington University
School of Medicine, Washington Orthopaedics and Sports Medicine, Washington, DC
Andrew J. Schoenfeld, MD
Robert Wood Johnson Clinical Scholar and Lecturer, Department of Orthopaedic Surgery, University of
Michigan, Ann Arbor, Michigan
J. Milo Sewards, MD
Program Director, Orthopaedic Surgery Residency, Associate Professor of Orthopaedic Surgery and
Sports Medicine, Team Physician, Temple University School of Medicine, Philadelphia, Pennsylvania
Anshuman Singh, MD
Shoulder and Elbow Surgeon, Department of Orthopaedic Surgery, Kaiser Permanente San Diego,
University of California at San Diego, San Diego, California
Michael J. Taunton, MD
Assistant Professor, College of Medicine, Mayo Clinic, Rochester, Minnesota
Krishna R. Tripuraneni, MD
Attending Physician, New Mexico Orthopaedic Associates, Albuquerque, New Mexico
Wakenda K. Tyler, MD
Associate Professor of Musculoskeletal Oncology, Department of Orthopaedic Surgery, Center for
Musculoskeletal Research and Wilmot Cancer Center, University of Rochester, Rochester, New York
Rahul Vaidya MD
Specialist in Chief Orthopaedic Surgery Detroit Medical Center, Clinical Associate Professor of Surgery
Wayne State University, Detroit, Michigan
Christina M Ward, MD
Assistant Professor, University of Minnesota, Department of Orthopaedic Surgery, Hand and Wrist, St
Paul, Minnesota
Anthony D. Watson, MD
Clinical Assistant Professor, Department of Orthopaedic Surgery, Drexel University School of Medicine,
Pittsburgh, Pennsylvania

© 2015 American Academy of Orthopaedic Surgeons 2015 Orthopaedic In-Training Examination


Peter G. Whang, MD, FACS
Associate Professor, Spine Service, Department of Orthopaedics and Rehabilitation, Yale University
School of Medicine, New Haven, Connecticut
Madhusudhan R Yakkanti, MD
Assistant Professor, Assistant Program Director, Department of Orthopaedic Surgery, University of
Louisville School of Medicine, Louisville, Kentucky

OITE Software Development


Internet Testing Systems, LLC
Baltimore, Maryland

Medical Editing
Brenda Moss Feinberg, ELS

Digital Media Specialist


John Cisco, AAOS Examinations Consultant

American Academy of Orthopaedic Surgeons


Ellen Moore, Chief Education Officer

Department of Electronic Media, Examinations, CME Course Operations,


& Practice Management
Howard Mevis, Director
Jonathan Sprague, MS, Manager, Examinations
Anna M. Scheer, Senior Coordinator, Examinations
Marcie L. Lampert, Senior Coordinator, Examinations
David Marshall, Coordinator & Technology Specialist, Examinations
Irene M. Bogdal, Administrative Assistant, Examinations

© 2015 American Academy of Orthopaedic Surgeons 2015 Orthopaedic In-Training Examination


Disclosure Information for the 2015 Orthopaedic In-Training Examination

Disclosure Information for the 2015 Orthopaedic In-Training Examination

M. Daniel Wongworawat, MD (Chair): Submitted on: 04/02/2015


AAOS: Board or committee member
Alumni Association School of Medicine of Loma Linda University: Board or committee member
Association of Bone and Joint Surgeons: Board or committee member
Clinical Orthopaedics and Related Research: Editorial or governing board
Health Services International: Board or committee member
Neufeld Society (Alumni Association): Board or committee member
San Bernardino County Medical Society: Board or committee member
Jaimo Ahn, MD, PhD: Submitted on: 06/01/2015
AAOS: Board or committee member
American Orthopaedic Association: Board or committee member
American Physician Scientists Association: Board or committee member
Cochrane: Publishing royalties, financial or material support
Foundation for Orthopaedic Trauma: Board or committee member
Fronteirs in Surgery: Editorial or governing board
Journal of Orthopaedic Trauma: Editorial or governing board
NBME: Board or committee member
Orthopaedic Research Society: Board or committee member
Orthopaedic Trauma Association: Board or committee member
Skelegen: Unpaid consultant
Synthes: Paid consultant

Joseph Benevenia, MD (Member): Submitted on: 04/06/2015


AAOS, Musculoskeletal Transplant Foundation, Musculoskeletal Tumor Society: Board or committee member
AAOS/OKU Tumors 2, 3, Musculoskeletal Tumor Society: Editorial or governing board
Biomet: Research support
Creosso LLC: Stock or stock Options
Implant Cast: Unpaid consultant
MereteNJOS: Unpaid consultant
Musculoskeletal Transplant Foundation: Paid presenter or speaker
Musculoskeletal Transplant Foundation, Synthes: Research support
Rutgers University/Creosso LLC: IP royalties
Brandon Dubose Bushnell, MBA, MD (Member): Submitted on: 07/28/2015
AAOS: Board or committee member
Arthroscopy Association of North America: Board or committee member
Mitek: Paid consultant
Rotation Medical: Research support; Unpaid consultant
Southern Medical Journal: Editorial or governing board

Samuel J Chmell, MD (Member): Submitted on: 04/03/2015


AAOS: Board or committee member
Iroko Pharmaceuticals: Paid consultant
Pacira Pharmaceuticals: Paid consultant
Robert Hyun Cho, MD (Member): Submitted on: 04/30/2015
DePuy Spine: Paid consultant
Medtronic Sofamor Danek: Paid consultant
OrthoPediatrics: Paid consultant
Orthopedics: Editorial or governing board

Michael Patrick Clare, MD (Member): Submitted on: 04/09/2015


BESPA: Paid consultant

© 2015 American Academy of Orthopaedic Surgeons 2015 Orthopaedic In-Training Examination


Disclosure Information for the 2015 Orthopaedic In-Training Examination

Disclosure Information for the 2015 Orthopaedic In-Training Examination

Bruce E Cohen, MD: Submitted on: 07/02/2015


Acumed, LLC: Research support
Amniox: Paid consultant
Arthrex, Inc: IP royalties; Research support
DJ Orthopaedics: IP royalties
Tissue Tech: Stock or stock Options
Wright Medical Technology, Inc.: IP royalties; Paid consultant

Sheila Ann Conway, MD (Member): (This individual reported nothing to disclose); Submitted on: 07/22/2015

Gregory John Della Rocca, MD, PhD: Submitted on: 05/05/2015


AAOS: Board or committee member
Amedica: Stock or stock Options
American College of Surgeons: Board or committee member
Bioventus: Paid consultant
Geriatric Orthopaedic Surgery and Rehabilitation: Editorial or governing board
Journal of Bone and Joint Surgery - American: Editorial or governing board
Journal of Orthopaedic Trauma: Editorial or governing board
Journal of the American Academy of Orthopaedic Surgeons: Editorial or governing board
LifeNet: Paid consultant
MergeNet: Stock or stock Options
Orthopaedic Trauma Association: Board or committee member
Pacira: Paid consultant
Synthes: Paid consultant; Paid presenter or speaker; Research support
The Orthopaedic Implant Company: Stock or stock Options

David G Dennison, MD (Member): Submitted on: 08/31/2015


AAOS: Board or committee member
American Society for Surgery of the Hand: Board or committee member
AO - instructor for AO hand fracture course: Paid presenter or speaker
DePuy, A Johnson & Johnson Company: Research support
Mid America Ortho Assoc Finance Committee: Board or committee member

Thomas Charles Dowd, MD (Member): Submitted on: 08/31/2015


American Orthopaedic Association: Board or committee member
American Orthopaedic Foot and Ankle Society: Board or committee member
Zimmer: Research support

Stephen Thomas Duncan, MD (Member): Submitted on: 05/30/2015


American Association of Hip and Knee Surgeons: Board or committee member
Journal of Arthroplasty: Editorial or governing board
Mitek: Paid consultant
Smith & Nephew: Paid consultant

Patrick Brian Ebeling, MD (Member): Submitted on: 04/06/2015


AAOS: Board or committee member
American Orthopaedic Foot and Ankle Society: Board or committee member
Foot and Ankle International: Editorial or governing board
Journal of Bone and Joint Surgery - American: Editorial or governing board

Michael Johnathon Elliott, MD (Member): (This individual reported nothing to disclose); Submitted on: 09/01/2015

Robert J Esther, MD: Submitted on: 05/01/2015


Musculoskeletal Transplant Foundation: Other financial or material support

© 2015 American Academy of Orthopaedic Surgeons 2015 Orthopaedic In-Training Examination


Disclosure Information for the 2015 Orthopaedic In-Training Examination

Disclosure Information for the 2015 Orthopaedic In-Training Examination

John C France, MD: Submitted on: 04/04/2015


AAOS: Board or committee member
Cervical Spine Research Society committee Scoliosis research Society education committee AAOS spine program
committee: Board or committee member

Keith Robert Gabriel, MD: Submitted on: 04/29/2014


American Academy of Pediatrics Section on Orthopaedics: Board or committee member
American Orthopaedic Association: Board or committee member
Pediatric Orthopaedic Society of North America: Board or committee member
State of Illinois Division of Specialized Care for Children: Board or committee member

Albert Ooguen Gee, MD (Member): Submitted on: 05/17/2015


Medbridge Inc.: Publishing royalties, financial or material support

Charles L Getz, MD: Submitted on: 04/15/2015


Cayenne Medical: Paid consultant
Integra: Research support
Mitek: Paid presenter or speaker
OBERD: Stock or stock Options
Rotation Medical: Research support
Zimmer: Research support; Unpaid consultant

Shawn R Gilbert, MD: Submitted on: 04/20/2015


AAOS POSNA: Board or committee member

Daniel William Green, MD: Submitted on: 04/16/2015


AAOS: Board or committee member
Arthrex, Inc: IP royalties; Paid presenter or speaker
Current Opinion in Pediatrics: Editorial or governing board; Publishing royalties, financial or material support
New York County Medical Society: Board or committee member
New York State Society of Orthopedic Surgeons: Board or committee member
Pediatric Orthopaedic Society of North America: Board or committee member
Pega Medical: IP royalties
Wolters Kluwer Health - Lippincott Williams & Wilkins: Publishing royalties, financial or material support

Konrad Izumi Gruson, MD (Member): Submitted on: 05/09/2015


AAOS: Board or committee member
Amgen Co: Stock or stock Options
Bristol-Myers Squibb: Stock or stock Options
Eli Lilly: Stock or stock Options
GlaxoSmithKline: Stock or stock Options
Johnson & Johnson: Stock or stock Options
Medtronic: Stock or stock Options
Merck: Stock or stock Options
Pfizer: Stock or stock Options
Procter & Gamble: Stock or stock Options
Stryker: Stock or stock Options
Zimmer: Stock or stock Options

Syed Ashfaq Hasan, MD: Submitted on: 05/31/2015


AAOS: Board or committee member

© 2015 American Academy of Orthopaedic Surgeons 2015 Orthopaedic In-Training Examination


Disclosure Information for the 2015 Orthopaedic In-Training Examination

Disclosure Information for the 2015 Orthopaedic In-Training Examination

William L Hennrikus Jr, MD: Submitted on: 05/26/2015


Pediatric Orthopaedic Society of North America: Board or committee member
Society of Military Orthopaedic Surgeons: Board or committee member

Kathleen Anne Hogan, MD: (This individual reported nothing to disclose); Submitted on: 07/21/2014

Joshua Langford, MD (Member): Submitted on: 06/01/2015


Advanced Orhopaedic Solutions: IP royalties
Core Orthopaedics: Stock or stock Options
Institute for Better Bone Health, LLC: Stock or stock Options
Smith & Nephew: Paid presenter or speaker
Stryker: Paid consultant

Alfonso Mejia, MD, MPH: Submitted on: 05/27/2015


AAOS, AALOS, IAOS, CMS, ISMS: Board or committee member
Acumed, LLC: Research support
Arthrex, Inc: Research support
Smith & Nephew: Research support
Synthes: Research support

Jose A Morcuende, MD: Submitted on: 05/29/2015


AAOSUSBJIPOSNA: Board or committee member
Clubfoot Solutions: Unpaid consultant

Varatharaj Mounasamy, MD (Member): Submitted on: 04/09/2015


European journal of orthopedic surgery and traumatology: Editorial or governing board

Fred R T Nelson, MD (Member): Submitted on: 04/07/2015


206Ortho: Paid consultant
Bioiberica: Paid consultant; Paid presenter or speaker
Elsevier for book royalty: Publishing royalties, financial or material support

Joseph R O'Brien, MD: Submitted on: 02/21/2015


Bioset: Research support
Globus: IP royalties; Paid consultant; Research support
K2M: Stock or stock Options
Nuvasive: IP royalties; Paid consultant; Research support
Relivant: Paid consultant
RTI: Paid consultant; Stock or stock Options
Spinicity / ISD: Stock or stock Options
Stryker: Paid consultant

Matthew Oetgen, MD (Member): Submitted on: 04/03/2015


AAOS: Board or committee member
Pediatric Orthopaedic Society of North America: Board or committee member
Scoliosis Research Society: Board or committee member

Shervin V Oskouei, MD (Member): Submitted on: 09/26/2014


Georgia Orthopaedic Society: Board or committee member

© 2015 American Academy of Orthopaedic Surgeons 2015 Orthopaedic In-Training Examination


Disclosure Information for the 2015 Orthopaedic In-Training Examination

Disclosure Information for the 2015 Orthopaedic In-Training Examination

Norman Yoshinobu Otsuka, MD (Member): Submitted on: 04/23/2015


AAOS: Board or committee member
American Academy of Pediatrics: Board or committee member
American College of Surgeons: Board or committee member
American Journal of Orthopedics: Editorial or governing board
Journal of Children's Orthopaedics: Editorial or governing board
Journal of Orthopaedic Surgical Advances: Editorial or governing board
Journal of Pediatric Orthopedics, Part B: Editorial or governing board
Medsonics: Unpaid consultant
Pediatric Orthopaedic Society of North America: Board or committee member

Alpesh Ashwin Patel, MD: Submitted on: 04/05/2015


AAOS: Board or committee member
Amedica: IP royalties; Paid consultant; Stock or stock Options
American College of Surgeons: Board or committee member
American Orthopaedic Association: Board or committee member
AO Spine North America: Board or committee member
Biomet: IP royalties; Paid consultant
Cervical Spine Research Society: Board or committee member
Cytonics: Stock or stock Options
DePuy, A Johnson & Johnson Company: Paid consultant
Journal of the American Academy of Orthopaedic Surgeons: Editorial or governing board
Nocimed: Stock or stock Options
North American Spine Society: Board or committee member
Relievant: Paid consultant
Springer: Publishing royalties, financial or material support
Stryker: Paid consultant
Surgical Neurology International: Editorial or governing board
Trinity Orthopaedics: Stock or stock Options
Ulrich Medical USA: IP royalties
Wolters Kluwer Health - Lippincott Williams & Wilkins: Editorial or governing board

Christopher William Peer, MD, MS (Member): (This individual reported nothing to disclose); Submitted on:
04/01/2014

Richard W Pope, MD (Member): Submitted on: 08/31/2015


AAOS Evaluations Committee (Hand/Wrist) starting in MAR11: Board or committee member

Sheeraz Qureshi, MD (Member): Submitted on: 04/25/2015


AAOS: Board or committee member
Cervical Spine Research Society: Board or committee member
Clinical Orthopaedics and Related Research: Editorial or governing board
Contemporary Spine Surgery: Editorial or governing board
Global Spine Journal: Editorial or governing board
Globus Medical: Paid presenter or speaker
Medtronic: Paid consultant
Medtronic Sofamor Danek: Paid presenter or speaker
Musculoskeletal Transplant Foundation: Board or committee member
NASS: Board or committee member
Orthofix, Inc.: Paid consultant
Spine (reviewer): Editorial or governing board
Spine Journal (reviewer): Editorial or governing board
Stryker: Paid consultant; Paid presenter or speaker
Zimmer: IP royalties; Paid consultant

© 2015 American Academy of Orthopaedic Surgeons 2015 Orthopaedic In-Training Examination


Disclosure Information for the 2015 Orthopaedic In-Training Examination

Disclosure Information for the 2015 Orthopaedic In-Training Examination

Timothy Rapp, MD (Member): Submitted on: 05/01/2015


AAOS: Board or committee member
American Board of Orthopaedic Surgery, Inc.: Board or committee member
Clinical Orthopaedics and Related Research Bulletin of the NYU Hospital for Joint Diseases American Journal of
Orthopaedics: Editorial or governing board
Research or Institutional Support-- Department of Orthopaedic Surgery Hospital for Joint Diseases at NYU
Langone Medical Center: AO Spine Arthrex Arthritis Foundation– NY Chapter Arthritis National Research
Foundation Asterland Biomet Depuy Encore Exactech/ DJO Ferring Pharmaceuticals Geisinger Integra Johnson &
Johnson KCI Medtronic NIHOMEGAOREF Orthopaedic Trauma Association Osteosynthesis and Trauma Care
Foundation Paradigm Spine Progenics Sbi Smith and Nephew Stryker Surgix Synthes: Research support

Mark C Reilly, MD (Member): Submitted on: 05/22/2015


Stryker: Paid consultant; Paid presenter or speaker

Tom E Reinsel, MD: Submitted on: 08/31/2015


Missouri State Orthopedic Ass'n (secretary): Board or committee member
Spine: Editorial or governing board
Spineology: Research support
The Spine Journal (only a reviewer): Editorial or governing board

Christopher James Roach, MD (Member): Submitted on 04/08/2015


AAOS Board or committee member

Jonas R Rudzki, MD (Member): Submitted on: 05/04/2015


AAOS Evaluation Committee & BOC, Washington Orthopaedic Society: Board or committee member
Arthrex, Inc: Paid consultant; Paid presenter or speaker

Andrew J Schoenfeld, MD: Submitted on: 04/02/2015


AAOS: Board or committee member
North American Spine Society: Board or committee member
Saunders/Mosby-Elsevier - The Spine Journal: Editorial or governing board
Wolters Kluwer Health - Lippincott Williams & Wilkins: Publishing royalties, financial or material support

Joseph Milo Sewards, MD (Member): Submitted on: 05/01/2015


Aesculap/B.Braun: Research support

Anshuman Singh, MD (Member): (This individual reported nothing to disclose); Submitted on: 04/02/2015

Michael J Taunton, MD: Submitted on: 08/19/2015


AAOS: Board or committee member
DJ Orthopaedics: Paid consultant
Journal of Arthroplasty: Editorial or governing board
Minnesota Orthopedic Society: Board or committee member
Stryker: Research support

Krishna Raj Tripuraneni, MD (Member): Submitted on: 04/05/2015


Arthroplasty Today: Editorial or governing board
DJO Surgical: Research support
Journal of Arthroplasty: Editorial or governing board
Orthopaedic Implant Company: Stock or stock Options
Orthoview: Paid consultant

© 2015 American Academy of Orthopaedic Surgeons 2015 Orthopaedic In-Training Examination


Disclosure Information for the 2015 Orthopaedic In-Training Examination

Disclosure Information for the 2015 Orthopaedic In-Training Examination

Wakenda K Tyler, MD, MPH: Submitted on: 04/09/2015


AAOS: Board or committee member
J. Robert Gladden Society: Board or committee member
Musculoskeletal Transplant Foundation: Unpaid consultant
Musculoskeletal Tumor Society: Board or committee member

Rahul Vaidya, MD (Member): Submitted on: 05/25/2015


European Spine Journal: Editorial or governing board
Smith & Nephew: IP royalties
Stryker: Paid consultant; Paid presenter or speaker; Unpaid consultant
Synthes: IP royalties; Other financial or material support; Paid presenter or speaker; Research support

Christina M Ward, MD (LFP Member): (This individual reported nothing to disclose); Submitted on: 05/02/2015

Anthony D Watson, MD: Submitted on: 04/02/2015


Foot and Ankle International: Editorial or governing board

Peter G Whang, MD (Member): Submitted on: 05/01/2015


Cerapedics: Paid consultant
DiFusion: Stock or stock Options; Unpaid consultant
Histogenics: Paid consultant
Medtronic: Paid consultant; Paid presenter or speaker
Pacira: Paid consultant
Pacira Pharmaceuticals: Paid presenter or speaker
Relievant: Paid consultant
SI Bone: Other financial or material support; Paid consultant
Spinal Kinetics: Other financial or material support
Spine: Editorial or governing board
Stryker: Paid consultant; Paid presenter or speaker
Vertiflex: Other financial or material support

Madhusudhan Reddy Yakkanti, MD (Member): Submitted on: 04/02/2015


Synthes- Received honorarium for participation as a table instructor in a shoulder course sponsored by
Synthes: Other financial or material support

AMERICAN ACADEMY OF ORTHOPAEDIC SURGEONS STAFF


Howard Mevis: Submitted on: 10/01/2015
Bristol-Myers Squibb: Stock or stock Options
GE Healthcare: Stock or stock Options
Orthopaedic Learning Center: Board or committee member

Jonathan Sprague, MS (Staff Liaison): (This individual reported nothing to disclose); Submitted on: 06/05/2015

© 2015 American Academy of Orthopaedic Surgeons 2015 Orthopaedic In-Training Examination


Disclaimer

The goals of the in-training examination are: 1) to provide a way for exam participants to assess personal
knowledge and compare scores with peers; and 2) to permit faculty to evaluate the strengths and weaknesses
of training programs as indicated by exam results. Residency training programs should not use examination
results as a basis for dismissal of a resident from the program.

The material presented in this in-training examination has been made available by the American Academy of
Orthopaedic Surgeons for educational purposes only. This material is not intended to present the only, or
necessarily the best, methods or procedures for the medical situations discussed, but rather is intended to
represent an approach, view, statement, or opinion of the author(s) or producer(s), which may be helpful to
others who face similar situations.

Some drugs or medical devices demonstrated in Academy courses or described in Academy print or
electronic publications have not been cleared by the Food and Drug Administration (FDA) or have been
cleared for specific uses only. The FDA has stated that it is the responsibility of the physician to determine
the FDA clearance status of each drug or device he or she wishes to use in clinical practice.

Furthermore, any statements about commercial products are solely the opinion(s) of the author(s) and do not
represent an Academy endorsement or evaluation of these products. These statements may not be used in
advertising or for any commercial purpose.

The Orthopaedic In-Training Examination® (OITE) is the intellectual property of the American Academy of
Orthopaedic Surgeons. A non-exclusive license to use OITE and its content does not confer on the user any
ownership rights, including but not limited to copyright.

© 2015 American Academy of Orthopaedic Surgeons 2015 Orthopaedic In-Training Examination


© 2015 American Academy of Orthopaedic Surgeons 2015 Orthopaedic In-Training Examination
2015 Orthopaedic In-Training Examination

TABLE OF CONTENTS

EXAMINATION QUESTIONS Page


SECTION 1: Pediatrics....................................................................................................... 21
SECTION 2: Trauma.......................................................................................................... 54
SECTION 3: Hand .............................................................................................................. 103
SECTION 4: Hip and Knee ................................................................................................ 118
SECTION 5: Adult Spine ................................................................................................... 137
SECTION 6: Foot and Ankle ............................................................................................. 159
SECTION 7: Sports Medicine ............................................................................................ 181
SECTION 8: Shoulder and Elbow...................................................................................... 202
SECTION 9: Oncology....................................................................................................... 226
SECTION 10: Basic Science ................................................................................................ 247

APPENDICES
Score Key ............................................................................................................................... 269
Content Domain Item Numbers............................................................................................. 270

 2013 American Academy of Orthopaedic Surgeons 2013 Orthopaedic In-Training Examination


© 2015 American Academy of Orthopaedic Surgeons 2015 Orthopaedic In-Training Examination
© 2015 American Academy of Orthopaedic Surgeons 2015 Orthopaedic In-Training Examination
SECTION 1: Pediatrics • 21

SECTION 1: Pediatrics

Figure 3

Question 3
Figure 3 is the radiograph of a 3-year-old boy who has a limb deformity. This condition should be further
investigated with

1. laboratory studies.
2. skin biopsy.
3. knee CT scan.
4. knee MRI.
5. spine radiographs.

PREFERRED RESPONSE: 1

RECOMMENDED READINGS
Fucentese SF, Neuhaus TJ, Ramseier LE, Ulrich Exner G. Metabolic and orthopedic management of
X-linked vitamin D-resistant hypophosphatemic rickets. J Child Orthop. 2008 Aug;2(4):285-91. doi:
10.1007/s11832-008-0118-9. Epub 2008 Jul 26. PubMed PMID: 19308556; PubMed Central PMCID:
PMC2656824.

Choi IH, Kim JK, Chung CY, Cho TJ, Lee SH, Suh SW, Whang KS, Park HW, Song KS. Deformity
correction of knee and leg lengthening by Ilizarov method in hypophosphatemic rickets: outcomes and
significance of serum phosphate level. J Pediatr Orthop. 2002 Sep-Oct;22(5):626-31. PubMed PMID:
12198465.
© 2015 American Academy of Orthopaedic Surgeons 2015 Orthopaedic In-Training Examination
22 • American Academy of Orthopaedic Surgeons

Video 13 (Scan to view)

Question 13
Video 13 shows axial MR images of a 15-year-old boy who experienced pain in his left hip while playing
football 2 days ago. Radiographs of his pelvis did not demonstrate any noticeable abnormalities; however,
he has persistent pain and a limp. What is the best next step?

1. Immediate pelvic CT scan


2. Open reduction and internal fixation of the injury
3. Urgent in situ proximal femoral physeal stabilization
4. Referral to hematology for evaluation of possible occult blood dyscrasia
5. Activity restriction and partial weight bearing with crutches until symptoms resolve

PREFERRED RESPONSE: 5

RECOMMENDED READINGS
McKinney BI, Nelson C, Carrion W. Apophyseal avulsion fractures of the hip and pelvis. Orthopedics.
2009 Jan;32(1):42. Review. PubMed PMID: 19226032.

Kocher MS, Tucker R. Pediatric athlete hip disorders. Clin Sports Med. 2006 Apr;25(2):241-53, viii.
Review. PubMed PMID: 16638489.

© 2015 American Academy of Orthopaedic Surgeons 2015 Orthopaedic In-Training Examination


SECTION 1: Pediatrics • 23

Figure 21a Figure 21b

Question 21
Figures 21a and 21b are the radiographs of a 15-year-old girl who fell on her right elbow and now has
pain. Treatment should consist of

1. splinting for comfort.


2. ligament reconstruction.
3. open reduction and internal fixation.
4. closed reduction and immobilization.
5. closed reduction and percutaneous pinning.

PREFERRED RESPONSE: 1

RECOMMENDED READINGS
Cleary JE, Omer GE Jr. Congenital proximal radio-ulnar synostosis. Natural history and functional
assessment. J Bone Joint Surg Am. 1985 Apr;67(4):539-45. PubMed PMID: 3980498.

Kozin SH. Congenital differences about the elbow. Hand Clin. 2009 May;25(2):277-91. doi: 10.1016/j.
hcl.2008.12.007. Review. PubMed PMID: 19380066.

© 2015 American Academy of Orthopaedic Surgeons 2015 Orthopaedic In-Training Examination


24 • American Academy of Orthopaedic Surgeons

Question 29
What is the most common type of neonatal brachial plexus palsy?

1. Suprascapular
2. Klumpke palsy
3. Upper trunk injury
4. Horner syndrome
5. Global plexus injury at C6 and T1

PREFERRED RESPONSE: 3

RECOMMENDED READINGS
Zlotolow DA, Kozin SH. Upper extremity disorders: pediatrics. In: Flynn JM, ed. Orthopaedic Knowledge
Update 10. Rosemont, IL: American Academy of Orthopaedic Surgeons; 2011:697-713.

Lagerkvist AL, Johansson U, Johansson A, Bager B, Uvebrant P. Obstetric brachial plexus palsy: a
prospective, population-based study of incidence, recovery, and residual impairment at 18 months of age.
Dev Med Child Neurol. 2010 Jun;52(6):529-34. doi: 10.1111/j.1469-8749.2009.03479.x. Epub 2009 Dec
23. PubMed PMID: 20041937.

© 2015 American Academy of Orthopaedic Surgeons 2015 Orthopaedic In-Training Examination


SECTION 1: Pediatrics • 25

Figure 33a Figure 33b

Question 33
A 2-year-old boy sustained the fracture shown in Figures 33a and 33b. Closed reduction was successful,
but a crossed pin technique was required to ensure fracture stability. In the recovery room, the boy could
not actively cross his fingers or perform a “scissors” movement with his fingers. What is the most likely
cause for the dense nerve palsy?

1. “Tenting” of the ulnar nerve over the medial pin


2. Penetration of the ulnar nerve by the proximal tip of the lateral pin
3. Penetration of the radial nerve by the proximal tip of the medial pin
4. Compression of the anterior interosseous nerve attributable to elbow swelling
5. Compartment syndrome of the forearm

PREFERRED RESPONSE: 1

RECOMMENDED READINGS
Ozçelik A, Tekcan A, Omeroğlu H. Correlation between iatrogenic ulnar nerve injury and angular
insertion of the medial pin in supracondylar humerus fractures. J Pediatr Orthop B. 2006 Jan;15(1):58-61.
PubMed PMID: 16280722.

Shim JS, Lee YS. Treatment of completely displaced supracondylar fracture of the humerus in children
by cross-fixation with three Kirschner wires. J Pediatr Orthop. 2002 Jan-Feb;22(1):12-6. PubMed PMID:
11744846.

Skaggs DL, Hale JM, Bassett J, Kaminsky C, Kay RM, Tolo VT. Operative treatment of supracondylar
fractures of the humerus in children. The consequences of pin placement. J Bone Joint Surg Am. 2001
May;83-A(5):735-40. PubMed PMID: 11379744.

Herring JA. Upper extremity injuries. In: Herring JA, ed. Tachdjian’s Pediatric Orthopaedics, 3rd ed.
Philadelphia, PA: WB Saunders, 2002; 2115-2250. 
© 2015 American Academy of Orthopaedic Surgeons 2015 Orthopaedic In-Training Examination
26 • American Academy of Orthopaedic Surgeons

Figure 38

Question 38
Figure 38 is the radiograph of a 12-year-old boy who has left foot pain. Examination of the foot will reveal

1. clawing of the toes.


2. cavus deformity of the midfoot.
3. weakness of the intrinsic muscles.
4. decreased ankle range of motion.
5. decreased subtalar joint range of motion.

PREFERRED RESPONSE: 5

RECOMMENDED READINGS
Vincent KA. Tarsal coalition and painful flatfoot. J Am Acad Orthop Surg. 1998 Sep-Oct;6(5):274-81.
Review. PubMed PMID: 9753754.

Mubarak SJ, Patel PN, Upasani VV, Moor MA, Wenger DR. Calcaneonavicular coalition:
treatment by excision and fat graft. J Pediatr Orthop. 2009 Jul-Aug;29(5):418-26. doi: 10.1097/
BPO.0b013e3181aa24c0. PubMed PMID: 19568010. 

© 2015 American Academy of Orthopaedic Surgeons 2015 Orthopaedic In-Training Examination


SECTION 1: Pediatrics • 27

Question 43
Which bacterial organism most commonly causes pediatric septic arthritis?

1. Kingella kingae (K. kingae)


2. Escherichia coli (E. coli)
3. Staphylococcus aureus (S. aureus)
4. Haemophilus influenzae (H. influenzae)
5. Borrelia burgdorferi (B. burgdorferi)

PREFERRED RESPONSE: 3

RECOMMENDED READINGS
Salava JK, Springer BD. Orthopaedic infections. In: Cannada LK, ed. Orthopaedic Knowledge Update 11.
Rosemont, IL: American Academy of Orthopaedic Surgeons; 2014:287-306.

Young TP, Maas L, Thorp AW, Brown L. Etiology of septic arthritis in children: an update for the new
millennium. Am J Emerg Med. 2011 Oct;29(8):899-902. doi: 10.1016/j.ajem.2010.04.008. Epub 2010 Aug
1. PubMed PMID: 20674219.

Question 49
What is the most effective initial treatment to improve an elbow flexion contracture exceeding 40 degrees
in a child with underlying brachial plexus palsy?

1. Serial casting
2. Biceps brachii tendon transfer
3. Arthroscopic elbow capsular release
4. Full-time elbow extension splinting
5. Nighttime elbow extension splinting

PREFERRED RESPONSE: 1

RECOMMENDED READINGS
Ho ES, Roy T, Stephens D, Clarke HM. Serial casting and splinting of elbow contractures in children with
obstetric brachial plexus palsy. J Hand Surg Am. 2010 Jan;35(1):84-91. doi: 10.1016/j.jhsa.2009.09.014.
Epub 2009 Dec 3. PubMed PMID: 19959298.

Sheffler LC, Lattanza L, Hagar Y, Bagley A, James MA. The prevalence, rate of progression, and
treatment of elbow flexion contracture in children with brachial plexus birth palsy. J Bone Joint Surg
Am. 2012 Mar 7;94(5):403-9. doi: 10.2106/JBJS.J.00750. PubMed PMID: 22398733; PubMed Central
PMCID: PMC3284859. 

© 2015 American Academy of Orthopaedic Surgeons 2015 Orthopaedic In-Training Examination


28 • American Academy of Orthopaedic Surgeons

Figure 56

Question 56
Figure 56 is the lateral radiograph of an 11-year-old boy who has back pain. Based on this radiographic
finding, what is the most likely cause of his back pain?

1. Trauma
2. Infection
3. Metabolic disease
4. Congenital anomaly
5. Developmental anomaly

PREFERRED RESPONSE: 2

RECOMMENDED READINGS
Jain AK. Tuberculosis of the spine: a fresh look at an old disease. J Bone Joint Surg Br. 2010
Jul;92(7):905-13. doi: 10.1302/0301-620X.92B7.24668. Review. PubMed PMID: 20595106.

Chunguang Z, Limin L, Rigao C, Yueming S, Hao L, Qingquan K, Quan G, Tao L, Jiancheng Z. Surgical
treatment of kyphosis in children in healed stages of spinal tuberculosis. J Pediatr Orthop. 2010 Apr-
May;30(3):271-6. doi: 10.1097/BPO.0b013e3181d39899. PubMed PMID: 20357594.

© 2015 American Academy of Orthopaedic Surgeons 2015 Orthopaedic In-Training Examination


SECTION 1: Pediatrics • 29

Figure 64a Figure 64b

Question 64
Figures 64a and 64b are the radiographs of a 1-year-old girl who underwent bilateral reconstructive
surgery for a similar bilateral hand deformity. During embryogenesis, there was a problem with control
of the anteroposterior (AP) axis (thumb to small finger, great toe to small toe) in all of her developing
limb buds. The AP axis, which is under the control of an area of tissue in the posterior aspect of the apical
ectodermal ridge, is known as the

1. area for preaxial focus (APF).


2. area of digital specification (ADS).
3. zone of polarizing activity (ZPA).
4. zone of axial determination (ZAD).
5. sonic hedgehog zone (SHH).

PREFERRED RESPONSE: 3

RECOMMENDED READINGS
Herring JA. Disorders of the upper extremity. In: Herring JA, ed. Tachdjian’s Pediatric Orthopaedics, 3rd
ed. Philadelphia, PA: WB Saunders; 2002:379-512.

Waters PM. The upper limb. In: Morrissy RT, Weinstein SL, eds. Lovell and Winter’s Pediatric
Orthopaedics. 6th ed. Philadelphia, PA: Lippincott Williams & Wilkins; 2006:921-986.

© 2015 American Academy of Orthopaedic Surgeons 2015 Orthopaedic In-Training Examination


30 • American Academy of Orthopaedic Surgeons

Figure 70a Figure 70b

Question 70
Figures 70a and 70b are the pelvic radiographs of an 11-year-old boy who has right hip pain. The
alignment of the right limb is

1. flexed.
2. adducted.
3. abducted.
4. internally rotated.
5. externally rotated.

PREFERRED RESPONSE: 5

RECOMMENDED READINGS
Peck K, Herrera-Soto J. Slipped capital femoral epiphysis: what’s new? Orthop Clin North Am. 2014
Jan;45(1):77-86. doi: 10.1016/j.ocl.2013.09.002. Review. PubMed PMID: 24267209.

Schoenecker PL, Gordon JE, Luhmann SJ, Dobbs MB, Keeler KA, Clohisy JC. A treatment algorithm for
stable slipped capital femoral epiphysis deformity. J Pediatr Orthop. 2013 Jul-Aug;33 Suppl 1:S103-11.
doi: 10.1097/BPO.0b013e31829774d6. PubMed PMID: 23764781.

© 2015 American Academy of Orthopaedic Surgeons 2015 Orthopaedic In-Training Examination


SECTION 1: Pediatrics • 31

Figure 74a Figure 74b

Question 74
Figures 74a and 74b are the clinical photographs of a newborn infant who has unilateral right arm
swelling, hypoperfusion, and skin desquamation after a prolonged labor. It was noted that the right
arm was adducted and flexed at the elbow behind the infant’s back during delivery. What is the most
appropriate next step?

1. Neonatal sepsis workup


2. Hematology evaluation for a bleeding disorder
3. Elevate and ice the arm with a repeat exam in 2 hours
4. Emergent fasciotomy for neonatal compartment syndrome
5. Local wound care and immediate compression dressings for edema control

PREFERRED RESPONSE: 4

RECOMMENDED READINGS
Ragland R 3rd, Moukoko D, Ezaki M, Carter PR, Mills J. Forearm compartment syndrome in the
newborn: report of 24 cases. J Hand Surg Am. 2005 Sep;30(5):997-1003. PubMed PMID: 16182057.

Allen LM, Benacci JC, Trane RN 3rd, Driscoll RJ. A case of neonatal compartment syndrome: importance
of early diagnosis in a rare and debilitating condition. Am J Perinatol. 2010 Feb;27(2):103-6. doi: 10.1055/
s-0029-1224870. Epub 2009 Jun 5. PubMed PMID: 19504429.

© 2015 American Academy of Orthopaedic Surgeons 2015 Orthopaedic In-Training Examination


32 • American Academy of Orthopaedic Surgeons

Question 81
Marfan syndrome is caused by a defect in the gene that encodes for which protein?

1. Fibrillin-1
2. Fibrillin-2
3. Neurofibromin
4. Type I collagen
5. Type III collagen

PREFERRED RESPONSE: 1

RECOMMENDED READINGS
Shirley ED, Sponseller PD. Marfan syndrome. J Am Acad Orthop Surg. 2009 Sep;17(9):572-81. Review.
PubMed PMID: 19726741.

Dean JC. Marfan syndrome: clinical diagnosis and management. Eur J Hum Genet. 2007 Jul;15(7):724-
33. Epub 2007 May 9. Review. PubMed PMID: 17487218.

© 2015 American Academy of Orthopaedic Surgeons 2015 Orthopaedic In-Training Examination


SECTION 1: Pediatrics • 33

Figure 86a Figure 86b

Question 86
Figures 86a and 86b are the radiographs of an infant who has a leg deformity. Treatment for this deformity
should include

1. amputation.
2. observation.
3. osteotomy at 1 year of age.
4. osteotomy at 4 years of age
5. guided tibia growth at 8 years of age.

PREFERRED RESPONSE: 2

RECOMMENDED READINGS
Shah HH, Doddabasappa SN, Joseph B. Congenital posteromedial bowing of the tibia: a retrospective
analysis of growth abnormalities in the leg. J Pediatr Orthop B. 2009 May;18(3):120-8. doi: 10.1097/
BPB.0b013e328329dc86. PubMed PMID: 19339901.

Pappas AM. Congenital posteromedial bowing of the tibia and fibula. J Pediatr Orthop. 1984
Sep;4(5):525-31. PubMed PMID: 6490868.

© 2015 American Academy of Orthopaedic Surgeons 2015 Orthopaedic In-Training Examination


34 • American Academy of Orthopaedic Surgeons

Figure 92a Figure 92b

Question 92
Figures 92a and 92b are the radiographs of a 5-year-old boy who was treated at birth in a Pavlik harness
for a right hip dislocation. Since that time, he has developed typically, has had no hip pain, and has a
typical gait. The left acetabular index is 15 degrees and the right is 31 degrees. What is the best next step?

1. Continued observation
2. Right hip Pemberton osteotomy
3. Right hip proximal femoral varus derotational osteotomy
4. Bilateral hip Pemberton osteotomies
5. Nighttime abduction bracing

PREFERRED RESPONSE: 2

RECOMMENDED READINGS
Faciszewski T, Kiefer GN, Coleman SS. Pemberton osteotomy for residual acetabular dysplasia in
children who have congenital dislocation of the hip. J Bone Joint Surg Am. 1993 May;75(5):643-9.
PubMed PMID: 8501078.

Gillingham BL, Sanchez AA, Wenger DR. Pelvic osteotomies for the treatment of hip dysplasia in
children and young adults. J Am Acad Orthop Surg. 1999 Sep-Oct;7(5):325-37. Review. PubMed PMID:
10504359.

© 2015 American Academy of Orthopaedic Surgeons 2015 Orthopaedic In-Training Examination


SECTION 1: Pediatrics • 35

Question 95
A 2-month-old infant with developmental dysplasia of the left hip that is being treated with a Pavlik
harness is unable to kick his leg. What is the best next step?

1. Reduce hip flexion to 90 degrees but continue the harness with closer follow-up for return of
function of the left lower extremity.
2. Continue the harness at daytime only and closely follow the infant for return of function of the
left lower extremity.
3. Discontinue the harness completely and closely follow the infant for return of function of the
left lower extremity.
4. Discontinue the harness and convert to closed reduction and hip spica casting of the
affected side.
5. Order MR images of the spine to assess for potential spinal dysraphism.

PREFERRED RESPONSE: 3

RECOMMENDED READINGS
Schoenecker JG, Podeszwa DA. Pediatric hip disorders. In: Cannada LK, ed. Orthopaedic Knowledge
Update 11. Rosemont, IL: American Academy of Orthopaedic Surgeons; 2014:859-873.

Guille JT, Pizzutillo PD, MacEwen GD. Development dysplasia of the hip from birth to six months. J Am
Acad Orthop Surg. 2000 Jul-Aug;8(4):232-42. Review. PubMed PMID: 10951112.

Murnaghan ML, Browne RH, Sucato DJ, Birch J. Femoral nerve palsy in Pavlik harness treatment
for developmental dysplasia of the hip. J Bone Joint Surg Am. 2011 Mar 2;93(5):493-9. doi: 10.2106/
JBJS.J.01210. PubMed PMID: 21368082.

© 2015 American Academy of Orthopaedic Surgeons 2015 Orthopaedic In-Training Examination


36 • American Academy of Orthopaedic Surgeons

Figure 104

Question 104
Figure 104 is the lateral ankle radiograph of a 10-year-old boy who has heel pain and a limp. There is no
history of trauma or recent illness. He is afebrile and has tenderness over the calcaneus. Laboratory results
should demonstrate

1. elevated Lyme titers.


2. elevated rheumatoid factor.
3. an increased C-reactive protein level (CRP).
4. an increased white blood cell count (WBC).
5. normal CRP and WBC values.

PREFERRED RESPONSE: 5

RECOMMENDED READINGS
Takada J, Hoshi M, Oebisu N, Ieguchi M, Kakehashi A, Wanibuchi H, Nakamura H. A comparative
study of clinicopathological features between simple bone cysts of the calcaneus and the long bone. Foot
Ankle Int. 2014 Apr;35(4):374-82. doi: 10.1177/1071100713519600. Epub 2014 Jan 9. PubMed PMID:
24406278.

Ishikawa SN. Conditions of the calcaneus in skeletally immature patients. Foot Ankle Clin. 2005
Sep;10(3):503-13, vi. Review. PubMed PMID: 16081017.

© 2015 American Academy of Orthopaedic Surgeons 2015 Orthopaedic In-Training Examination


SECTION 1: Pediatrics • 37

Figure 113

Question 113
Figure 113 is the postreduction CT scan of a 14-month-old girl with a left dislocated hip who underwent
closed reduction after adductor tenotomy. What is the best next step?

1. Acetabuloplasty
2. Maintainence of the left hip in a hip spica cast
3. Open reduction of the left hip
4. Closed reduction of the left hip
5. Varus femoral derotational osteotomy

PREFERRED RESPONSE: 2

RECOMMENDED READINGS
Cooper A, Evans O, Ali F, Flowers M. A novel method for assessing postoperative femoral head reduction
in developmental dysplasia of the hip. J Child Orthop. 2014 Aug;8(4):319-24. doi: 10.1007/s11832-014-
0600-5. Epub 2014 Jul 4. PubMed PMID: 24993902; PubMed Central PMCID: PMC4128942.

Madhu TS, Akula M, Scott BW, Templeton PA. Treatment of developmental dislocation of hip: does
changing the hip abduction angle in the hip spica affect the rate of avascular necrosis of the femoral
head? J Pediatr Orthop B. 2013 May;22(3):184-8. doi: 10.1097/BPB.0b013e32835ec690. PubMed PMID:
23407430.

© 2015 American Academy of Orthopaedic Surgeons 2015 Orthopaedic In-Training Examination


38 • American Academy of Orthopaedic Surgeons

Figure 122

Question 122
Figure 122 is the bilateral standing alignment radiograph of a 2-year-old boy who has bowed legs. His
mother states that he was born with bowed legs, and the deformity seems to have worsened since he
started walking at 11 months of age. The metaphyseal-diaphyseal angles are 18 degrees bilaterally. What
is the best treatment option?

1. Observation
2. Valgus-inducing knee-ankle-foot orthotic bracing
3. Bilateral proximal tibia and fibula epiphysiodesis
4. Immediate bilateral tibia and fibular valgus osteotomies with gradual correction with
external fixators
5. Immediate bilateral tibia and fibular valgus osteotomies with acute correction and
internal fixation

PREFERRED RESPONSE: 2

RECOMMENDED READINGS
Levine AM, Drennan JC. Physiological bowing and tibia vara. The metaphyseal-diaphyseal angle in the
measurement of bowleg deformities. J Bone Joint Surg Am. 1982 Oct;64(8):1158-63. PubMed PMID:
7130229.

Birch JG. Blount disease. J Am Acad Orthop Surg. 2013 Jul;21(7):408-18. doi: 10.5435/
JAAOS-21-07-408. Review. PubMed PMID: 23818028.

© 2015 American Academy of Orthopaedic Surgeons 2015 Orthopaedic In-Training Examination


SECTION 1: Pediatrics • 39

Figure 129

Question 129
Figure 129 is the pelvic radiograph of a 17-year-old boy who has cerebral palsy and Level V functioning
as measured by the Gross Motor Function Classification System. He has pain and decreased range of
motion of the right hip. Treatment should consist of

1. total hip arthroplasty.


2. arthroscopic repair of the labrum.
3. open reduction of the hip with osteotomies.
4. closed reduction of the hip with osteotomies.
5. femoral head resection and proximal femoral osteotomy.

PREFERRED RESPONSE: 5

RECOMMENDED READINGS
K Graham H, Narayanan UG. Salvage hip surgery in severe cerebral palsy: some answers, more
questions? Bone Joint J. 2014 May;96-B(5):567-8. doi: 10.1302/0301-620X.96B5.34119. PubMed PMID:
24788487.

Larnert P, Risto O, Hägglund G, Wagner P. Hip displacement in relation to age and gross motor function
in children with cerebral palsy. J Child Orthop. 2014 Mar;8(2):129-34. doi: 10.1007/s11832-014-0570-7.
Epub 2014 Mar 5. PubMed PMID: 24595560; PubMed Central PMCID: PMC3965763.

© 2015 American Academy of Orthopaedic Surgeons 2015 Orthopaedic In-Training Examination


40 • American Academy of Orthopaedic Surgeons

Question 138
Which percentage best describes the relative contribution of the proximal humeral growth plate to overall
humeral length?

1. 20%
2. 40%
3. 60%
4. 80%
5. 100%

PREFERRED RESPONSE: 4

RECOMMENDED READINGS
Wattenbarger JM, Frick SL. Shoulder, upper arm, and elbow trauma: pediatrics. In: Flynn JM, ed.
Orthopaedic Knowledge Update 10. Rosemont, IL: American Academy of Orthopaedic Surgeons;
2011:675-686.

Hensinger RL. Standards in Pediatric Orthopaedics: Tables, Charts, and Graphs Illustrating Growth.
New York, NY: Raven Press; 1986:142.

© 2015 American Academy of Orthopaedic Surgeons 2015 Orthopaedic In-Training Examination


SECTION 1: Pediatrics • 41

Figure 143

Question 143
Figure 143 is the pelvic radiograph of a 2-year-old girl who has a limp on her left side. Her left leg is
shorter than her right leg, and her left hip has less range of motion than her right hip. Treatment should
consist of

1. observation.
2. abduction bracing.
3. closed reduction of the hip.
4. closed reduction of the hip with adductor tenotomy.
5. open reduction of the hip with osteotomies.

PREFERRED RESPONSE: 5

RECOMMENDED READINGS
Wenger DR. Surgical treatment of developmental dysplasia of the hip. Instr Course Lect. 2014;63:313-23.
PubMed PMID: 24720317.

Gholve PA, Flynn JM, Garner MR, Millis MB, Kim YJ. Predictors for secondary procedures in walking
DDH. J Pediatr Orthop. 2012 Apr-May;32(3):282-9. doi: 10.1097/BPO.0b013e31824b21a6. PubMed
PMID: 22411335.

© 2015 American Academy of Orthopaedic Surgeons 2015 Orthopaedic In-Training Examination


42 • American Academy of Orthopaedic Surgeons

Figure 152a Figure 152b

Question 152
Figures 152a and 152b are the spine radiographs of a 17-year-old boy who has scoliosis that was noticed
during his well-child pediatric evaluation 2 months ago. He has no back pain or other symptoms and does
not notice any impairment related to his spinal deformity. The Cobb angle of the left upper thoracic curve
is 28 degrees, and the Cobb angle of the right main thoracic curve is 29 degrees. What is the best treatment
option?

1. Observation
2. Boston brace treatment
3. Anterior spinal fusion of both curves
4. Posterior spinal fusion of both curves
5. Selective posterior spinal fusion of the main thoracic curve only

PREFERRED RESPONSE: 1

RECOMMENDED READINGS
Dolan LA, Wright JG, Weinstein SL. Effects of bracing in adolescents with idiopathic scoliosis. N Engl J
Med. 2014 Feb 13;370(7):681. doi: 10.1056/NEJMc1314229. PubMed PMID: 24521128.

Schlenzka D, Yrjönen T. Bracing in adolescent idiopathic scoliosis. J Child Orthop. 2013 Feb;7(1):51-
5. doi: 10.1007/s11832-012-0464-5. Epub 2012 Nov 30. Review. PubMed PMID: 24432059; PubMed
Central PMCID: PMC3566257.

© 2015 American Academy of Orthopaedic Surgeons 2015 Orthopaedic In-Training Examination


SECTION 1: Pediatrics • 43

Figure 162

Question 162
Figure 162 is the right shoulder radiograph of a 5-year-old boy who fell and now has right shoulder pain.
Prior to the fall he had no history of symptoms. What is the best next step?

1. Bone scan
2. Chest radiograph
3. No further testing
4. Assessment of calcium and vitamin D levels
5. Skin biopsy to check for collagen abnormalities

PREFERRED RESPONSE: 3

RECOMMENDED READINGS
Pretell-Mazzini J, Murphy RF, Kushare I, Dormans JP. Unicameral bone cysts: general characteristics
and management controversies. J Am Acad Orthop Surg. 2014 May;22(5):295-303. doi: 10.5435/
JAAOS-22-05-295. Review. PubMed PMID: 24788445.

Donaldson S, Wright JG. Simple bone cysts: better with age? J Pediatr Orthop. 2015 Jan;35(1):108-14.
doi: 10.1097/BPO.0000000000000336. PubMed PMID: 25436480.

© 2015 American Academy of Orthopaedic Surgeons 2015 Orthopaedic In-Training Examination


44 • American Academy of Orthopaedic Surgeons

Figure 169

Question 169
Figure 169 is the lateral radiograph of a 6-year-old boy who has a 2-week history of left foot pain without
trauma. Examination reveals an antalgic gait with diffuse foot swelling. Treatment should consist of

1. foot immobilization.
2. midfoot biopsy.
3. midfoot irrigation and debridement.
4. open reduction and internal fixation.
5. closed reduction and internal fixation.

PREFERRED RESPONSE: 1

RECOMMENDED READINGS
DiGiovanni CW, Patel A, Calfee R, Nickisch F. Osteonecrosis in the foot. J Am Acad Orthop Surg. 2007
Apr;15(4):208-17. Review. PubMed PMID: 17426292.

Aiyer A, Hennrikus W. Foot pain in the child and adolescent. Pediatr Clin North Am 2014 Dec; 61(6):
1185-1205. PMID: 25439019.

© 2015 American Academy of Orthopaedic Surgeons 2015 Orthopaedic In-Training Examination


SECTION 1: Pediatrics • 45

Figure 174a Figure 174b

Question 174
Figures 174a and 174b are the anteroposterior and lateral tibia/fibula radiographs of a 6-year-old girl who
has a leg-length discrepancy with a short right leg. She has 4 toes on her right side. Her knee and ankle
examination is stable and she walks on her toes on her right leg. What is the most likely diagnosis?

1. Tibial deficiency
2. Fibular deficiency
3. Perthes disease
4. Gorham disease
5. Osteosarcoma of the fibula

PREFERRED RESPONSE: 2

RECOMMENDED READINGS
Oberc A, Sułko J. Fibular hemimelia - diagnostic management, principles, and results of treatment.
J Pediatr Orthop B. 2013 Sep;22(5):450-6. doi: 10.1097/BPB.0b013e32836330dd. PubMed PMID:
23807497.

Birch JG, Lincoln TL, Mack PW, Birch CM. Congenital fibular deficiency: a review of thirty years’
experience at one institution and a proposed classification system based on clinical deformity. J Bone Joint
Surg Am. 2011 Jun 15;93(12):1144-51. doi: 10.2106/JBJS.J.00683. PubMed PMID: 21776551.

© 2015 American Academy of Orthopaedic Surgeons 2015 Orthopaedic In-Training Examination


46 • American Academy of Orthopaedic Surgeons

Figure 183

Question 183
Figure 183 is the radiograph of a 6-year-old girl who has Morquio syndrome. She is scheduled for lower
extremity surgery to address the deformity. Presurgical testing should include

1. urinalysis.
2. cervical spine radiographs.
3. a chest radiograph.
4. an abdominal ultrasound.
5. electrolyte laboratory studies.

PREFERRED RESPONSE: 2

RECOMMENDED READINGS
White KK, Jester A, Bache CE, Harmatz PR, Shediac R, Thacker MM, Mackenzie WG.Orthopedic
management of the extremities in patients with Morquio A syndrome. J Child Orthop. 2014 Aug;8(4):295-
304. doi: 10.1007/s11832-014-0601-4. Epub 2014 Jul 8. PubMed PMID: 25001525; PubMed Central
PMCID: PMC4128951.

Baratela WA, Bober MB, Thacker MM, Belthur MV, Oto M, Rogers KJ, Mackenzie WG. Cervicothoracic
myelopathy in children with Morquio syndrome A: a report of 4 cases. J Pediatr Orthop. 2014
Mar;34(2):223-8. doi: 10.1097/BPO.0000000000000074.PubMed PMID: 24096444.

© 2015 American Academy of Orthopaedic Surgeons 2015 Orthopaedic In-Training Examination


SECTION 1: Pediatrics • 47

Question 189
A 4-year-old boy has been limping on his left leg for 4 days. His parents say he was fine until he began
limping. They report no fevers, but the boy had an upper respiratory infection about 2 weeks ago. His
white blood cell count is 8 109/L (reference range [rr], 4.5-11 109/L), his erythrocyte sedimentation rate is
30 mm/h (rr, 0-20 mm/h), and his C-reactive protein level is within defined limits. Pelvic radiographs are
unremarkable. What is the best next step?

1. Spica cast
2. MR image of the pelvis
3. Nonsteroidal anti-inflammatory drugs (NSAIDs) and observation
4. Irrigation and debridement of the left hip
5. Broad-spectrum antibiotics and observation

PREFERRED RESPONSE: 3

RECOMMENDED READINGS
Nouri A, Walmsley D, Pruszczynski B, Synder M. Transient synovitis of the hip: a comprehensive review.
J Pediatr Orthop B. 2014 Jan;23(1):32-6. doi: 10.1097/BPB.0b013e328363b5a3. Review. PubMed PMID:
23812087.

Singhal R, Perry DC, Khan FN, Cohen D, Stevenson HL, James LA, Sampath JS, Bruce CE. The use of
CRP within a clinical prediction algorithm for the differentiation of septic arthritis and transient synovitis
in children. J Bone Joint Surg Br. 2011 Nov;93(11):1556-61. doi: 10.1302/0301-620X.93B11.26857.
PubMed PMID: 22058311.

© 2015 American Academy of Orthopaedic Surgeons 2015 Orthopaedic In-Training Examination


48 • American Academy of Orthopaedic Surgeons

Figure 195a Figure 195b

Question 195
Figures 195a and 195b are the radiographs of a 14-year-old postmenarchal girl who has a prominence
over her right hip. She has pain with walking and running. The prominence is mildly tender without any
overlying skin changes. There is full hip range of motion. What is the best next step?

1. Excision
2. Radiotherapy
3. Chemotherapy
4. Hemipelvectomy alone
5. Hemipelvectomy with adjuvant chemotherapy

PREFERRED RESPONSE: 1

RECOMMENDED READINGS
Wodajo FM. Top five lesions that do not need referral to orthopedic oncology. Orthop Clin North Am.
2015 Apr;46(2):303-14. doi: 10.1016/j.ocl.2014.11.012. Review. PubMed PMID: 25771324.

Nystrom LM, DeYoung BR, Morcuende JA. Secondary chondrosarcoma of the pelvis arising from
a solitary exostosis in an 11-year-old patient: a case report with 5-year follow-up. Iowa Orthop J.
2013;33:213-6. PubMed PMID: 24027486; PubMed Central PMCID: PMC3748883.

© 2015 American Academy of Orthopaedic Surgeons 2015 Orthopaedic In-Training Examination


SECTION 1: Pediatrics • 49

Figure 210

Question 210
Figure 210 is the standing alignment image of a 3½-year-old girl. She has a painless limp on her left side
and a small lateral thrust on the left side during the stance phase. What is the best treatment option?

1. Observation
2. Tibia and fibular valgus-producing osteotomy
3. Varus thrust knee-ankle-foot orthosis at night
4. Varus thrust knee-ankle-foot orthosis at all times
5. Varus thrust knee-ankle-foot orthosis during the day only

PREFERRED RESPONSE: 2

RECOMMENDED READINGS
Birch JG. Blount disease. J Am Acad Orthop Surg. 2013 Jul;21(7):408-18. doi: 10.5435/
JAAOS-21-07-408. Review. PubMed PMID: 23818028.

Shinohara Y, Kamegaya M, Kuniyoshi K, Moriya H. Natural history of infantile tibia vara. J Bone Joint
Surg Br. 2002 Mar;84(2):263-8. PubMed PMID: 11922370.

© 2015 American Academy of Orthopaedic Surgeons 2015 Orthopaedic In-Training Examination


50 • American Academy of Orthopaedic Surgeons

Figure 223a Figure 223b

Question 223
These coronal ultrasound images show the right (Figure 223a) and left (Figure 223b) hips of a 7-month-
old girl who is seen for hip dysplasia. What is the best next step?

1. Bilateral Salter innominate osteotomies


2. Bilateral closed reduction and spica casting
3. Observation for 6 weeks with repeat ultrasound
4. Immediate Pavlik harness application
5. Immediate open reduction of both hips through medial approaches

PREFERRED RESPONSE: 2

RECOMMENDED READINGS
Vitale MG, Skaggs DL. Developmental dysplasia of the hip from six months to four years of age. J Am
Acad Orthop Surg. 2001 Nov-Dec;9(6):401-11. Review. PubMed PMID: 11730331.

Murray T, Cooperman DR, Thompson GH, Ballock T. Closed reduction for treatment of development
dysplasia of the hip in children. Am J Orthop (Belle Mead NJ).2007 Feb;36(2):82-4. PubMed PMID:
17676175.

© 2015 American Academy of Orthopaedic Surgeons 2015 Orthopaedic In-Training Examination


SECTION 1: Pediatrics • 51

Figure 229

Question 229
The deformity shown in Figure 229 is classified as

1. central.
2. terminal.
3. preaxial.
4. postaxial.
5. intercalary.

PREFERRED RESPONSE: 3

RECOMMENDED READINGS
Guo B, Lee SK, Paksima N. Polydactyly: a review. Bull Hosp Jt Dis (2013).2013;71(1):17-23. Review.
PubMed PMID: 24032579.

Al-Qattan MM. Preaxial polydactyly of the upper limb viewed as a spectrum of severity of embryonic
events. Ann Plast Surg. 2013 Jul;71(1):118-24. doi: 10.1097/SAP.0b013e318248b67f. Review. PubMed
PMID: 23364674.

© 2015 American Academy of Orthopaedic Surgeons 2015 Orthopaedic In-Training Examination


52 • American Academy of Orthopaedic Surgeons

Question 239
What is the typical age range for a child to walk without assistance?

1. 3 to 6 months
2. 6 to 12 months
3. 12 to 18 months
4. 18 to 24 months
5. 24 to 30 months

PREFERRED RESPONSE: 3

RECOMMENDED READINGS
Burnett CN, Johnson EW. Development of gait in childhood. II. Dev Med Child Neurol. 1971
Apr;13(2):207-15. PubMed PMID: 5562863.

Sutherland DH, Olshen R, Cooper L, Woo SL. The development of mature gait. J Bone Joint Surg Am.
1980 Apr;62(3):336-53. PubMed PMID: 7364807.

Question 247
A 12-year-old girl has joint pain. The pain started 2 years ago as knee pain with intermittent swelling
but no antecedent symptoms. Six months later she had hand, wrist, and ankle joint pain and swelling.
Examination reveals that her eyes are inflamed and the affected joints are tender, swollen, and exhibit
painful range of motion. Laboratory studies reveal elevated erythrocyte sedimentation rate (ESR) findings
and an elevated antinuclear antibody count. What is the mostly likely diagnosis?

1. Lyme disease
2. Reiter syndrome
3. Juvenile idiopathic arthritis
4. Systemic lupus erythematosus
5. Gonococcal arthritis

PREFERRED RESPONSE: 3

RECOMMENDED READINGS
Klatt BA, Chen A, Tuan R. Arthritis and other cartilage disorders. In: Cannada LK, ed. Orthopaedic
Knowledge Update 11. Rosemont, IL: American Academy of Orthopaedic Surgeons; 2014:207-222.

Hofer M, Southwood TR. Classification of childhood arthritis. Best Pract Res Clin Rheumatol. 2002
Jul;16(3):379-96. Review. PubMed PMID: 12387806.

© 2015 American Academy of Orthopaedic Surgeons 2015 Orthopaedic In-Training Examination


SECTION 1: Pediatrics • 53

Question 265
After closed reduction for a displaced pediatric supracondylar humeral fracture, pin removal is typically
recommended during which postsurgical time period?

1. 1 week
2. 3 weeks
3. 6 weeks
4. 9 weeks
5. 12 weeks

PREFERRED RESPONSE: 2

RECOMMENDED READINGS
Wattenbarger JM, Frick SL. Shoulder, upper arm, & elbow trauma: pediatrics. In: Flynn J, ed. Orthopaedic
Knowledge Update 10. Rosemont, IL: American Academy of Orthopaedic Surgeons; 2011:675-86.

Green DW, Widmann RF, Frank JS, Gardner MJ. Low incidence of ulnar nerve injury with crossed pin
placement for pediatric supracondylar humerus fractures using a mini-open technique. J Orthop Trauma.
2005 Mar;19(3):158-63. Review. PubMed PMID: 15758668.

© 2015 American Academy of Orthopaedic Surgeons 2015 Orthopaedic In-Training Examination


54 • American Academy of Orthopaedic Surgeons

SECTION 2: Trauma

Question 2
A 5-year-old girl has a type III supracondylar humeral fracture after falling off of the monkey bars.
Examination reveals normal motor function and sensation in all nerve distributions and a pink perfused
hand but no palpable pulse. An urgent closed reduction of the fracture is performed with percutaneous
pinning. Immediately after pinning, the hand is noted to be white; there is no pulse and no signal by
Doppler. What is the most appropriate action?

1. Splint the arm in 95 degrees of flexion and start a heparin drip


2. Immediate angiogram of the arm
3. Immediate removal of the pins; unreduce the fracture and assess perfusion of the hand
4. Immediate antecubital fossa exploration and forearm fasciotomy
5. Leave the pin fixation, splint the arm in 45 degrees of flexion, and monitor for overnight return
of perfusion

PREFERRED RESPONSE: 3

RECOMMENDED READINGS
Kelly DM, Meier J. Shoulder, upper arm, and elbow trauma: Pediatrics. In: Cannada LK, ed. Orthopaedic
Knowledge Update 11. Rosemont, IL: American Academy of Orthopaedic Surgeons; 2014:785-795.

Shah AS, Waters PM, Bae DS. Treatment of the “pink pulseless hand” in pediatric supracondylar humerus
fractures. J Hand Surg Am. 2013 Jul;38(7):1399-403; quiz 1404. doi: 10.1016/j.jhsa.2013.03.047. Review.
PubMed PMID: 23790425.

Weller A, Garg S, Larson AN, Fletcher ND, Schiller JR, Kwon M, Copley LA, Browne R, Ho CA.
Management of the pediatric pulseless supracondylar humeral fracture: is vascular exploration necessary?
J Bone Joint Surg Am. 2013 Nov 6;95(21):1906-12. doi: 10.2106/JBJS.L.01580. PubMed PMID:
24196459.

© 2015 American Academy of Orthopaedic Surgeons 2015 Orthopaedic In-Training Examination


SECTION 2: Trauma • 55

Figure 6

Question 6
Figure 6 is the radiograph of a 65-year-old right-hand-dominant woman with right upper extremity pain
and deformity after falling down several steps. Her sensory functions are grossly intact, but motor strength
is 4/5 for anterior interosseous nerve, posterior interosseous nerve, and ulnar nerve distributions distally.
Her capillary refill is 2 seconds, and her skin is intact. Evaluation reveals no other major injuries. In
addition to careful evaluation of the joint proximally and distally, what is the best next step?

1. Traction radiographs to assess the fracture pattern


2. MR imaging to fully evaluate for ligamentous injuries
3. Electromyography (EMG)/nerve conduction studies to evaluate neurologic deficit
4. Emergent surgical management
5. Urgent surgical management sometime that evening

PREFERRED RESPONSE: 1

RECOMMENDED READINGS
Doornberg J, Lindenhovius A, Kloen P, van Dijk CN, Zurakowski D, Ring D. Two and three-dimensional
computed tomography for the classification and management of distal humeral fractures. Evaluation of
reliability and diagnostic accuracy. J Bone Joint Surg Am. 2006 Aug;88(8):1795-801. PubMed PMID:
16882904.

Galano GJ, Ahmad CS, Levine WN. Current treatment strategies for bicolumnar distal humerus fractures.
J Am Acad Orthop Surg. 2010 Jan;18(1):20-30. Review. PubMed PMID: 20044489.

© 2015 American Academy of Orthopaedic Surgeons 2015 Orthopaedic In-Training Examination


56 • American Academy of Orthopaedic Surgeons

Figure 10a Figure 10b

Question 10
Figures 10a and 10b are the radiographs of a 6-year-old girl who fell on her outstretched right hand. What
is the best next step?

1- Closed reduction and casting of the supracondylar humeral fracture


2- Closed reduction and pinning with 2 or 3 laterally introduced pins of the supracondylar
humeral fracture
3- Closed reduction and pinning with 2 laterally introduced pins and 1 medially introduced pin of
the supracondylar humeral fracture
4- Open reduction and pinning of the supracondylar humeral fracture with 2 or 3 laterally
introduced pins
5- Open reduction and pinning of the supracondylar humeral fracture with 2 laterally introduced
pins and 1 medially introduced pin

PREFERRED RESPONSE: 2

RECOMMENDED READINGS
American Academy of Orthopaedic Surgeons: The Treatment of Pediatric Supracondylar Humerus
Fractures. Rosemont, IL: American Academy of Orthopaedic Surgeons, September 2011. Available at
http://www.aaos.org/research/guidelines/guide.asp. Accessed September 8, 2015.

Mallo G, Stanat SJ, Gaffney J. Use of the Gartland classification system for treatment of pediatric
supracondylar humerus fractures. Orthopedics. 2010 Jan;33(1):19. doi: 10.3928/01477447-20091124-08.
PubMed PMID: 20055347. 

© 2015 American Academy of Orthopaedic Surgeons 2015 Orthopaedic In-Training Examination


SECTION 2: Trauma • 57

Figure 14a Figure 14b

Question 14
Figures 14a and 14b are the radiographs of a 6-year-old girl who fell on her outstretched right hand and is
seen in the emergency department. She has decreased pulses and an under perfused hand. Closed reduction
and pinning with 3 laterally introduced pins is performed. Pulses at the wrist are absent, and her hand is
still under perfused. What is the best next step?

1. Explore the antecubital fossa


2. Add a medial pin with an open approach
3. Immediately remove the pins and check the pulses
4. Immediately remove the pins and perform an open reduction and internal fixation of the
fracture under direct vision.
5. Remove the pins and repeat the closed reduction followed by repeat pinning

PREFERRED RESPONSE: 1

RECOMMENDED READINGS
American Academy of Orthopaedic Surgeons: The Treatment of Pediatric Supracondylar Humerus
Fractures. Rosemont, IL: American Academy of Orthopaedic Surgeons, September 2011. Available at
http://www.aaos.org/research/guidelines/guide.asp. Accessed September 8, 2015.

Garbuz DS, Leitch K, Wright JG. The treatment of supracondylar fractures in children with an absent
radial pulse. J Pediatr Orthop. 1996 Sep-Oct;16(5):594-6. PubMed PMID: 8865043.

© 2015 American Academy of Orthopaedic Surgeons 2015 Orthopaedic In-Training Examination


58 • American Academy of Orthopaedic Surgeons

Question 18
A 24-year-old man has a low-velocity gunshot wound to his right humerus. It is an isolated injury, and he
has complete radial nerve palsy. Optimal treatment should consist of antibiotics and

1. irrigation, debridement, and external fixation of the humerus, with exploration of the
radial nerve.
2. irrigation, debridement, and intramedullary nailing of the humerus, with exploration of the
radial nerve.
3. irrigation, debridement, and open reduction and internal fixation (ORIF) of the fracture using a
plate and screws with exploration of the radial nerve.
4. exploration of the radial nerve and immobilization in a coaptation splint.
5. immobilization in a coaptation splint and observation of the radial nerve to see if
function returns.

PREFERRED RESPONSE: 5

RECOMMENDED READINGS
Guo Y, Chiou-Tan FY. Radial nerve injuries from gunshot wounds and other trauma: comparison of
electrodiagnostic findings. Am J Phys Med Rehabil. 2002 Mar;81(3):207-11. PubMed PMID: 11989518.

Vaidya R, Sethi A, Oliphant BW, Gibson V, Sethi S, Meehan R. Civilian gunshot injuries of the humerus.
Orthopedics. 2014 Mar;37(3):e307-12. doi: 10.3928/01477447-20140225-66. PubMed PMID: 24762161. 

© 2015 American Academy of Orthopaedic Surgeons 2015 Orthopaedic In-Training Examination


SECTION 2: Trauma • 59

Figure 32a Figure 32b Figure 32c

Figure 32d Figure 32e

Question 32
Figures 32a through 32e are the radiographs, CT scans, and arthroscopic view of a 26-year-old man who
was hit by a car and sustained an injury to his right knee. Treatment of his injury should include

1. open reduction and internal fixation (ORIF) of the medial tibial plateau with a locked plate.
2. ORIF of the medial tibial plateau and repair or reconstruction of the lateral collateral ligament
(LCL) and posterolateral corner.
3. revascularization of the leg, ORIF of the medial tibial plateau, and repair or reconstruction of
the LCL and posterolateral corner.
4. revascularization of the leg and repair of the LCL and posterolateral corner.
5. revascularization of the leg and ORIF of the lateral and medial tibial plateaus.

PREFERRED RESPONSE: 3
© 2015 American Academy of Orthopaedic Surgeons 2015 Orthopaedic In-Training Examination
60 • American Academy of Orthopaedic Surgeons

RECOMMENDED READINGS
Cole P, Levy B, Schatzker J, Watson JT. Tibial plateau fractures. In: Browner B, Levine A, Jupiter J,
Trafton P, Krettek C, eds. Skeletal Trauma: Basic Science Management and Reconstruction. Philadelphia,
PA: Saunders Elsevier; 2009:2201-2287.

Chang SM, Zhang YQ, Yao MW, Du SC, Li Q, Guo Z. Schatzker type IV medial tibial plateau fractures:
a computed tomography-based morphological subclassification. Orthopedics. 2014 Aug;37(8):e699-706.
doi: 10.3928/01477447-20140728-55. PubMed PMID: 25102505.

© 2015 American Academy of Orthopaedic Surgeons 2015 Orthopaedic In-Training Examination


SECTION 2: Trauma • 61

Figure 40a Figure 40b Figure 40c

Figure 40d

Question 40
Figures 40a through 40d are the radiographs and CT scans of an 18-year-old woman who sustained a tibia/
fibula fracture. Prior to intramedullary nailing of the tibia, the physician should

1. plate the fibula.


2. place an external fixator.
3. perform a 4-compartment fasciotomy.
4. perform reduction and internal fixation of the intra-articular split.
5. perform a stress examination to see if there is syndesmotic disruption.

PREFERRED RESPONSE: 4

© 2015 American Academy of Orthopaedic Surgeons 2015 Orthopaedic In-Training Examination


62 • American Academy of Orthopaedic Surgeons

RECOMMENDED READINGS
Tejwani N, Polonet D, Wolinsky PR. Controversies in the intramedullary nailing of proximal and distal
tibia fractures. J Am Acad Orthop Surg. 2014 Oct;22(10):665-73. doi: 10.5435/JAAOS-22-10-665.
PubMed PMID: 25281261.

Casstevens C, Le T, Archdeacon MT, Wyrick JD. Management of extra-articular fractures of the distal
tibia: intramedullary nailing versus plate fixation. J Am Acad Orthop Surg. 2012 Nov;20(11):675-83. doi:
10.5435/JAAOS-20-11-675. PubMed PMID: 23118133.

© 2015 American Academy of Orthopaedic Surgeons 2015 Orthopaedic In-Training Examination


SECTION 2: Trauma • 63

Figure 57a Figure 57b

Question 57 (Item Deleted)


Figures 57a and 57b are the CT scanograms of a 24-year-old man who was shot in the left thigh. He
sustained an isolated comminuted femoral shaft fracture. After performing a locked intramedullary nail
procedure, the scanogram was taken to check rotational alignment. After reading the scanogram, what is
the best next step?

1. The shaft needs to be internally rotated 25.94 degrees to correct the deformity; the right
femoral anteversion is 43.33 degrees and the left is 17.39 degrees; the left leg is
overrotated externally.
2. The shaft needs to be externally rotated 25.94 degrees to correct the deformity; the right
femoral anteversion is 43.33 degrees and the left is 17.39 degrees; the left leg is
overrotated internally.
3. The right femoral anteversion is 6.39 degrees and the left femoral anteversion is 7.17 degrees,
so the result is good.
4. The right femoral anteversion is 6.39 degrees and the left is 17.39 degrees; the deformity
should be corrected by rotating the left leg externally 11 degrees.
5. The right femoral anteversion is 6.39 degrees and the left is 17.39 degrees; the deformity
should be corrected by rotating the left leg internally 11 degrees.

PREFERRED RESPONSE: 3

RECOMMENDED READINGS
Lindsey JD, Krieg JC. Femoral malrotation following intramedullary nail fixation. J Am Acad Orthop
Surg. 2011 Jan;19(1):17-26. PubMed PMID: 21205764.

Gugala Z, Qaisi YT, Hipp JA, Lindsey RW. Long-term functional implications of the iatrogenic rotational
malalignment of healed diaphyseal femur fractures following intramedullary nailing. Clin Biomech
(Bristol, Avon). 2011 Mar;26(3):274-7. doi: 10.1016/j.clinbiomech.2010.11.005. Epub 2010 Nov 30.
PubMed PMID: 21122956.

© 2015 American Academy of Orthopaedic Surgeons 2015 Orthopaedic In-Training Examination


64 • American Academy of Orthopaedic Surgeons

Figure 63

Question 63
Figure 63 is the CT scan of a 43-year-old woman who was involved in a motor vehicle collision and
sustained multiple injuries including a pelvic fracture. The injury shown in the CT scan is most consistent
with a (an)

1. combined-mechanism (CM) injury with a VS mechanism with a completely unstable right


hemipelvis and a LC injury resulting in a completely unstable left hemipelvis.
2. anteroposterior compression (APC) mechanism resulting in a completely unstable
right hemipelvis.
3. APC mechanism resulting in a partially unstable right and left hemipelvis.
4. vertical shear mechanism resulting in a partially unstable left hemipelvis internal rotation and
a completely unstable right hemipelvis external rotation (open book).
5. lateral compression mechanism type 3 resulting in a partially unstable left hemipelvis internal
rotation and a partially unstable right hemipelvis external rotation (open book).

PREFERRED RESPONSE: 5

RECOMMENDED READINGS
Pennal GF, Tile M, Waddell JP, Garside H. Pelvic disruption: assessment and classification. Clin Orthop
Relat Res. 1980 Sep;(151):12-21. PubMed PMID: 7418295.

Marsh JL, Slongo TF, Agel J, Broderick JS, Creevey W, DeCoster TA, Prokuski L, Sirkin MS, Ziran B,
Henley B, Audigé L. Fracture and dislocation classification compendium - 2007: Orthopaedic Trauma
Association classification, database and outcomes committee. J Orthop Trauma. 2007 Nov-Dec;21(10
Suppl):S1-133. PubMed PMID: 18277234.

Young JW, Burgess AR, Brumback RJ, Poka A. Pelvic fractures: value of plain radiography in early
assessment and management. Radiology. 1986 Aug;160(2):445-51. PubMed PMID: 3726125.

© 2015 American Academy of Orthopaedic Surgeons 2015 Orthopaedic In-Training Examination


SECTION 2: Trauma • 65

Figure 73a Figure 73b Figure 73c

Question 73
Figure 73a is the radiograph of a healthy 50-year-old farmer who was driving a pickup truck when he was
involved in a motor vehicle collision. He arrived at the hospital 3 hours after the injury with right hip pain.
You elect to reduce and fix his hip. After obtaining informed consent, he is placed under general anesthesia
and positioned on the fracture table. Figures 73b and 73c are the fluoroscopic figures after your best
attempt at reduction. What is the best next step?

1. Fix the hip in its current position.


2. Change plans and do a hemiarthroplasty.
3. Change plans and do a total hip arthroplasty.
4. Perform an open reduction of the femoral neck and then fix it.
5. Ask a partner to try to do a closed reduction and see if he or she can do a better job.

PREFERRED RESPONSE: 4

© 2015 American Academy of Orthopaedic Surgeons 2015 Orthopaedic In-Training Examination


66 • American Academy of Orthopaedic Surgeons

RECOMMENDED READINGS
Pauyo T, Drager J, Albers A, Harvey EJ. Management of femoral neck fractures in the young patient: A
critical analysis review. World J Orthop. 2014 Jul 18;5(3):204-17. doi: 10.5312/wjo.v5.i3.204. eCollection
2014 Jul 18. Review. PubMed PMID: 25035822.

Kregor PJ. The effect of femoral neck fractures on femoral head blood flow. Orthopedics. 1996
Dec;19(12):1031-6; quiz 1037-8. Review. PubMed PMID: 8972521.

Upadhyay A, Jain P, Mishra P, Maini L, Gautum VK, Dhaon BK. Delayed internal fixation of fractures
of the neck of the femur in young adults. A prospective, randomised study comparing closed and open
reduction. J Bone Joint Surg Br. 2004 Sep;86(7):1035-40. PubMed PMID: 15446534.3.

Gautam VK, Anand S, Dhaon BK. Management of displaced femoral neck fractures in young adults (a
group at risk). Injury. 1998 Apr;29(3):215-8. PubMed PMID: 9709424.

Parker MJ. The management of intracapsular fractures of the proximal femur. J Bone Joint Surg Br. 2000
Sep;82(7):937-41. Review. PubMed PMID: 11041577.

Weinrobe M, Stankewich CJ, Mueller B, Tencer AF. Predicting the mechanical outcome of femoral
neck fractures fixed with cancellous screws: an in vivo study. J Orthop Trauma. 1998 Jan;12(1):27-36;
discussion 36-7. PubMed PMID: 9447516.

Bosch U, Schreiber T, Krettek C. Reduction and fixation of displaced intracapsular fractures of the
proximal femur. Clin Orthop Relat Res. 2002 Jun;(399):59-71. Review. PubMed PMID: 12011695.

Garden RS. Malreduction and avascular necrosis in subcapital fractures of the femur. J Bone Joint Surg
Br. 1971 May;53(2):183-97. PubMed PMID: 5578215.

© 2015 American Academy of Orthopaedic Surgeons 2015 Orthopaedic In-Training Examination


SECTION 2: Trauma • 67

Figure 78a Figure 78b

Question 78
Figures 78a and 78b are the radiographs of a 62-year-old woman with long-standing type 1 diabetes
mellitus who fell and injured her right ankle. Her HbA1c level is 8%, or 64 mmol/mol. She has loss of
protective sensibility that is confirmed via testing with a 5.07 Semmes-Weinstein monofilament. What is
the best next step?

1. Nonsurgical treatment with a cast and prolonged nonweight-bearing activity


2. Surgical delay until her HgA1c level is normalized
3. Surgical treatment with minimal fixation and a prolonged period of postsurgical
nonweight-bearing activity
4. Surgical treatment including multiple syndesmotic screws and a prolonged period of
postsurgical nonweight-bearing activity
5. Surgical treatment without syndesmotic screws unless there is a syndesmotic injury and a
prolonged period of postsurgical nonweight-bearing activity

PREFERRED RESPONSE: 4

RECOMMENDED READINGS
Rosenbaum AJ, Dellenbaugh SG, Dipreta JA, Uhl RL. The management of ankle fractures in diabetics:
results of a survey of the American Orthopaedic Foot and Ankle Society membership. Foot Ankle Spec.
2013 Jun;6(3):201-5. doi: 10.1177/1938640013477132. Epub 2013 Feb 19. PubMed PMID: 23424187.

McCormack RG, Leith JM. Ankle fractures in diabetics. Complications of surgical management. J Bone
Joint Surg Br. 1998 Jul;80(4):689-92. PubMed PMID: 9699839.

Wukich DK, Kline AJ. The management of ankle fractures in patients with diabetes. J Bone Joint Surg
Am. 2008 Jul;90(7):1570-8. doi: 10.2106/JBJS.G.01673. Review. PubMed PMID: 18594108.

© 2015 American Academy of Orthopaedic Surgeons 2015 Orthopaedic In-Training Examination
68 • American Academy of Orthopaedic Surgeons

Question 82
A 23-year-old African American mother of 2 children from a low-income household is seen in the
emergency department with a subtrochanteric hip fracture. She has a history of frequent emergency room
visits for chronic low-back pain, abdominal pain, a sleep disorder, and severe headaches. She most likely

1. has lead poisoning.


2. has Crohn’s disease.
3. has osteogenesis imperfecta.
4. has secondary hyperparathyroidism.
5. is a victim of domestic violence.

PREFERRED RESPONSE: 5

RECOMMENDED READINGS
Zillmer DA. Domestic violence: the role of the orthopaedic surgeon in identification and treatment. J Am
Acad Orthop Surg. 2000 Mar-Apr;8(2):91-6. Review. PubMed PMID: 10799094.

AAOS Information Statement


Child Abuse or Maltreatment, Elder Maltreatment, and Intimate Partner Violence (IPV): The Orthopaedic
Surgeon’s Responsibilities in Domestic and Family Violence http://www.aaos.org/about/papers/
advistmt/1030.asp Accessed September 8, 2015.

© 2015 American Academy of Orthopaedic Surgeons 2015 Orthopaedic In-Training Examination


SECTION 2: Trauma • 69

Figure 91a Figure 91b

Question 91
Figures 91a and 91b are the current radiographs of a 60-year-old woman with type 1 diabetes mellitus who
sustained a bimalleolar ankle fracture treated with open reduction and internal fixation 6 weeks ago. What
is the best next step?

1. Allow the patient to start weight-bearing activity as tolerated


2. Remove the syndesmotic screws and start weight-bearing activity as tolerated
3. Keep the patient in a cast and start weight-bearing activity as tolerated
4. Keep the patient in a protective device and delay advancing weight-bearing status for an
additional month
5. Keep the patient in a protective device and advance weight-bearing status to weight-bearing
activity as tolerated

PREFERRED RESPONSE: 4

RECOMMENDED READINGS
Rosenbaum AJ, Dellenbaugh SG, Dipreta JA, Uhl RL. The management of ankle fractures in diabetics:
results of a survey of the American Orthopaedic Foot and Ankle Society membership. Foot Ankle Spec.
2013 Jun;6(3):201-5. doi: 10.1177/1938640013477132. Epub 2013 Feb 19. PubMed PMID: 23424187.

Wukich DK, Kline AJ. The management of ankle fractures in patients with diabetes. J Bone Joint Surg
Am. 2008 Jul;90(7):1570-8. doi: 10.2106/JBJS.G.01673. Review. PubMed PMID: 18594108.

© 2015 American Academy of Orthopaedic Surgeons 2015 Orthopaedic In-Training Examination


70 • American Academy of Orthopaedic Surgeons

Figure 97

Question 97
Figure 97 reveals a fracture sustained by a 60-year-old man 5 weeks after he underwent total hip
arthroplasty. What is the most appropriate way to treat this fracture?

1. Open reduction and internal fixation (ORIF) with locked-plate fixation


2. ORIF with cortical strut graft and cable fixation
3. Percutaneous submuscular locked-plate fixation
4. Femoral component revision with cerclage cable fixation
5. Revision to a proximal femoral replacement

PREFERRED RESPONSE: 4

RECOMMENDED READINGS
Masri BA, Meek RM, Duncan CP. Periprosthetic fractures evaluation and treatment. Clin Orthop Relat
Res. 2004 Mar;(420):80-95. Review. PubMed PMID: 15057082.

Ko PS, Lam JJ, Tio MK, Lee OB, Ip FK. Distal fixation with Wagner revision stem in treating Vancouver
type B2 periprosthetic femur fractures in geriatric patients. J Arthroplasty. 2003 Jun;18(4):446-52.
PubMed PMID: 12820087. 

© 2015 American Academy of Orthopaedic Surgeons 2015 Orthopaedic In-Training Examination


SECTION 2: Trauma • 71

Figure 102a Figure 102b

Question 102
Figures 102a and 102b are the radiographs of a 60-year-old woman who slipped and fell and is unable to
bear weight. In the emergency department, she was found to be neurologically intact and had good pulses.
What is the most likely diagnosis?

1. Bicondylar injury
2. Fracture of the posterior and medial tibial plateau
3. Split depression fracture of the anterolateral tibial plateau
4. Split depression fracture of the posterolateral tibial plateau
5. Split depression fracture of the posteromedial tibial plateau

PREFERRED RESPONSE: 4

RECOMMENDED READINGS
Sohn HS, Yoon YC, Cho JW, Cho WT, Oh CW, Oh JK. Incidence and fracture morphology of
posterolateral fragments in lateral and bicondylar tibial plateau fractures. J Orthop Trauma. 2015
Feb;29(2):91-7. doi: 10.1097/BOT.0000000000000170. PubMed PMID: 24978940.

Marsh JL. Tibial plateau fractures. In: Bucholz RW, Heckman JD, Court-Brown CM, Tornetta P, eds.
Rockwood and Green’s Fractures in Adults. 7th ed. Philadelphia, PA: Lippincott Williams & Wilkins;
2012:1787-1789.

© 2015 American Academy of Orthopaedic Surgeons 2015 Orthopaedic In-Training Examination


72 • American Academy of Orthopaedic Surgeons

Figure 107a Figure 107b Figure 107c

Question 107
Figures 107a through 107c are the radiographs of a 9-year-old girl who is seen in the emergency
department 1 hour after a fall from monkey bars. A gross deformity is noted at the left elbow, but there are
no lacerations or open areas in the skin. She is further assessed and found to have delayed capillary refill,
diminished radial artery pulsation in the ipsilateral hand, and decreased ability to flex her fingers and wrist
(when compared to the contralateral side). What is the best next step?

1. Perform an immediate closed reduction and percutaneous pinning


2. Perform closed reduction and hyperflexion splinting
3. Take the patient emergently to the operating room for open reduction and
nerve/artery exploration
4. Obtain an emergent CT angiogram of the limb to assess location of arterial flow interruption
5. Immediately elevate the extremity and splint at 40 degrees relative extension to decrease
swelling and improve blood flow

PREFERRED RESPONSE: 1

© 2015 American Academy of Orthopaedic Surgeons 2015 Orthopaedic In-Training Examination


SECTION 2: Trauma • 73

RECOMMENDED READINGS
Kelly DM, Meier J. Shoulder, upper arm, and elbow trauma: Pediatrics. In: Cannada LK, ed. Orthopaedic
Knowledge Update 11. Rosemont, IL: American Academy of Orthopaedic Surgeons; 2014:785-795.

Carter CT, Bertrand SL, Cearley DM. Management of pediatric type III supracondylar humerus fractures
in the United States: results of a national survey of pediatric orthopaedic surgeons. J Pediatr Orthop. 2013
Oct-Nov;33(7):750-4. doi: 10.1097/BPO.0b013e31829f92f3. PubMed PMID: 24025582.

Abzug JM, Herman MJ. Management of supracondylar humerus fractures in children: current concepts. J
Am Acad Orthop Surg. 2012 Feb;20(2):69-77. doi: 10.5435/JAAOS-20-02-069. Review. PubMed PMID:
22302444.

Babal JC, Mehlman CT, Klein G. Nerve injuries associated with pediatric supracondylar
humeral fractures: a meta-analysis. J Pediatr Orthop. 2010 Apr-May;30(3):253-63. doi: 10.1097/
BPO.0b013e3181d213a6. PubMed PMID: 20357592.

Question 111 (Item Deleted)


A woman with multiple injuries is unconscious. Her injuries include a closed comminuted tibial shaft
fracture for which there is concern for the development of compartment syndrome. As intracompartmental
pressure (ICP), systolic blood pressure (sBP), and diastolic blood pressure (dBP) are monitored, at which
point is fasciotomy indicated?

1. When the ICP is elevated to 20 mm Hg


2. When the ICP is elevated to 30 mm Hg
3. When the ICP is elevated to 20 mm Hg below dBP
4. When the ICP is elevated to 30 mm Hg below dBP
5. Only when the ICP is equal to or greater than the dBP

PREFERRED RESPONSE: 3

RECOMMENDED READINGS
Whitesides TE, Heckman MM. Acute Compartment Syndrome: Update on Diagnosis and Treatment. J Am
Acad Orthop Surg. 1996 Jul;4(4):209-218. PubMed PMID: 10795056.

Olson SA, Glasgow RR. Acute compartment syndrome in lower extremity musculoskeletal trauma. J Am
Acad Orthop Surg. 2005 Nov;13(7):436-44. Review.PubMed PMID: 16272268.

© 2015 American Academy of Orthopaedic Surgeons 2015 Orthopaedic In-Training Examination


74 • American Academy of Orthopaedic Surgeons

Question 114
Scapular fractures are noteworthy for their associated injuries. Which imaging modality would identify the
injury most commonly associated with scapular fractures?

1. CT of the head
2. CT of the chest
3. Focused assessment with sonography for trauma (FAST)
4. Angiogram
5. Anteroposterior pelvis radiograph

PREFERRED RESPONSE: 2

RECOMMENDED READINGS
Lunsjo K, Tadros, Czechowski Jk, Abu-Zidan. Scapular fractures and associated injuries in blunt trauma: a
prospective study. J Bone Joint Surg Br. 2006;88:Supp 1:141.

Baldwin KD, Ohman-Strickland P, Mehta S, Hume E. Scapula fractures: a marker for concomitant injury?
A retrospective review of data in the National Trauma Database. J Trauma. 2008 Aug;65(2):430-5. doi:
10.1097/TA.0b013e31817fd928. PubMed PMID: 18695481.

© 2015 American Academy of Orthopaedic Surgeons 2015 Orthopaedic In-Training Examination


SECTION 2: Trauma • 75

Figure 120a Figure 120b

Question 120
Figures 120a and 120b are the sagittal MR images of a man who injured his knee after he slipped and fell
on ice 2 days ago. He has severe knee pain and instability. Examination is difficult because of swelling,
guarding, and apprehension. What is the indicated treatment?

1. Hinged knee brace


2. Anterior cruciate ligament (ACL) reconstruction
3. Open posterior cruciate ligament (PCL) reconstruction
4. Patellar tendon repair
5. Arthroscopic medial meniscectomy

PREFERRED RESPONSE: 4

RECOMMENDED READINGS
Matava MJ. Patellar Tendon Ruptures. J Am Acad Orthop Surg. 1996 Nov;4(6):287-296. PubMed PMID:
10797196.

Volk WR, Yagnik GP, Uribe JW. Complications in brief: Quadriceps and patellar tendon tears. Clin Orthop
Relat Res. 2014 Mar;472(3):1050-7. doi: 10.1007/s11999-013-3396-6. Epub 2013 Dec 12. Review.
PubMed PMID: 24338040; PubMed Central PMCID: PMC3916631.

© 2015 American Academy of Orthopaedic Surgeons 2015 Orthopaedic In-Training Examination


76 • American Academy of Orthopaedic Surgeons

Figure 123

Question 123
Figure 123 is the anteroposterior radiograph of a 69-year-old active woman who fell off of a ladder and is
unable to ambulate. Which treatment offers the best long-term solution?

1. Hemiarthroplasty
2. Total hip arthroplasty (THA)
3. 3 cannulated screws
4. A cephalomedullary device
5. A dynamic hip screw with side plate

PREFERRED RESPONSE: 2

RECOMMENDED READINGS
Healy WL, Iorio R. Total hip arthroplasty: optimal treatment for displaced femoral neck fractures in
elderly patients. Clin Orthop Relat Res. 2004 Dec;(429):43-8. PubMed PMID: 15577464.

Yu L, Wang Y, Chen J. Total hip arthroplasty versus hemiarthroplasty for displaced femoral neck
fractures: meta-analysis of randomized trials. Clin Orthop Relat Res. 2012 Aug;470(8):2235-43. doi:
10.1007/s11999-012-2293-8. Epub 2012 Mar 1. PubMed PMID: 22395872; PubMed Central PMCID:
PMC3392403.

© 2015 American Academy of Orthopaedic Surgeons 2015 Orthopaedic In-Training Examination


SECTION 2: Trauma • 77

Question 128
A 54-year-old man sustained a closed tibial shaft fracture that was treated with open reduction and internal
fixation using an intramedullary nail. On his follow-up visit, he noted that his foot was rotated differently
than the contralateral foot. Which imaging modality can be used to best evaluate the deformity?

1. Ultrasound
2. MR imaging
3. Stress radiograph
4. Standing radiograph
5. Computed tomography

PREFERRED RESPONSE: 5

RECOMMENDED READINGS
Puloski S, Romano C, Buckley R, Powell J. Rotational malalignment of the tibia following reamed
intramedullary nail fixation. J Orthop Trauma. 2004 Aug;18(7):397-402. PubMed PMID: 15289683.

Theriault B, Turgeon AF, Pelet S. Functional impact of tibial malrotation following intramedullary nailing
of tibial shaft fractures. J Bone Joint Surg Am. 2012 Nov 21;94(22):2033-9. doi: 10.2106/JBJS.K.00859.
PubMed PMID: 23172320.

Question 132
Which malunion is most commonly associated with intramedullary nailing (IMN) fixation of proximal
tibial shaft fractures?

1. Procurvatum, varus
2. Procurvatum, valgus
3. Recurvatum, varus
4. Recurvatum, valgus
5. Recurvatum, internal rotation

PREFERRED RESPONSE: 2

RECOMMENDED READINGS
Ricci WM, O’Boyle M, Borrelli J, Bellabarba C, Sanders R. Fractures of the proximal third of the tibial
shaft treated with intramedullary nails and blocking screws. J Orthop Trauma. 2001 May;15(4):264-70.
PubMed PMID: 11371791.

Nork SE, Barei DP, Schildhauer TA, Agel J, Holt SK, Schrick JL, Sangeorzan BJ. Intramedullary nailing
of proximal quarter tibial fractures. J Orthop Trauma. 2006 Sep;20(8):523-8. PubMed PMID: 16990722.

Hiesterman TG, Shafiq BX, Cole PA. Intramedullary nailing of extra-articular proximal tibia fractures. J
Am Acad Orthop Surg. 2011 Nov;19(11):690-700. Review. PubMed PMID: 22052645. 

© 2015 American Academy of Orthopaedic Surgeons 2015 Orthopaedic In-Training Examination


78 • American Academy of Orthopaedic Surgeons

Figure 135a Figure 135b

Figure 135c

Question 135
Figures 135a through 135c are the radiographs of a 78-year-old woman who sustained an elbow injury to
her nondominant arm after a fall from a standing height. If considering total elbow arthroplasty vs internal
fixation, the preferred surgical approach is

1. medial through a flexor-pronator split.


2. straight posterior through an olecranon osteotomy.
3. anterolateral between the brachialis and biceps.
4. lateral column, elevating off of common extensors and the capsule.
5. posterior triceps sparing.

PREFERRED RESPONSE: 5

© 2015 American Academy of Orthopaedic Surgeons 2015 Orthopaedic In-Training Examination


SECTION 2: Trauma • 79

RECOMMENDED READINGS
Sørensen BW, Brorson S, Olsen BS. Primary total elbow arthroplasty in complex fractures of the distal
humerus. World J Orthop. 2014 Jul 18;5(3):368-72. doi: 10.5312/wjo.v5.i3.368. eCollection 2014 Jul 18.
PubMed PMID: 25035841; PubMed Central PMCID: PMC4095031.

McKee MD, Veillette CJ, Hall JA, Schemitsch EH, Wild LM, McCormack R, Perey B, Goetz T, Zomar
M, Moon K, Mandel S, Petit S, Guy P, Leung I. A multicenter, prospective, randomized, controlled trial
of open reduction--internal fixation versus total elbow arthroplasty for displaced intra-articular distal
humeral fractures in elderly patients. J Shoulder Elbow Surg. 2009 Jan-Feb;18(1):3-12. doi: 10.1016/j.
jse.2008.06.005. Epub 2008 Sep 26. PubMed PMID: 18823799.

© 2015 American Academy of Orthopaedic Surgeons 2015 Orthopaedic In-Training Examination


80 • American Academy of Orthopaedic Surgeons

Figure 137a Figure 137b


From Journal of the American Academy of Orthopaedic Surgeons 16(12), pages 716-728.

Question 137
Figures 137a and 137b are the radiographs of a 35-year-old man who has an elbow injury. What is the best
surgical approach for this fracture?

1. Medial through the flexor-pronator split


2. Straight posterior, through an olecranon osteotomy
3. Anterolateral between the brachialis and biceps
4. Lateral column, elevating off of common extensors and the capsule
5. Limited Kocher approach between the anconeus and extensor carpi ulnaris

PREFERRED RESPONSE: 4

RECOMMENDED READINGS
McKee MD, Jupiter JB, Bamberger HB. Coronal shear fractures of the distal end of the humerus. J Bone
Joint Surg Am. 1996 Jan;78(1):49-54. PubMed PMID: 8550679.

Ruchelsman DE, Tejwani NC, Kwon YW, Egol KA. Coronal plane partial articular fractures of the distal
humerus: current concepts in management. J Am Acad Orthop Surg. 2008 Dec;16(12):716-28. Review.
PubMed PMID: 19056920.

© 2015 American Academy of Orthopaedic Surgeons 2015 Orthopaedic In-Training Examination


SECTION 2: Trauma • 81

Question 139
A 19-year-old man broke his radius and ulna in the mid forearm. Following closed reduction and splinting,
his ulna is translated 20% with less than 5 degrees of angulation, and the radius is well aligned with only 5
degrees of apex-volar angulation. The physician should recommend

1. an above-elbow splint for 3 weeks followed by a forearm fracture brace.


2. an above-elbow cast for 6 weeks.
3. open reduction and internal fixation (ORIF) of the ulna.
4. ORIF of the radius.
5. ORIF of the radius and ulna.

PREFERRED RESPONSE: 5

RECOMMENDED READINGS
Schulte LM, Meals CG, Neviaser RJ. Management of adult diaphyseal both-bone forearm fractures. J Am
Acad Orthop Surg. 2014 Jul;22(7):437-46. doi: 10.5435/JAAOS-22-07-437. Review. PubMed PMID:
24966250.

Anderson LD, Sisk D, Tooms RE, Park WI 3rd. Compression-plate fixation in acute diaphyseal fractures
of the radius and ulna. J Bone Joint Surg Am. 1975 Apr;57(3):287-97. PubMed PMID: 1091653.

© 2015 American Academy of Orthopaedic Surgeons 2015 Orthopaedic In-Training Examination


82 • American Academy of Orthopaedic Surgeons

Figure 145
From Journal of the American Academy of Orthopaedic Surgeons 20(8), pages 498-505.

Question 145
Figure 145 is the radiograph of a 34-year-old man who sustained a clavicle fracture after a dirt bike
accident. When considering benefits, risks, and costs, the most cost-effective treatment method that
minimizes morbidity is

1. figure-of-8 bracing.
2. open reduction plate fixation.
3. locked intramedullary fixation.
4. sling management with delayed surgery (if required).
5. multiple percutaneous thread Kirschner wire fixation.

PREFERRED RESPONSE: 4

RECOMMENDED READINGS
Rehn CH, Kirkegaard M, Viberg B, Larsen MS. Operative versus nonoperative treatment of displaced
midshaft clavicle fractures in adults: a systematic review. Eur J Orthop Surg Traumatol. 2014
Oct;24(7):1047-53. doi: 10.1007/s00590-013-1370-3. Epub 2013 Dec 10. Review. PubMed PMID:
24322539.

Robinson CM, Goudie EB, Murray IR, Jenkins PJ, Ahktar MA, Read EO, Foster CJ, lark K, Brooksbank
AJ, Arthur A, Crowther MA, Packham I, Chesser TJ. Open reduction and plate fixation versus
nonoperative treatment for displaced midshaft clavicular fractures: a multicenter, randomized, controlled
trial. J Bone Joint Surg Am. 2013 Sep 4;95(17):1576-84. doi: 10.2106/JBJS.L.00307. PubMed PMID:
24005198.

Walton B, Meijer K, Melancon K, Hartman M. A cost analysis of internal fixation versus nonoperative
treatment in adult midshaft clavicle fractures using multiple randomized controlled trials. J Orthop
Trauma. 2015 Apr;29(4):173-80. doi: 10.1097/BOT.0000000000000225. PubMed PMID: 25233160.

© 2015 American Academy of Orthopaedic Surgeons 2015 Orthopaedic In-Training Examination


SECTION 2: Trauma • 83

Question 150
When compared to compression plating, antegrade intramedullary nailing (IMN) of humeral shaft
fractures in adults is associated with increased risk for

1. implant failure.
2. fracture nonunion.
3. radial nerve palsy.
4. periprosthetic fracture.
5. rotational shoulder stiffness.

PREFERRED RESPONSE: 5

RECOMMENDED READINGS
Kurup H, Hossain M, Andrew JG. Dynamic compression plating versus locked intramedullary nailing
for humeral shaft fractures in adults. Cochrane Database Syst Rev. 2011 Jun 15;(6):CD005959. doi:
10.1002/14651858.CD005959.pub2. Review. PubMed PMID: 21678350.

Li Y, Wang C, Wang M, Huang L, Huang Q. Postoperative malrotation of humeral shaft fracture after
plating compared with intramedullary nailing. J Shoulder Elbow Surg. 2011 Sep;20(6):947-54. doi:
10.1016/j.jse.2010.12.016. Epub 2011 Mar 26. PubMed PMID: 21440461.

© 2015 American Academy of Orthopaedic Surgeons 2015 Orthopaedic In-Training Examination


84 • American Academy of Orthopaedic Surgeons

Figure 157

Question 157
A 38-year-old patient sustained the isolated left foot injury seen in Figure 157 after falling from a ladder.
Which treatment will most likely produce the best clinical outcome and lowest risk for revision surgery?

1. Arthrodesis
2. Arthroplasty
3. Closed reduction and casting
4. Closed reduction and percutaneous pinning
5. Open reduction and internal fixation (ORIF)

PREFERRED RESPONSE: 1

RECOMMENDED READINGS
Ly TV, Coetzee JC. Treatment of primarily ligamentous Lisfranc joint injuries: primary arthrodesis
compared with open reduction and internal fixation. A prospective, randomized study. J Bone Joint Surg
Am. 2006 Mar;88(3):514-20. PubMed PMID: 16510816.

Henning JA, Jones CB, Sietsema DL, Bohay DR, Anderson JG. Open reduction internal fixation
versus primary arthrodesis for lisfranc injuries: a prospective randomized study. Foot Ankle Int. 2009
Oct;30(10):913-22. doi: 10.3113/FAI.2009.0913. PubMed PMID: 19796583.

© 2015 American Academy of Orthopaedic Surgeons 2015 Orthopaedic In-Training Examination


SECTION 2: Trauma • 85

Question 160
A 40-year-old man sustains a low-velocity gunshot wound to the pelvis. The entry wound is on his right
flank, resulting in a fracture of the right ilium and left acetabulum. The bullet is lodged in his left hip. He
is hemodynamically stable and has good peripheral pulses, decreased motor and sensory function of the
right sciatic nerve, and rebound tenderness of the abdomen. The next step is to

1. perform a sigmoidoscopy.
2. order an electromyography/nerve conduction studies (EMG/NCS) to assess if the sciatic nerve
has a neurapraxia.
3. order an angiography to assess for an internal iliac injury.
4. take him for a laparotomy.
5. remove the bullet from his left hip.

PREFERRED RESPONSE: 4

RECOMMENDED READINGS
Bartkiw MJ, Sethi A, Coniglione F, Holland D, Hoard D, Colen R, Tyburski JG, Vaidya R. Civilian
gunshot wounds of the hip and pelvis. J Orthop Trauma. 2010 Oct;24(10):645-52. doi: 10.1097/
BOT.0b013e3181cf03ea. PubMed PMID: 20871253.

Najibi S, Matta JM, Dougherty PJ, Tannast M. Gunshot wounds to the acetabulum. J Orthop Trauma. 2012
Aug;26(8):451-9. doi: 10.1097/BOT.0b013e31822c085d. PubMed PMID: 22357085.

© 2015 American Academy of Orthopaedic Surgeons 2015 Orthopaedic In-Training Examination


86 • American Academy of Orthopaedic Surgeons

Figure 161a Figure 161b

Question 161 (Item Deleted)


Figures 161a and 161b are the axial-cut CT scans of a patient with an acetabular fracture. The arrow is
pointing to which structure?

1. Femoral vein
2. Femoral artery
3. Femoral nerve
4. Iliopsoas tendon
5. Iliopectineal fascia

PREFERRED RESPONSE: 3

RECOMMENDED READINGS
Standring S (ed): Gray’s Anatomy: The Anatomical Basis of Clinical Practice. JAMA 2009;301(17):1825-
1831.

Hoppenfeld S, deBoer P (eds): Surgical Exposures in Orthopaedics: The Anatomic Approach. 2nd ed.
Philadelphia, PA: JB Lippincott;1994:368-375.

© 2015 American Academy of Orthopaedic Surgeons 2015 Orthopaedic In-Training Examination


SECTION 2: Trauma • 87

Question 164
To be most effective, poller screws should be placed at which location when treating a proximal third tibial
shaft fracture that tends to adopt a valgus position?

1. Medial to the nail in the metaphyseal segment


2. Medial side of the nail in the diaphyseal segment
3. Medial to lateral in the metaphyseal segment posterior to the nail
4. Lateral to the nail in the metaphyseal segment
5. Lateral side of the nail in the diaphyseal segment

PREFERRED RESPONSE: 4

RECOMMENDED READINGS
Stedtfeld HW, Mittlmeier T, Landgraf P, Ewert A. The logic and clinical applications of blocking screws. J
Bone Joint Surg Am. 2004;86-A Suppl 2:17-25. PubMed PMID: 15691104.

Stinner DJ, Mir H. Techniques for intramedullary nailing of proximal tibia fractures. Orthop Clin North
Am. 2014 Jan;45(1):33-45. doi: 10.1016/j.ocl.2013.09.001. Epub 2013 Oct 5. Review. PubMed PMID:
24267205.

Hiesterman TG, Shafiq BX, Cole PA. Intramedullary nailing of extra-articular proximal tibia fractures. J
Am Acad Orthop Surg. 2011 Nov;19(11):690-700. Review. PubMed PMID: 22052645. 

Question 168
A 25-year-old Hispanic man with a gunshot wound to his right humerus and femur is seen in the trauma
bay. He is neurovascularly intact throughout and has had both fractures immobilized with splints in
preparation for surgery. The nurse says that he has received pain medication but is requesting more. In
addition to assessing for compartment syndrome, the physician should

1. counsel the patient that he will feel better after surgery.


2. order a toxicology screen to assess for recent narcotic use.
3. be aware that pain medication dosing may be influenced by implicit bias.
4. assume the nurse may have a racial bias and order more pain medication for the patient.
5. ignore the request for increased pain medicine.

PREFERRED RESPONSE: 3

© 2015 American Academy of Orthopaedic Surgeons 2015 Orthopaedic In-Training Examination


88 • American Academy of Orthopaedic Surgeons

RECOMMENDED READINGS
Mir HR, Rao RD. Patient-centered care: communication skills and cultural competence. In: Cannada LK,
ed. Orthopaedic Knowledge Update 11. Rosemont, IL: American Academy of Orthopaedic Surgeons;
2014:91-101.

Stone J, Moskowitz GB. Non-conscious bias in medical decision making: what can be done to reduce it?
Med Educ. 2011 Aug;45(8):768-76. doi: 10.1111/j.1365-2923.2011.04026.x. PubMed PMID: 21752073.

Sabin J, Nosek BA, Greenwald A, Rivara FP. Physicians’ implicit and explicit attitudes about race by
MD race, ethnicity, and gender. J Health Care Poor Underserved. 2009 Aug;20(3):896-913. doi: 10.1353/
hpu.0.0185. PubMed PMID: 19648715; PubMed Central PMCID: PMC3320738. 

Figure 173c

Figure 173a Figure 173b Figure 173d

Figure 173e Figure 173f Figure 173g Figure 173h

© 2015 American Academy of Orthopaedic Surgeons 2015 Orthopaedic In-Training Examination


SECTION 2: Trauma • 89

Question 173
A 90-year-old woman sustained a spontaneous left femur fracture at the tip of a cemented revision total
hip arthroplasty femoral stem, as seen in Figures 173a through 173d. Prior to her fracture, she reported
no hip problems but had intermittent midthigh pain for 5 years and had seen a hip surgeon 3 years ago to
discuss this pain (Figures 173e through 173h). The revision arthroplasty was performed 18 years ago with
a long trochanteric osteotomy. Her comorbidities include mild chronic obstructive pulmonary disease,
coronary artery disease, S/P 3-vessel coronary artery bypass grafting, hypertension, and type II diabetes
mellitus. What is the best next step in treatment?

1. Open reduction and internal fixation with a long anatomically contoured locking femoral plate,
use of monocortical locking screws and cables adjacent to the femoral stem, and hip
arthroplasty implant retention
2. Open reduction and internal fixation with a long proximal femoral locking plate bypassing the
fracture by 4 holes, use of monocortical locking screws adjacent to the femoral stem, and hip
arthroplasty implant retention
3. Open reduction and internal fixation with an 8-hole compression plate, use of cables adjacent
to the femoral stem and screws below the stem, and hip arthroplasty implant retention
4. Open reduction and internal fixation with a cortical strut allograft secured with cables and hip
arthroplasty implant retention
5. Removal of all implants and hip arthroplasty revision using a long modular femoral stem that
passes the fracture site

PREFERRED RESPONSE: 1

RECOMMENDED READINGS
Wood GC, Naudie DR, McAuley J, McCalden RW. Locking compression plates for the treatment of
periprosthetic femoral fractures around well-fixed total hip and knee implants. J Arthroplasty. 2011
Sep;26(6):886-92. doi: 10.1016/j.arth.2010.07.002. Epub 2010 Sep 3. PubMed PMID: 20817391.

Bryant GK, Morshed S, Agel J, Henley MB, Barei DP, Taitsman LA, Nork SE.Isolated locked
compression plating for Vancouver Type B1 periprosthetic femoral fractures. Injury. 2009
Nov;40(11):1180-6. doi: 10.1016/j.injury.2009.02.017.Epub 2009 Jun 18. PubMed PMID: 19539924

Fulkerson E, Koval K, Preston CF, Iesaka K, Kummer FJ, Egol KA. Fixation of periprosthetic femoral
shaft fractures associated with cemented femoral stems: a biomechanical comparison of locked plating and
conventional cable plates. J Orthop Trauma. 2006 Feb;20(2):89-93. PubMed PMID: 16462560.

Ricci WM, Bolhofner BR, Loftus T, Cox C, Mitchell S, Borrelli J Jr. Indirect reduction and plate fixation,
without grafting, for periprosthetic femoral shaft fractures about a stable intramedullary implant. J Bone
Joint Surg Am. 2005 Oct;87(10):2240-5. PubMed PMID: 16203889.

© 2015 American Academy of Orthopaedic Surgeons 2015 Orthopaedic In-Training Examination


90 • American Academy of Orthopaedic Surgeons

Question 182
After hip hemiarthroplasty to address a fracture, use of which test can help surgeons best predict the need
for a walking aid 2 years after surgery?

1. Shuttle run
2. Illinois agility
3. 1-leg stance
4. Timed up and go (TUG)
5. Timed stair ascent

PREFERRED RESPONSE: 3

RECOMMENDED READINGS
Springer BA, Marin R, Cyhan T, Roberts H, Gill NW. Normative values for the unipedal stance test with
eyes open and closed. J Geriatr Phys Ther. 2007;30(1):8-15. PubMed PMID: 19839175.

Laflamme GY, Rouleau DM, Leduc S, Roy L, Beaumont E. The Timed Up and Go test is an early
predictor of functional outcome after hemiarthroplasty for femoral neck fracture. J Bone Joint Surg Am.
2012 Jul 3;94(13):1175-9. doi: 10.2106/JBJS.J.01952. PubMed PMID: 22760384.

Kristensen MT, Henriksen S, Stie SB, Bandholm T. Relative and absolute intertester reliability of the
timed up and go test to quantify functional mobility in patients with hip fracture. J Am Geriatr Soc. 2011
Mar;59(3):565-7.doi: 10.1111/j.1532-5415.2010.03293.x. PubMed PMID: 21391955. 

Question 190
A 40-year-old man sustained a low-velocity gunshot wound to his right ilium and acetabulum. What is the
most common associated injury?

1. Bladder injury
2. Perforated viscus
3. Major venous injury
4. Major arterial injury
5. Major peripheral nerve injury

PREFERRED RESPONSE: 2

RECOMMENDED READINGS
Bartkiw MJ, Sethi A, Coniglione F, Holland D, Hoard D, Colen R, Tyburski JG, Vaidya R. Civilian
gunshot wounds of the hip and pelvis. J Orthop Trauma. 2010 Oct;24(10):645-52. doi: 10.1097/
BOT.0b013e3181cf03ea. PubMed PMID: 20871253.

Najibi S, Matta JM, Dougherty PJ, Tannast M. Gunshot wounds to the acetabulum. J Orthop Trauma. 2012
Aug;26(8):451-9. doi: 10.1097/BOT.0b013e31822c085d. PubMed PMID: 22357085. 

© 2015 American Academy of Orthopaedic Surgeons 2015 Orthopaedic In-Training Examination


SECTION 2: Trauma • 91

Figure 196a Figure 196b

Question 196
Figures 196a and 196b are the radiographs of a 3-year-old child with an elbow injury. Pulses are not
palpable at the wrist, but the child’s fingers are pink with sluggish capillary refill. Treatment should consist
of

1. splinting and observation for return of pulses.


2. reduction and fixation of the fracture with reassessment of perfusion.
3. exploration of the brachial artery after reduction and fixation.
4. exploration of the brachial artery before reduction and fixation.
5. angiogram to assess arterial flow before reduction and fixation.

PREFERRED RESPONSE: 2

RECOMMENDED READINGS
Weller A, Garg S, Larson AN, Fletcher ND, Schiller JR, Kwon M, Copley LA, Browne R, Ho CA.
Management of the pediatric pulseless supracondylar humeral fracture: is vascular exploration necessary?
J Bone Joint Surg Am. 2013 Nov 6;95(21):1906-12. doi: 10.2106/JBJS.L.01580. PubMed PMID:
24196459.

Scannell BP, Jackson JB 3rd, Bray C, Roush TS, Brighton BK, Frick SL. The perfused, pulseless
supracondylar humeral fracture: intermediate-term follow-up of vascular status and function. J Bone Joint
Surg Am. 2013 Nov 6;95(21):1913-9. doi: 10.2106/JBJS.L.01584. PubMed PMID: 24196460. 

© 2015 American Academy of Orthopaedic Surgeons 2015 Orthopaedic In-Training Examination


92 • American Academy of Orthopaedic Surgeons

Figure 204a Figure 204b

Question 204
Figures 204a and 204b are the 3-dimensional CT scan and CT scan of a 23-year-old basketball player who
fell on his left elbow 7 days ago. Current motion is 80 to 110 degrees in flexion-extension, 5 degrees of
supination, and 20 degrees of pronation limited by pain in the area of the mobile wad. Aspiration of 7 ml
of blood from the joint and a lidocaine injection do not improve his motion. What is the best next step?

1. Radial head replacement


2. Open reduction and internal fixation of the radial head
3. Splint for 7 to 10 days and re-examine
4. Sling and gradual increase of activities to tolerance
5. A 2-week trial of physical therapy to restore motion

PREFERRED RESPONSE: 2

RECOMMENDED READINGS
Ikeda M, Sugiyama K, Kang C, Takagaki T, Oka Y. Comminuted fractures of the radial head. Comparison
of resection and internal fixation. J Bone Joint Surg Am. 2005 Jan;87(1):76-84. PubMed PMID:
15634816.

Businger A, Ruedi TP, Sommer C. On-table reconstruction of comminuted fractures of the radial head.
Injury. 2010 Jun;41(6):583-8. doi: 10.1016/j.injury.2009.10.026. Epub 2009 Nov 22. PubMed PMID:
19932475.

© 2015 American Academy of Orthopaedic Surgeons 2015 Orthopaedic In-Training Examination


SECTION 2: Trauma • 93

Question 208
When performing the initial evaluation of an injured patient in the emergency department, adherence to
the Advanced Trauma Life Support (ATLS) protocol has been shown to

1. decrease the amount of time to initial patient imaging.


2. decrease the cost associated with providing Level I trauma care.
3. increase the number of patients receiving head CT scans.
4. improve patient satisfaction with emergency department care.
5. improve patient outcomes and decrease initial management errors.

PREFERRED RESPONSE: 5

RECOMMENDED READINGS
van Olden GD, Meeuwis JD, Bolhuis HW, Boxma H, Goris RJ. Clinical impact of advanced trauma life
support. Am J Emerg Med. 2004 Nov;22(7):522-5. PubMed PMID: 15666253.

Ali J, Adam R, Butler AK, Chang H, Howard M, Gonsalves D, Pitt-Miller P, Stedman M, Winn J,
Williams JI. Trauma outcome improves following the advanced trauma life support program in a
developing country. J Trauma. 1993 Jun;34(6):890-8; discussion 898-9. PubMed PMID: 8315686.

Question 212
In a patient with a high-energy comminuted femoral shaft fracture, which ipsilateral fracture is present as
often as 10% of the time and initially missed up to 50% of the time?

1. Talus
2. Calcaneus
3. Tibial spine
4. Tibial plafond
5. Femoral neck

PREFERRED RESPONSE: 5

RECOMMENDED READINGS
Tornetta P 3rd, Kain MS, Creevy WR. Diagnosis of femoral neck fractures in patients with a femoral shaft
fracture. Improvement with a standard protocol. J Bone Joint Surg Am. 2007 Jan;89(1):39-43. PubMed
PMID: 17200308.

Riemer BL, Butterfield SL, Ray RL, Daffner RH. Clandestine femoral neck fractures with ipsilateral
diaphyseal fractures. J Orthop Trauma. 1993;7(5):443-9. PubMed PMID: 8229381.

Swiontkowski MF, Hansen ST Jr, Kellam J. Ipsilateral fractures of the femoral neck and shaft. A treatment
protocol. J Bone Joint Surg Am. 1984 Feb;66(2):260-8.
PubMed PMID: 6693453. 

© 2015 American Academy of Orthopaedic Surgeons 2015 Orthopaedic In-Training Examination


94 • American Academy of Orthopaedic Surgeons

Question 218
Which factor is associated with increased failure rates for the treatment of stable intertrochanteric femur
fractures with a dynamic hip screw (DHS)?

1. Long barrel
2. Early weight bearing
3. Younger patient age
4. 2-hole plate instead of a 4-hole plate
5. Tip-apex distance exceeding 25 mm

PREFERRED RESPONSE: 5

RECOMMENDED READINGS
Baumgaertner MR, Curtin SL, Lindskog DM, Keggi JM. The value of the tip-apex distance in predicting
failure of fixation of peritrochanteric fractures of the hip. J Bone Joint Surg Am. 1995 Jul;77(7):1058-64.
PubMed PMID: 7608228.

Bolhofner BR, Russo PR, Carmen B. Results of intertrochanteric femur fractures treated with a
135-degree sliding screw with a two-hole side plate. J Orthop Trauma. 1999 Jan;13(1):5-8. PubMed
PMID: 9892117. 

© 2015 American Academy of Orthopaedic Surgeons 2015 Orthopaedic In-Training Examination


SECTION 2: Trauma • 95

Figure 219a Figure 219b Figure 219c

Question 219
Figures 219a through 219c are the postreduction radiographs of a 28-year-old man who had left ankle
pain after falling down a flight of stairs. He underwent closed reduction in the emergency department. To
maximize rotational stability of the distal fibula, fixation should include

1. intramedullary fixation of the fibula.


2. screw-and-suture button fixation of the syndesmosis.
3. quadricortical screw fixation of the syndesmosis over a 4-hole plate.
4. locking plate fixation of the fibula and deltoid ligament repair.
5. plate fixation of the fibula and fixation of the posterior malleolus.

PREFERRED RESPONSE: 5

RECOMMENDED READINGS
Irwin TA, Lien J, Kadakia AR. Posterior malleolus fracture. J Am Acad Orthop Surg. 2013 Jan;21(1):32-
40. doi: 10.5435/JAAOS-21-01-32. Review. PubMed PMID: 23281469.

Gardner MJ, Brodsky A, Briggs SM, Nielson JH, Lorich DG. Fixation of posterior malleolar fractures
provides greater syndesmotic stability. Clin Orthop Relat Res. 2006 Jun;447:165-71. PubMed PMID:
16467626.

Hartford JM, Gorczyca JT, McNamara JL, Mayor MB. Tibiotalar contact area. Contribution of posterior
malleolus and deltoid ligament. Clin Orthop Relat Res. 1995 Nov;(320):182-7. PubMed PMID: 7586825. 

© 2015 American Academy of Orthopaedic Surgeons 2015 Orthopaedic In-Training Examination


96 • American Academy of Orthopaedic Surgeons

Question 224
Which imaging method is optimal when attempting to determine displacement of a medial humeral
epicondyle fracture?

1. Distal humeral axial view


2. External oblique radiographs
3. Internal oblique radiographs
4. Standard anteroposterior and lateral elbow radiographs
5. Fluoroscopic examination of the elbow

PREFERRED RESPONSE: 1

RECOMMENDED READINGS
Edmonds EW. How displaced are “nondisplaced” fractures of the medial humeral epicondyle in children?
Results of a three-dimensional computed tomography analysis. J Bone Joint Surg Am. 2010 Dec
1;92(17):2785-91. doi: 10.2106/JBJS.I.01637. PubMed PMID: 21123608.

Souder CD, Farnsworth CL, McNeil NP, Bomar JD, Edmonds EW. The Distal Humerus Axial
View: Assessment of Displacement in Medial Epicondyle Fractures. J Pediatr Orthop. 2015 Jul-
Aug;35(5):449-54. doi: 10.1097/BPO.0000000000000306. PubMed PMID: 25171678.

Question 226
Titanium is often described as the best implant for fracture internal fixation because its elastic modulus
is closest to that of bone. Cortical bone has an approximate modulus of elasticity of 15 (in megapascals
[MPa], range 7 to 30). What is the approximate modulus of elasticity of titanium?

1. 20 MPa
2. 40 MPa
3. 60 MPa
4. 80 MPa
5. 100 MPa

PREFERRED RESPONSE: 5

RECOMMENDED READINGS
Rho JY, Ashman RB, Turner CH. Young's modulus of trabecular and cortical bone material: ultrasonic and
microtensile measurements. J Biomech. 1993 Feb;26(2):111-9. PubMed PMID: 8429054.

Niinomi M. Mechanical properties of biomedical titanium alloys. Materials Science and Engineering: A.
1998;243(1-2):231-6.

© 2015 American Academy of Orthopaedic Surgeons 2015 Orthopaedic In-Training Examination


SECTION 2: Trauma • 97

Question 230
What is the in-hospital mortality and 1-year mortality after hip fracture among elderly people in the United
States?

1. 6%; 15%
2. 6%; 30%
3. 15%; 30%
4. 15%; 40%
5. 6%; 40%

PREFERRED RESPONSE: 2

RECOMMENDED READINGS
Brauer CA, Coca-Perraillon M, Cutler DM, Rosen AB. Incidence and mortality of hip fractures in the
United States. JAMA. 2009 Oct 14;302(14):1573-9. doi:10.1001/jama.2009.1462. PubMed PMID:
19826027; PubMed Central PMCID: PMC4410861.

Jiang HX, Majumdar SR, Dick DA, Moreau M, Raso J, Otto DD, Johnston DW.Development and initial
validation of a risk score for predicting in-hospital and 1-year mortality in patients with hip fractures. J
Bone Miner Res. 2005 Mar;20(3):494-500. Epub 2004 Nov 29. PubMed PMID: 15746995.

Question 235
A healthy 70-year-old man has an isolated high-energy open bimalleolar ankle fracture after a fall from a
bicycle. Compared to a 30-year-old man with the same injury, his postsurgical course will be associated
with a (an)

1. higher infection rate.


2. shorter time to union.
3. equivalent complication rate.
4. increased incidence of nerve injury.
5. increased rate of nonunion.

PREFERRED RESPONSE: 3

RECOMMENDED READINGS
Herscovici D Jr, Scaduto JM. Management of high-energy foot and ankle injuries in the geriatric
population. Geriatr Orthop Surg Rehabil. 2012 Mar;3(1):33-44. doi: 10.1177/2151458511436112. PubMed
PMID: 23569695; PubMed Central PMCID: PMC3617904.

Pagliaro AJ, Michelson JD, Mizel MS. Results of operative fixation of unstable ankle fractures in geriatric
patients. Foot Ankle Int. 2001 May;22(5):399-402. PubMed PMID: 11428758.

© 2015 American Academy of Orthopaedic Surgeons 2015 Orthopaedic In-Training Examination


98 • American Academy of Orthopaedic Surgeons

Question 240
What is the current U.S. Food and Drug Administration (FDA) indication for recombinant human bone
morphogenetic protein-2 (rhBMP-2) implantation after a fracture occurs?

1. Aseptic tibial nonunion repair


2. Acute open tibial shaft fracture
3. Acute open femoral fracture after intramedullary nail (IMN) fixation
4. Acute open femoral fracture with segmental bone loss after IMN
5. There is currently no FDA indication for use of rhBMP-2 after fracture

PREFERRED RESPONSE: 2

RECOMMENDED READINGS
Govender S, Csimma C, Genant HK, Valentin-Opran A, Amit Y, Arbel R, Aro H, Atar D, Bishay M,
Börner MG, Chiron P, Choong P, Cinats J, Courtenay B, Feibel R, Geulette B, Gravel C, Haas N,
Raschke M, Hammacher E, van der Velde D, Hardy P, Holt M, Josten C, Ketterl RL, Lindeque B, Lob
G, Mathevon H, McCoy G, Marsh D, Miller R, Munting E, Oevre S, Nordsletten L, Patel A, Pohl A,
Rennie W, Reynders P, Rommens PM, Rondia J, Rossouw WC, Daneel PJ, Ruff S, Rüter A, Santavirta S,
Schildhauer TA, Gekle C, Schnettler R, Segal D, Seiler H, Snowdowne RB, Stapert J, Taglang G, Verdonk
R, Vogels L, Weckbach A, Wentzensen A, Wisniewski T; BMP-2 Evaluation in Surgery for Tibial Trauma
(BESTT) Study Group. Recombinant human bone morphogenetic protein-2 for treatment of open tibial
fractures: a prospective, controlled, randomized study of four hundred and fifty patients. J Bone Joint Surg
Am. 2002 Dec;84-A(12):2123-34. PubMed PMID: 12473698.

Alt V, Borgman B, Eicher A, Heiss C, Kanakaris NK, Giannoudis PV, Song F. Effects of recombinant
human Bone Morphogenetic Protein-2 (rhBMP-2) in grade III open tibia fractures treated with unreamed
nails-A clinical and health-economic analysis. Injury. 2015 Nov;46(11):2267-72. doi: 10.1016/j.
injury.2015.07.013. Epub 2015 Aug 13. PubMed PMID: 26374949.

Wei S, Cai X, Huang J, Xu F, Liu X, Wang Q. Recombinant human BMP-2 for the treatment of open tibial
fractures. Orthopedics. 2012 Jun;35(6):e847-54. doi: 10.3928/01477447-20120525-23. Review. PubMed
PMID: 22691656.

© 2015 American Academy of Orthopaedic Surgeons 2015 Orthopaedic In-Training Examination


SECTION 2: Trauma • 99

Question 250
Electrical stimulation of bone is thought to promote bone healing through

1. downregulation of osteoclasts.
2. micromechanical stimulation of osteoblasts.
3. upregulation of growth factors transforming growth factor-beta and other bone
morphogenetic proteins.
4. induced mechanical microtrauma at boundary tissues.
5. enhancement of intracellular calcium and signal transduction.

PREFERRED RESPONSE: 3

RECOMMENDED READINGS
Nauth A, Kuzyk PR, Einhorn TA, Schemitsch EH. Advances in the enhancement of bone healing and bone
graft substitutes. In: Schmidt AH, Teague DC, eds. Orthopaedic Knowledge Update: Trauma 4. Rosemont,
IL: American Academy of Orthopaedic Surgeons; 2010:133-143.

Aaron RK, Boyan BD, Ciombor DM, Schwartz Z, Simon BJ. Stimulation of growth factor synthesis by
electric and electromagnetic fields. Clin Orthop Relat Res. 2004 Feb;(419):30-7. Review. PubMed PMID:
15021128.

© 2015 American Academy of Orthopaedic Surgeons 2015 Orthopaedic In-Training Examination


100 • American Academy of Orthopaedic Surgeons

Figure 259a Figure 259b

Question 259
Figures 259a and 259b are the radiograph and clinical photograph of a 50-year-old woman who fell and
sustained a distal humeral fracture. You opt to perform an osteotomy to expose the injury. What is the best
option?

1. A transverse osteotomy 1 cm distal to the triceps insertion in the bare area of the ulna
2. A transverse osteotomy 2 cm distal to the triceps insertion in the bare area of the ulna
3. A Chevron osteotomy 2 cm distal to the triceps insertion in the bare area of the ulna
4. A Chevron osteotomy 3 cm distal to the triceps insertion in the bare area of the ulna
5. An oblique osteotomy that completely avoids the articular surface of the ulnohumeral joint

PREFERRED RESPONSE: 3

RECOMMENDED READINGS
Wang AA, Mara M, Hutchinson DT. The proximal ulna: An anatomic study with relevance to olecranon
osteotomy and fracture fixation. J Shoulder Elbow Surg. 2003 May-Jun;12(3):293-6. PubMed PMID:
12851585.

Elmadag M, Erdil M, Bilsel K, Acar MA, Tuncer N, Tuncay I. The olecranon osteotomy provides better
outcome than the triceps-lifting approach for the treatment of distal humerus fractures. Eur J Orthop Surg
Traumatol. 2014 Jan;24(1):43-50. doi: 10.1007/s00590-012-1149-y. Epub 2012 Dec 21. PubMed PMID:
23412273.

© 2015 American Academy of Orthopaedic Surgeons 2015 Orthopaedic In-Training Examination


SECTION 2: Trauma • 101

Question 261
A 19-year-old woman was a pedestrian when she was hit by a car. She arrives in the emergency
department confused and anxious. Her heart rate (HR) is 125 beats per minute (BPM) and her blood
pressure (BP) is 90/50 mm Hg. Two large-bore intravenous lines are started, and 2 liters of normal saline
are given. She can move all 4 extremities on command and has good pulses in all 4 limbs. Radiographs
reveal pelvic and bilateral tibia fractures. According to the Advanced Trauma Life Support for Doctors
(ATLS) handbook, her hemorrhage meets criteria for which class?

1. I
2. II
3. III
4. IV
5. V

PREFERRED RESPONSE: 3

RECOMMENDED READINGS
ATLS: Advanced Trauma Life Support for Doctors (Student Course Manual). Chicago9, IL: American
College of Surgeons.

Guly HR, Bouamra O, Spiers M, Dark P, Coats T, Lecky FE; Trauma Audit and Research Network. Vital
signs and estimated blood loss in patients with major trauma: testing the validity of the ATLS classification
of hypovolaemic shock. Resuscitation. 2011 May;82(5):556-9. doi: 10.1016/j.resuscitation.2011.01.013.
Epub 2011 Feb 23. PubMed PMID: 21349628.

Question 266
An 85-year-old woman with mild dementia fell and is taken to a hospital. She is unable to bear weight on
her right leg, and diagnostics reveal a displaced intertrochanteric right femur fracture. To improve short-
and intermediate-term outcomes for this patient,

1. her pain should be controlled primarily with meperidine instead of morphine.


2. she should be transported to the operating room emergently without any medical evaluation
within 12 hours for fracture fixation.
3. she should be admitted to the geriatric service or a hospitalist (if available) for medical
optimization in preparation for surgical fracture management.
4. she should undergo a neurology service consultation for evaluation of dementia in preparation
for surgical fracture management.
5. she should undergo a routine echocardiogram with a cardiology consultation to address risk
stratification in advance of surgical fracture management.

PREFERRED RESPONSE: 3

© 2015 American Academy of Orthopaedic Surgeons 2015 Orthopaedic In-Training Examination


102 • American Academy of Orthopaedic Surgeons

RECOMMENDED READINGS
Friedman SM, Mendelson DA, Kates SL, McCann RM. Geriatric co-management of proximal femur
fractures: total quality management and protocol-driven care result in better outcomes for a frail patient
population. J Am Geriatr Soc. 2008 Jul;56(7):1349-56. doi: 10.1111/j.1532-5415.2008.01770.x. Epub
2008 May 22. PubMed PMID: 18503520.

Kates SL, Mendelson DA, Friedman SM. The value of an organized fracture program for the elderly: early
results. J Orthop Trauma. 2011 Apr;25(4):233-7. doi: 10.1097/BOT.0b013e3181e5e901. PubMed PMID:
21399474.

Ricci WM, Della Rocca GJ, Combs C, Borrelli J. The medical and economic impact of preoperative
cardiac testing in elderly patients with hip fractures. Injury. 2007 Sep;38 Suppl 3:S49-52. PubMed PMID:
17723792.

Potter JF. The older orthopaedic patient: general considerations. Clin Orthop Relat Res. 2004
Aug;(425):44-9. Review. PubMed PMID: 15292786.

Question 269
A 76-year-old man has a Vancouver type C periprosthetic fracture. What is the best option to fix this
fracture?

1. A plate with the proximal extent 2 canal diameters above the tip of the femoral prosthesis
2. A plate with the proximal extent 2 canal diameters below the tip of the femoral prosthesis
3. A plate that stops adjacent to the tip of the femoral prosthesis
4. A long-stem revision of the prosthesis
5. A long-stem revision of the prosthesis with a plate placed 2 canal diameters above the tip of
the prosthesis.

PREFERRED RESPONSE: 1

RECOMMENDED READINGS
Masri BA, Meek RM, Duncan CP. Periprosthetic fractures evaluation and treatment. Clin Orthop Relat
Res. 2004 Mar;(420):80-95. Review. PubMed PMID: 15057082.

Wolff LH III, Berry DJ. Periprosthetic fractures of the lower extremity. In: Browner BD, Jupiter
JB, Levine AM, Trafton PG, Krettek C, eds. Skeletal Trauma: Basic Science, Management, and
Reconstruction. Vol 2. 4th ed. Philadelphia, PA: Saunders Elsevier; 2011:2843-2858.

© 2015 American Academy of Orthopaedic Surgeons 2015 Orthopaedic In-Training Examination


SECTION 3: Hand • 103

SECTION 3: Hand

Question 8
A 76-year-old woman underwent a left below-elbow amputation. Prior to the amputation, she had a
stroke that caused a paretic, painful, clenched hand. Her hand became severely infected, necessitating
amputation. She continues to have phantom pain, perceiving that her amputated hand is clenched. What is
the best therapy modality at this time?

1. Iontophoresis
2. Contrast baths
3. Mirror therapy
4. Low-level laser therapy
5. Electrical stimulation

PREFERRED RESPONSE: 3

RECOMMENDED READINGS
Hartzell TL, Rubinstein R, Herman M. Therapeutic modalities--an updated review for the hand surgeon.
J Hand Surg Am. 2012 Mar;37(3):597-621. doi: 10.1016/j.jhsa.2011.12.042. Epub 2012 Feb 2. Review.
PubMed PMID: 22305724.

Rostami HR, Arefi A, Tabatabaei S. Effect of mirror therapy on hand function in patients with hand
orthopaedic injuries: a randomized controlled trial. Disabil Rehabil. 2013 Sep;35(19):1647-51. doi:
10.3109/09638288.2012.751132. Epub 2013 Jan 22. PubMed PMID: 23336124.

Moseley GL. Graded motor imagery is effective for long-standing complex regional pain syndrome: a
randomised controlled trial. Pain. 2004 Mar;108(1-2):192-8. PubMed PMID: 15109523. 

© 2015 American Academy of Orthopaedic Surgeons 2015 Orthopaedic In-Training Examination


104 • American Academy of Orthopaedic Surgeons

Question 15
A 40-year-old woman has a closed oblique ring finger metacarpal fracture. There is no rotational
deformity. Radiographs reveal a 10-degree apex-dorsal angulation and 2 mm of shortening. What is the
expected outcome for fracture healing in this position?

1. Nonunion
2. Normal function
3. Extensor tendonitis
4. Flexion deformity of the metacarpophalangeal (MCP) joint
5. Flexion deformity of the proximal interphalangeal (PIP) joint

PREFERRED RESPONSE: 2

RECOMMENDED READINGS
Strauch RJ, Rosenwasser MP, Lunt JG. Metacarpal shaft fractures: the effect of shortening on the extensor
tendon mechanism. J Hand Surg Am. 1998 May;23(3):519-23. PubMed PMID: 9620194.

Al-Qattan MM. Outcome of conservative management of spiral/long oblique fractures of the metacarpal
shaft of the fingers using a palmar wrist splint and immediate mobilisation of the fingers. J Hand Surg
Eur Vol. 2008 Dec;33(6):723-7. doi: 10.1177/1753193408093559. Epub 2008 Jul 28. PubMed PMID:
18662959.

© 2015 American Academy of Orthopaedic Surgeons 2015 Orthopaedic In-Training Examination


SECTION 3: Hand • 105

Figure 28

Question 28
Figure 28 shows an injury sustained by a 60-year-old man 4 weeks ago. Since that time he has had
substantial pain and catching of his finger during attempts at range of motion. What is the most
appropriate treatment at this point?

1. Tendon debridement
2. Release of the A2 pulley
3. Tendon repair with core sutures
4. Tendon repair with epitendinous sutures
5. Wound closure without tendon repair or debridement

PREFERRED RESPONSE: 4

RECOMMENDED READINGS
Haddad R, Scherman P, Peltz T, Nicklin S, Walsh WR. A biomechanical assessment of repair versus
nonrepair of sheep flexor tendons lacerated to 75 percent. J Hand Surg Am. 2010 Apr;35(4):546-51. doi:
10.1016/j.jhsa.2009.12.039. Epub 2010 Mar 2. PubMed PMID: 20189731.

Mostofi A, Palmer J, Akelman E. Flexor tendon injury. In: Chung KC, ed. Hand Surgery Update V.
Rosemont, IL: American Academy of Orthopaedic Surgeons; 2012:181-192.

© 2015 American Academy of Orthopaedic Surgeons 2015 Orthopaedic In-Training Examination


106 • American Academy of Orthopaedic Surgeons

Figure 39

Question 39
Figure 39 is the clinical photograph of a 2-year-boy who has bilateral camptodactyly of 25 degrees at
each of his ring finger proximal interphalangeal joints. No other abnormalities are present, and his parents
report developmental milestones have all been reached on time. The boy’s father is concerned that the
contractures will negatively influence his ability to engage in sports. What is the best next step?

1. Observation
2. Volar plate release
3. Proximal phalanx extension osteotomies
4. A progressive stretching and splinting program
5. Rerouting a slip of the flexor digitorum superficialis to the extensor central slip

PREFERRED RESPONSE: 4

RECOMMENDED READINGS
Goldfarb CA. Congenital hand anomalies: a review of the literature, 2009-2012. J Hand Surg Am.
2013 Sep;38(9):1854-9. doi: 10.1016/j.jhsa.2013.03.023. Epub 2013 May 14. Review. PubMed PMID:
23683863.

Rhee SH, Oh WS, Lee HJ, Roh YH, Lee JO, Baek GH. Effect of passive stretching on simple
camptodactyly in children younger than three years of age. J Hand Surg Am. 2010 Nov;35(11):1768-73.
doi: 10.1016/j.jhsa.2010.07.032. PubMed PMID: 21050962.

© 2015 American Academy of Orthopaedic Surgeons 2015 Orthopaedic In-Training Examination


SECTION 3: Hand • 107

Question 48
A 23-year-old man cut the dorsal and ulnar aspects of his long finger on a table saw. The dorsal and ulnar
skin over the middle phalanx is missing, with a 2-cm x 2-cm area of loss. There is a 50% loss of the
extensor tendon (ulnar), and the remaining tendon has no tenosynovium. The physician should recommend
irrigation and debridement and

1. wet-to-dry dressing and early motion.


2. tendon repair, and thenar flap coverage.
3. full-thickness skin graft.
4. reversed cross-finger flap from the ring finger.
5. cross-finger flap coverage from the ring finger.

PREFERRED RESPONSE: 4

RECOMMENDED READINGS
Atasoy E. Reversed cross-finger subcutaneous flap. J Hand Surg Am. 1982 Sep;7(5):481-3. PubMed
PMID: 7130658.

Kappel DA, Burech JG. The cross-finger flap. An established reconstructive procedure. Hand Clin. 1985
Nov;1(4):677-83. PubMed PMID: 3831054. 

Question 58
A 25-year-old snowboarder has a contaminated open tibial fracture. He was down for several hours before
being rescued and transported. He also sustained a cold injury to his fingers, which are blue and have
multiple hemorrhagic blisters. What is the most appropriate timing for aggressive finger debridement/
amputation?

1. Prior to definitive fixation of the tibial fracture


2. Prior to irrigation and debridement (I & D) of the tibial fracture
3. During I & D of the tibial fracture
4. During definitive fixation of the tibia
5. No set time; delay until necrotic tissue demarcation

PREFERRED RESPONSE: 5

RECOMMENDED READINGS
Golant A, Nord RM, Paksima N, Posner MA. Cold exposure injuries to the extremities. J Am Acad Orthop
Surg. 2008 Dec;16(12):704-15. Review. PubMed PMID:19056919.

Wolfe SW, Hotchkiss RN, Pederson WC, Kozin SH, eds. Green’s Operative Hand Surgery. 2nd ed.
Philadelphia, PA: Churchill Livingstone Elsevier; 2011:2114-2116.

© 2015 American Academy of Orthopaedic Surgeons 2015 Orthopaedic In-Training Examination


108 • American Academy of Orthopaedic Surgeons

Question 65
Which examination finding points toward a brachial plexus injury rather than root avulsion?

1. Winging of the scapula


2. Intact rhomboid function
3. A biceps with 0/5 strength
4. An ipsilateral clavicle fracture
5. Decreased radial artery pulse

PREFERRED RESPONSE: 2

RECOMMENDED READINGS
Caporrino FA, Moreira L, Moraes VY, Belloti JC, Gomes dos Santos JB, Faloppa F. Brachial plexus
injuries: diagnosis performance and reliability of everyday tools. Hand Surg. 2014;19(1):7-11. doi:
10.1142/S0218810414500026. PubMed PMID: 24641734.

Tubbs RS, Tyler-Kabara EC, Aikens AC, Martin JP, Weed LL, Salter EG, Oakes WJ. Surgical anatomy of
the dorsal scapular nerve. J Neurosurg. 2005 May;102(5):910-1. PubMed PMID: 15926718.

Question 75
A 72-year-old woman with diabetes mellitus has right hand numbness. Provocative test findings are
consistent with carpal tunnel syndrome, and electrodiagnostic study (EDS) findings show prolonged
median motor and sensory distal latencies with low-amplitude thenar compound muscle action potential.
Poor prognosis is most associated with which factor?

1. Diabetes
2. Older age
3. Female gender
4. Right-hand involvement
5. Severity of EDS findings

PREFERRED RESPONSE: 5

© 2015 American Academy of Orthopaedic Surgeons 2015 Orthopaedic In-Training Examination


SECTION 3: Hand • 109

RECOMMENDED READINGS
Kronlage SC, Menendez ME. The benefit of carpal tunnel release in patients with electrophysiologically
moderate and severe disease. J Hand Surg Am. 2015 Mar;40(3):438-444.e1. doi: 10.1016/j.
jhsa.2014.12.012. PubMed PMID: 25708432.

Bland JD. Carpal tunnel syndrome. BMJ. 2007 Aug 18;335(7615):343-6. Review. PubMed PMID:
17703044.

Bland JD. Do nerve conduction studies predict the outcome of carpal tunnel decompression? Muscle
Nerve. 2001 Jul;24(7):935-40. PubMed PMID: 11410921.

Tomaino MM, Weiser RW. Carpal tunnel release for advanced disease in patients 70 years and older: does
outcome from the patient’s perspective justify surgery? J Hand Surg Br. 2001 Oct;26(5):481-3. PubMed
PMID: 11560433.

Stevens JC. AAEM minimonograph #26: the electrodiagnosis of carpal tunnel syndrome. American
Association of Electrodiagnostic Medicine. Muscle Nerve. 1997 Dec;20(12):1477-86. Review. PubMed
PMID: 9390659.

Question 88
Which method of flexor tendon repair that necessitates excursion through the A2 pulley allows for the
most thorough assessment of tendon gliding?

1. 4-strand repair with 6-0 epitendinous suture with Bier block anesthesia
2. 4-strand repair with 6-0 epitendinous suture under local anesthesia only
3. 6-strand repair with regional anesthesia
4. 8-strand repair with regional anesthesia
5. Repair of the flexor tendon with incision of the remaining A2 pulley

PREFERRED RESPONSE: 2

RECOMMENDED READINGS
Lalonde D. Minimally invasive anesthesia in wide awake hand surgery. Hand Clin. 2014 Feb;30(1):1-6.
doi: 110.1016/j.hcl.2013.08.015. Epub 2013 Nov 9. Review. PubMed PMID: 24286736.

Lalonde DH, Martin AL. Wide-awake flexor tendon repair and early tendon mobilization in zones 1 and 2.
Hand Clin. 2013 May;29(2):207-13. doi: 10.1016/j.hcl.2013.02.009. Epub 2013 Mar 15. Review. PubMed
PMID: 23660056.

© 2015 American Academy of Orthopaedic Surgeons 2015 Orthopaedic In-Training Examination


110 • American Academy of Orthopaedic Surgeons

Figure 96a Figure 96b

Question 96
Figures 96a and 96b are the radiographs of an 18-year-old man who had surgery 6 months ago at an
outside institution. He is being referred now because he has persistent pain. He is tender over the scaphoid
at the snuffbox. What is the most appropriate next imaging step in his pain workup?

1. Bone scan
2. MR imaging with contrast
3. MR imaging without contrast
4. CT scan along the scaphoid axis
5. Axial-cut CT scans with reformats

PREFERRED RESPONSE: 4

RECOMMENDED READINGS
Ring D, Jupiter JB, Herndon JH. Acute fractures of the scaphoid. J Am Acad Orthop Surg. 2000 Jul-
Aug;8(4):225-31. Review. PubMed PMID: 10951111.

Lutsky K, Matzon JL. Persistent fracture line after scaphoid fracture fixation. J Hand Surg Am. 2014
Nov;39(11):2294-6. doi: 10.1016/j.jhsa.2014.08.030. Epub 2014 Oct 3. PubMed PMID: 25282721.

Yin ZG, Zhang JB, Gong KT. Cost-Effectiveness of Diagnostic Strategies for Suspected Scaphoid
Fractures. J Orthop Trauma. 2015 Aug;29(8):e245-52. doi: 10.1097/BOT.0000000000000316. PubMed
PMID: 25756914.

Yin ZG, Zhang JB, Kan SL, Wang XG. Diagnosing suspected scaphoid fractures: a systematic review and
meta-analysis. Clin Orthop Relat Res. 2010 Mar;468(3):723-34. doi: 10.1007/s11999-009-1081-6. Epub
2009 Sep 15. Review. PubMed PMID: 19756904.


© 2015 American Academy of Orthopaedic Surgeons 2015 Orthopaedic In-Training Examination
SECTION 3: Hand • 111

Figure 103

Question 103 (Item Deleted)


Which muscle or tendon function is most likely disrupted by the finding in the MR image shown in
Figure 103?

1. First dorsal interosseous


2. Flexor pollicis longus (FPL)
3. Long-finger flexor digitorum profundus (FDP)
4. Small-finger FDP
5. Extensor pollicis longus (EPL)

PREFERRED RESPONSE: 4

RECOMMENDED READINGS
Yamazaki H, Kato H, Nakatsuchi Y, Murakami N, Hata Y. Closed rupture of the flexor tendons of the little
finger secondary to non-union of fractures of the hook of the hamate. J Hand Surg Br. 2006 Jun;31(3):337-
41. Epub 2006 Mar 30. PubMed PMID: 16580104.

Klausmeyer MA, Mudgal CS. Hook of hamate fractures. J Hand Surg Am. 2013 Dec;38(12):2457-60; quiz
2460. doi: 10.1016/j.jhsa.2013.06.004. Epub 2013 Jul 26. Review. PubMed PMID: 23891177.

© 2015 American Academy of Orthopaedic Surgeons 2015 Orthopaedic In-Training Examination


112 • American Academy of Orthopaedic Surgeons

Question 133
A 34-year-old smoker has acute (less than 6 hours) onset of right hand ischemia. His blood pressure is
188/90 mm Hg. His hand is cool and pale; however, he can move his fingers with some discomfort. There
is no swelling and no history of trauma. Radial and ulnar pulses are not palpable, but they are present by
Doppler examination. What is the best next step?

1. Angiogram
2. Ulnar artery thrombectomy
3. Magnetic resonance angiogram (MRA)
4. Transthoracic echocardiogram
5. Measurement of compartment pressures

PREFERRED RESPONSE: 1

RECOMMENDED READINGS
Rapp JH, Reilly LM, Goldstone J, Krupski WC, Ehrenfeld WK, Stoney RJ. Ischemia of the upper
extremity: significance of proximal arterial disease. Am J Surg. 1986 Jul;152(1):122-6. PubMed PMID:
3728805.

Pellerin O, Delorme L, Bellmann L, Sapoval M. Clinical presentation and percutaneous endovascular


management of acute left subclavian artery thrombosis: report of two cases. Diagn Interv Imaging. 2014
Jan;95(1):95-9. doi:10.1016/j.diii.2013.07.005. Epub 2013 Sep 4. PubMed PMID: 24012286.

© 2015 American Academy of Orthopaedic Surgeons 2015 Orthopaedic In-Training Examination


SECTION 3: Hand • 113

Figure 140

Question 140 (Item Deleted)


Figure 140 is the T1-weighted MR image of a 24-year-old man who has a 6-month history of right wrist
pain. He fell on an outstretched hand while skateboarding 18 months ago and had pain that resolved after
several weeks. Radiographs of his right wrist demonstrate an established scaphoid nonunion at the waist
with a humpback deformity and no signs of arthritis. What is the best next step?

1. Proximal row carpectomy


2. Scaphoid excision and 4-corner fusion
3. Vascularized dorsal distal radial pedicle graft and fixation
4. Vascularized medial femoral condyle graft and fixation
5. Open reduction and internal fixation with a cancellous bone graft

PREFERRED RESPONSE: 4

RECOMMENDED READINGS
Al-Jabri T, Mannan A, Giannoudis P. The use of the free vascularised bone graft for nonunion of the
scaphoid: a systematic review. J Orthop Surg Res. 2014 Apr 1;9:21. doi: 10.1186/1749-799X-9-21.
Review. PubMed PMID: 24690301; PubMed Central PMCID: PMC3976175.

Jones DB Jr, Bьrger H, Bishop AT, Shin AY. Treatment of scaphoid waist nonunions with an avascular
proximal pole and carpal collapse. A comparison of two vascularized bone grafts. J Bone Joint Surg Am.
2008 Dec;90(12):2616-25. doi: 10.2106/JBJS.G.01503. PubMed PMID: 19047706.

© 2015 American Academy of Orthopaedic Surgeons 2015 Orthopaedic In-Training Examination


114 • American Academy of Orthopaedic Surgeons

Question 175
During closed reduction of an apex-volar angulated distal radius fracture, which maneuver, in addition to
traction, likely will provide the best fracture reduction?

1. Pronation of the wrist


2. Ulnar deviation of the wrist
3. Volar translation of the lunate
4. Flexion of the lunocapitate joint
5. Extension of the radiocarpal joint

PREFERRED RESPONSE: 3

RECOMMENDED READINGS
Agee JM. Distal radius fractures. Multiplanar ligamentotaxis. Hand Clin. 1993 Nov;9(4):577-85. Review.
PubMed PMID: 8300728.

Agee JM, Szabo RM, Chidgey LK, King FC, Kerfoot C. Treatment of comminuted distal radius fractures:
an approach based on pathomechanics. Orthopedics. 1994 Dec;17(12):1115-22. PubMed PMID: 7899154. 

Question 228
When viewing the wrist joint arthroscopically from the 3-4 portal, what is the correct order of the volar
extrinsic wrist ligaments from radial to ulnar?

1. Scapholunate, lunotriquetral, ulnotriquetral


2. Short radiolunate, long radiolunate, radioscaphocapitate
3. Radioscapholunate, radioscaphocapitate, long radiolunate
4. Radioscaphocapitate, long radiolunate, short radiolunate
5. Radial collateral, long radiolunate, short radiolunate

PREFERRED RESPONSE: 4

RECOMMENDED READINGS
Bettinger PC, Cooney WP 3rd, Berger RA. Arthroscopic anatomy of the wrist. Orthop Clin North Am.
1995 Oct;26(4):707-19. PubMed PMID: 7566915.

Berger RA. Arthroscopic anatomy of the wrist and distal radioulnar joint. Hand Clin. 1999 Aug;15(3):393-
413, vii. Review. PubMed PMID: 10451815.

Berger RA. The anatomy of the ligaments of the wrist and distal radioulnar joints. Clin Orthop Relat Res.
2001 Feb;(383):32-40. Review. PubMed PMID: 11210966.

© 2015 American Academy of Orthopaedic Surgeons 2015 Orthopaedic In-Training Examination


SECTION 3: Hand • 115

Question 246
A 32-year-old man has a thumb metacarpophalangeal joint (MCP) ulnar collateral ligament (UCL) injury.
The joint is unstable, and an MR image reveals a displaced distal avulsion of the ligament off the base of
the proximal phalanx. During repair, which structure(s) block(s) reduction of the ligament?

1. Adductor aponeurosis
2. Extensor pollicis longus tendon
3. Extensor pollicis brevis (EPB) tendon
4. Ulnar sesamoid bone
5. EPB and dorsal capsule

PREFERRED RESPONSE: 1

RECOMMENDED READINGS
Stener B. Skeletal injuries associated with rupture of the ulnar collateral ligament of the
metacarpophalangeal joint of the thumb. A clinical and anatomical study. Acta Chir Scand. 1963
Jun;125:583-6. PubMed PMID: 13983826.

Bean CH, Tencer AF, Trumble TE. The effect of thumb metacarpophalangeal ulnar collateral ligament
attachment site on joint range of motion: an in vitro study. J Hand Surg Am. 1999 Mar;24(2):283-7.
PubMed PMID: 10194011.

Carlson MG, Warner KK, Meyers KN, Hearns KA, Kok PL. Anatomy of the thumb metacarpophalangeal
ulnar and radial collateral ligaments. J Hand Surg Am. 2012 Oct;37(10):2021-6. doi: 10.1016/j.
jhsa.2012.06.024. Epub 2012 Aug 31. PubMed PMID: 22939823.

Question 256
A 25-year-old man has an acute scaphoid fracture in the proximal third of the scaphoid. It is displaced 1
mm. What is the recommended treatment for his fracture?

1. Long-arm cast with a thumb spica


2. Closed reduction and pin fixation with casting
3. Open reduction and internal fixation (ORIF) through a volar approach
4. ORIF with vascularized bone graft
5. ORIF through a dorsal approach

PREFERRED RESPONSE: 5

RECOMMENDED READINGS
Rettig ME, Kozin SH, Cooney WP. Open reduction and internal fixation of acute displaced scaphoid waist
fractures. J Hand Surg Am. 2001 Mar;26(2):271-6. PubMed PMID: 11279573.

Raskin KB, Parisi D, Baker J, Rettig ME. Dorsal open repair of proximal pole scaphoid fractures. Hand
Clin. 2001 Nov;17(4):601-10, ix. PubMed PMID: 11775471.

© 2015 American Academy of Orthopaedic Surgeons 2015 Orthopaedic In-Training Examination
116 • American Academy of Orthopaedic Surgeons

Question 263
A 50-year-old man experienced a dorsal dislocation of his thumb carpometacarpal (CMC) joint without an
associated fracture 1 week ago. He self-reduced his thumb. Radiographs reveal slight subluxation of the
joint with minimal arthritis. Which region of the thumb CMC capsuloligamentous complex most likely
prevents dorsal dislocation of the thumb CMC joint?

1. Dorsal radial
2. Intermetacarpal
3. Ulnar collateral
4. Volar oblique deep
5. Volar oblique superficial

PREFERRED RESPONSE: 1

RECOMMENDED READINGS
Strauch RJ, Behrman MJ, Rosenwasser MP. Acute dislocation of the carpometacarpal joint of the thumb:
an anatomic and cadaver study. J Hand Surg Am. 1994 Jan;19(1):93-8. PubMed PMID: 8169374.

Bettinger PC, Berger RA. Functional ligamentous anatomy of the trapezium and trapeziometacarpal joint
(gross and arthroscopic). Hand Clin. 2001 May;17(2):151-68, vii. Review. PubMed PMID: 11478038.

Question 267
After performing an uneventful partial palmar fasciectomy for Dupuytren contracture of the palm and ring
finger, a general postsurgical pain medication prescription should include how many narcotic pills?

1. 0
2. 10
3. 20
4. 30
5. 40

PREFERRED RESPONSE: 2

RECOMMENDED READINGS
Rodgers J, Cunningham K, Fitzgerald K, Finnerty E. Opioid consumption following outpatient upper
extremity surgery. J Hand Surg Am. 2012 Apr;37(4):645-50. doi: 10.1016/j.jhsa.2012.01.035. Epub 2012
Mar 10. PubMed PMID: 22410178.

Stanek JJ, Renslow MA, Kalliainen LK. The effect of an educational program on opioid prescription
patterns in hand surgery: a quality improvement program. J Hand Surg Am. 2015 Feb;40(2):341-6. doi:
10.1016/j.jhsa.2014.10.054. Epub 2014 Dec 24. PubMed PMID: 25542435.

© 2015 American Academy of Orthopaedic Surgeons 2015 Orthopaedic In-Training Examination


SECTION 3: Hand • 117

Question 271
A 42-year-old worker has failed nonsurgical treatment of painful thumb carpometacarpal (CMC) arthritis.
The decision has been made to perform ligamentous reconstruction and tendon interposition arthroplasty.
However, when the bony ridge of the trapezium is removed, the entire distal attachment of the flexor carpi
radialis (FCR) tendon is inadvertently detached. What is the best next step?

1. Perform suspension arthroplasty with the extensor carpi radialis longus (ECRL)
2. Perform a suspension arthroplasty with the ring finger flexor digitorum superficialis
3. Perform a suspension arthroplasty with the palmaris longus tendon
4. Reattach the insertion of the FCR with suture anchors
5. Perform a CMC arthrodesis

PREFERRED RESPONSE: 1

RECOMMENDED READINGS
Jones DB Jr, Rhee PC, Shin AY, Kakar S. Salvage options for flexor carpi radialis tendon disruption during
ligament reconstruction and tendon interposition or suspension arthroplasty of the trapeziometacarpal
joint. J Hand Surg Am. 2013 Sep;38(9):1806-11. doi: 10.1016/j.jhsa.2013.06.014. Epub 2013 Aug 6.
PubMed PMID: 23928018.

Conolly WB, Rath S. Revision procedures for complications of surgery for osteoarthritis of the
carpometacarpal joint of the thumb. J Hand Surg Br. 1993 Aug;18(4):533-9. PubMed PMID: 8409675.

© 2015 American Academy of Orthopaedic Surgeons 2015 Orthopaedic In-Training Examination


118 • American Academy of Orthopaedic Surgeons

SECTION 4: Hip and Knee

Question 5
When performing total hip arthroplasty, what is the traditionally accepted acetabular component’s ideal
position or “safe zone” within which risk for dislocation is minimized?

1. 20 to 40 degrees of abduction and 0 to 20 degrees of anteversion


2. 25 to 45 degrees of abduction and 0 to 20 degrees of anteversion
3. 25 to 45 degrees of abduction and 5 to 25 degrees of anteversion
4. 30 to 50 degrees of abduction and 5 to 25 degrees of anteversion
5. 35 to 55 degrees of abduction and 10 to 35 degrees of anteversion

PREFERRED RESPONSE: 4

RECOMMENDED READINGS
Lewinnek GE, Lewis JL, Tarr R, Compere CL, Zimmerman JR. Dislocations after total hip-replacement
arthroplasties. J Bone Joint Surg Am. 1978 Mar;60(2):217-20. PubMed PMID: 641088.

Barrack RL, Krempec JA, Clohisy JC, McDonald DJ, Ricci WM, Ruh EL, Nunley RM. Accuracy of
acetabular component position in hip arthroplasty. J Bone Joint Surg Am. 2013 Oct 2;95(19):1760-8. doi:
10.2106/JBJS.L.01704. PubMed PMID: 24088968.

Liu F, Gross TP. A safe zone for acetabular component position in metal-on-metal hip resurfacing
arthroplasty: winner of the 2012 HAP PAUL award. J Arthroplasty. 2013 Aug;28(7):1224-30. doi:
10.1016/j.arth.2013.02.033. Epub 2013 Mar 26. PubMed PMID: 23540536. 

© 2015 American Academy of Orthopaedic Surgeons 2015 Orthopaedic In-Training Examination


SECTION 4: Hip and Knee • 119

Figure 20

Question 20
Which mode of corrosion best describes the likely failure mechanism shown in Figure 20?

1. Pitting
2. Fretting
3. Crevice
4. Galvanic
5. Intergranular

PREFERRED RESPONSE: 2

RECOMMENDED READINGS
Buttaro MA, Mayor MB, Van Citters D, Piccaluga F. Fatigue fracture of a proximally modular, distally
tapered fluted implant with diaphyseal fixation. J Arthroplasty. 2007 Aug;22(5):780-3. Epub 2007 Mar 28.
PubMed PMID: 17689793.

Lakstein D, Eliaz N, Levi O, Backstein D, Kosashvili Y, Safir O, Gross AE. Fracture of cementless
femoral stems at the mid-stem junction in modular revision hip arthroplasty systems. J Bone Joint Surg
Am. 2011 Jan 5;93(1):57-65. doi: 10.2106/JBJS.I.01589. PubMed PMID: 21209269. 

© 2015 American Academy of Orthopaedic Surgeons 2015 Orthopaedic In-Training Examination


120 • American Academy of Orthopaedic Surgeons

Figure 30a Figure 30b

Figure 30c

Question 30
Figures 30a through 30c are the radiograph and CT scans of a 70-year-old woman who has persistent
groin and lateral hip pain 6 months after undergoing open reduction and internal fixation (ORIF) of a hip
fracture. Physical therapy, topical and oral nonsteroidal anti-inflammatory drugs, and activity modification
have not provided relief. She had temporary but incomplete relief of her lateral-sided pain after receiving a
trochanteric bursal injection, but she now uses a cane. What is the best treatment option?

1. Total hip arthroplasty (THA)


2. Open bursectomy
3. Bone grafting and revision ORIF
4. Hardware removal
5. Valgus intertrochanteric osteotomy

PREFERRED RESPONSE: 1

© 2015 American Academy of Orthopaedic Surgeons 2015 Orthopaedic In-Training Examination


SECTION 4: Hip and Knee • 121

RECOMMENDED READINGS
Yang JJ, Lin LC, Chao KH, Chuang SY, Wu CC, Yeh TT, Lian YT. Risk factors for nonunion in patients
with intracapsular femoral neck fractures treated with three cannulated screws placed in either a triangle
or an inverted triangle configuration. J Bone Joint Surg Am. 2013 Jan 2;95(1):61-9. doi:10.2106/
JBJS.K.01081. PubMed PMID: 23283374.

Archibeck MJ, Carothers JT, Tripuraneni KR, White RE Jr. Total hip arthroplasty after failed internal
fixation of proximal femoral fractures. J Arthroplasty. 2013 Jan;28(1):168-71. doi: 10.1016/j.
arth.2012.04.003. Epub 2012 Jun 6. PubMed PMID: 22682040. 

Figure 44a Figure 44b

Question 44
When performing total knee arthroplasty on the patient whose radiographs are shown in Figures 44a and
44b, particular attention should be directed toward avoiding which intraoperative femoral component
placement error?

1. Flexion
2. Oversize
3. Undersize
4. Internal rotation
5. External rotation

PREFERRED RESPONSE: 4

© 2015 American Academy of Orthopaedic Surgeons 2015 Orthopaedic In-Training Examination


122 • American Academy of Orthopaedic Surgeons

RECOMMENDED READINGS
Miller MC, Berger RA, Petrella AJ, Karmas A, Rubash HE. Optimizing femoral component rotation in
total knee arthroplasty. Clin Orthop Relat Res. 2001 Nov;(392):38-45. PubMed PMID: 11716411.

Steinberg ME, Bands RE, Parry S, Hoffman E, Chan T, Hartman KM. Does lesion size affect the outcome
in avascular necrosis? Clin Orthop Relat Res. 1999 Oct;(367):262-71. PubMed PMID: 10546624.

Victor J. Rotational alignment of the distal femur: a literature review. Orthop Traumatol Surg Res. 2009
Sep;95(5):365-72. doi: 10.1016/j.otsr.2009.04.011. Epub 2009 Jul 9. Review. PubMed PMID: 19592323.

Pagnano MW, Hanssen AD. Varus tibial joint line obliquity: a potential cause of femoral component
malrotation. Clin Orthop Relat Res. 2001 Nov;(392):68-74. PubMed PMID: 11716427. 

Figure 50a Figure 50b

Question 50
Figures 50a and 50b are the MR images of a 40-year-old man who has left hip pain. He has a history
of steroid use. When using the modified Kerboul method, angle degree exceeding which measure is
associated with high risk for progression to femoral head collapse?

1. 150 degrees
2. 190 degrees
3. 220 degrees
4. 240 degrees
5. 270 degrees

PREFERRED RESPONSE: 4

© 2015 American Academy of Orthopaedic Surgeons 2015 Orthopaedic In-Training Examination


SECTION 4: Hip and Knee • 123

RECOMMENDED READINGS
Glassman AH, Lachiewicz PF, Tanzer M, eds. Orthopaedic Knowledge Update: Hip and Knee
Reconstruction. 4th ed. Rosemont, IL: American Academy of Orthopaedic Surgeons; 2011.

Ha YC, Jung WH, Kim JR, Seong NH, Kim SY, Koo KH. Prediction of collapse in femoral head
osteonecrosis: a modified Kerboul method with use of magnetic resonance images. J Bone Joint Surg Am.
2006 Nov;88 Suppl 3:35-40. PubMed PMID: 17079365.

Question 59
A 58-year-old healthy woman had hip arthroplasty 3 years ago with a recalled modular stem and metal-
on-metal articulation. She continued to have hip pain following her surgery. Her cobalt and chromium ion
levels are elevated, but her C-reactive protein level and erythrocyte sedimentation rate are within defined
limits. A small fluid collection is seen around the hip on metal artifact reduction sequence MRI. At the
time of revision, there was a large amount of cloudy fluid around the hip joint. Her cell count is 1000/mm3
with 45% nucleated cells (by manual count). The femoral and acetabular components are in good position
and are well fixed to the bone. There is evidence of taper corrosion. What is the most appropriate next
step?

1. Remove all components and place the cement spacer.


2. Remove all components and revise the femur and acetabulum.
3. Retain the components and place a new femoral head and acetabular liner.
4. Revise the femoral component and acetabular liner.
5. Close the wound and await culture results before proceeding with the revision.

PREFERRED RESPONSE: 4

RECOMMENDED READINGS
Goyal N, Ho H, Fricka KB, Engh CA Jr. Do you have to remove a corroded femoral stem? J Arthroplasty.
2014 Sep;29(9 Suppl):139-42. doi: 10.1016/j.arth.2014.03.055. Epub 2014 May 27. PubMed
PMID: 24973927.

Pivec R, Meneghini RM, Hozack WJ, Westrich GH, Mont MA. Modular taper junction corrosion and
failure: how to approach a recalled total hip arthroplasty implant. J Arthroplasty. 2014 Jan;29(1):1-6. doi:
10.1016/j.arth.2013.08.026. Epub 2013 Sep 30. Review. PubMed PMID: 24090661.

Wyles CC, Larson DR, Houdek MT, Sierra RJ, Trousdale RT. Utility of synovial fluid aspirations in
failed metal-on-metal total hip arthroplasty. J Arthroplasty. 2013 May;28(5):818-23. doi: 10.1016/j.
arth.2012.11.006. Epub 2013 Mar 15. PubMed PMID: 23499404.

© 2015 American Academy of Orthopaedic Surgeons 2015 Orthopaedic In-Training Examination


124 • American Academy of Orthopaedic Surgeons

Question 76
A 125-pound, 65-year-old woman fell from a height and sustained an isolated displaced femoral shaft
fracture. There is concern for hemorrhage because she also has chronic anemia. What is the average
estimated adult blood loss when such an injury occurs?

1. 250 cc
2. 500 cc
3. 1250 cc
4. 2250 cc
5. 3250 cc

PREFERRED RESPONSE: 3

RECOMMENDED READINGS
Lieurance R, Benjamin JB, Rappaport WD. Blood loss and transfusion in patients with isolated femur
fractures. J Orthop Trauma. 1992;6(2):175-9. PubMed PMID: 1602337.

Lee C, Porter KM. Prehospital management of lower limb fractures. Emerg Med J. 2005 Sep;22(9):660-3.
Review. PubMed PMID: 16113195.

Question 98
A 64-year-old Jehovah’s witness has end-stage hip arthritis. The decision is made to proceed with total
hip arthroplasty. Which intervention will most effectively reduce potential for postoperative anemia and
subsequent transfusion?

1. General anesthesia
2. Bipolar sealer electrocautery
3. Preoperative iron supplementation
4. Hypotensive epidural/spinal anesthesia
5. Local skin infiltration with anesthetic/epinephrine

PREFERRED RESPONSE: 4

RECOMMENDED READINGS
Moonen AF, Neal TD, Pilot P. Peri-operative blood management in elective orthopaedic surgery. A
critical review of the literature. Injury. 2006 Dec;37 Suppl 5:S11-6. Review. Erratum in: Injury. 2007
Oct;38(10):1224. PubMed PMID: 17338906.

Morris MJ, Barrett M, Lombardi AV Jr, Tucker TL, Berend KR. Randomized blinded study comparing
a bipolar sealer and standard electrocautery in reducing transfusion requirements in anterior
supine intermuscular total hip arthroplasty. J Arthroplasty. 2013 Oct;28(9):1614-7. doi: 10.1016/j.
arth.2013.01.032. Epub 2013 Mar 16. PubMed PMID: 23507071.

© 2015 American Academy of Orthopaedic Surgeons 2015 Orthopaedic In-Training Examination


SECTION 4: Hip and Knee • 125

Figure 108a Figure 108b Figure 108c

Question 108
Figures 108a through 108c are the radiographs and clinical photograph of a 50-year-old woman who
underwent uncomplicated total knee arthroplasty 1 year ago. She was never satisfied with the results of
her surgery. She fell at home and was admitted to the hospital with severe knee pain. Her medical issues
include atrial fibrillation, vascular insufficiency, and obesity. Her C-reactive protein level and erythrocyte
sedimentation rate are elevated. Knee aspiration reveals a bloody effusion, and a cell count could not be
obtained. Cultures show no growth after 2 days. What is the most appropriate next step?

1. 2-component revision
2. Revision of the tibial component only
3. Removal of the components and placement of a cement spacer
4. Knee immobilizer, limited weight bearing, and delayed revision after her skin
quality improves
5. Knee fusion

PREFERRED RESPONSE: 3

RECOMMENDED READINGS
Watts CD, Wagner ER, Houdek MT, Osmon DR, Hanssen AD, Lewallen DG, Mabry TM. Morbid obesity:
a significant risk factor for failure of two-stage revision total knee arthroplasty for infection. J Bone Joint
Surg Am. 2014 Sep 17;96(18):e154. doi: 10.2106/JBJS.M.01289. PubMed PMID: 25232084.

Kuzyk PR, Dhotar HS, Sternheim A, Gross AE, Safir O, Backstein D. Two-stage revision arthroplasty for
management of chronic periprosthetic hip and knee infection: techniques, controversies, and outcomes.
J Am Acad Orthop Surg. 2014 Mar;22(3):153-64. doi: 10.5435/JAAOS-22-03-153. Review. PubMed
PMID: 24603825.

© 2015 American Academy of Orthopaedic Surgeons 2015 Orthopaedic In-Training Examination


126 • American Academy of Orthopaedic Surgeons

Question 121
Following knee arthroplasty, a patient has a painful catching sensation with knee extension. A clunk is
reproducible on examination. Which factor may increase the incidence of this complication?

1. Varus alignment
2. Smaller patella component size
3. Polyethylene thickness
4. Larger femoral component size
5. Cruciate-retaining knee arthroplasty

PREFERRED RESPONSE: 2

RECOMMENDED READINGS
Costanzo JA, Aynardi MC, Peters JD, Kopolovich DM, Purtill JJ. Patellar clunk syndrome after total knee
arthroplasty; risk factors and functional outcomes of arthroscopic treatment. J Arthroplasty. 2014 Sep;29(9
Suppl):201-4. doi: 10.1016/j.arth.2014.03.045. Epub 2014 May 24. PubMed PMID: 25034884.

Dennis DA, Kim RH, Johnson DR, Springer BD, Fehring TK, Sharma A. The John Insall Award: control-
matched evaluation of painful patellar Crepitus after total knee arthroplasty. Clin Orthop Relat Res. 2011
Jan;469(1):10-7. doi: 10.1007/s11999-010-1485-3. PubMed PMID: 20706813; PubMed Central PMCID:
PMC3008897.

Snir N, Schwarzkopf R, Diskin B, Takemoto R, Hamula M, Meere PA. Incidence of patellar clunk
syndrome in fixed versus high-flex mobile bearing posterior-stabilized total knee arthroplasty. J
Arthroplasty. 2014 Oct;29(10):2021-4. doi: 10.1016/j.arth.2014.05.011. Epub 2014 May 24. PubMed
PMID: 24961894.

© 2015 American Academy of Orthopaedic Surgeons 2015 Orthopaedic In-Training Examination


SECTION 4: Hip and Knee • 127

Question 136
What is the approximate normal ratio of the length of the patellar tendon to the height of the patella
(distance from the superior to inferior poles)?

1. 0.75:1
2. 1:1
3. 1.75:1
4. 2:1
5. 3:1

PREFERRED RESPONSE: 2

RECOMMENDED READINGS
Insall J, Salvati E. Patella position in the normal knee joint. Radiology. 1971 Oct;101(1):101-4. PubMed
PMID: 5111961.

Shabshin N, Schweitzer ME, Morrison WB, Parker L. MRI criteria for patella alta and baja. Skeletal
Radiol. 2004 Aug;33(8):445-50. Epub 2004 Jun 24. PubMed PMID: 15221214.

Aglietti P, Insall JN, Cerulli G. Patellar pain and incongruence. I: Measurements of incongruence. Clin
Orthop Relat Res. 1983 Jun;(176):217-24. PubMed PMID: 6851329.

Question 144
Which medium of polyethylene sterilization by gamma radiation is associated with distinctively rapid in
vivo wear?

1. Argon
2. Plasma
3. Oxygen
4. Nitrogen
5. Ethylene oxide

PREFERRED RESPONSE: 3

RECOMMENDED READINGS
Collier MB, Engh CA Jr, McAuley JP, Engh GA. Factors associated with the loss of thickness of
polyethylene tibial bearings after knee arthroplasty. J Bone Joint Surg Am. 2007 Jun;89(6):1306-14.
PubMed PMID: 17545435.

McGovern TF, Ammeen DJ, Collier JP, Currier BH, Engh GA. Rapid polyethylene failure of unicondylar
tibial components sterilized with gamma irradiation in air and implanted after a long shelf life. J Bone
Joint Surg Am. 2002 Jun;84-A(6):901-6. PubMed PMID: 12063322.

© 2015 American Academy of Orthopaedic Surgeons 2015 Orthopaedic In-Training Examination


128 • American Academy of Orthopaedic Surgeons

Video 154 (scan to view)

Question 154
What is the function of the structure visualized during the knee arthroscopy seen in Video 154?

1. Lateral stability in flexion


2. Lateral stability in extension
3. Medial stability in flexion
4. Medial stability in extension
5. Posterior stability

PREFERRED RESPONSE: 1

RECOMMENDED READINGS
LaPrade MD, Kennedy MI, Wijdicks CA, LaPrade RF. Anatomy and biomechanics of the medial side
of the knee and their surgical implications. Sports Med Arthrosc. 2015 Jun;23(2):63-70. doi: 10.1097/
JSA.0000000000000054. PubMed PMID: 25932874.

James EW, LaPrade CM, LaPrade RF. Anatomy and biomechanics of the lateral side of the knee and
surgical implications. Sports Med Arthrosc. 2015 Mar;23(1):2-9. doi: 10.1097/JSA.0000000000000040.
Review. PubMed PMID: 25545644.

© 2015 American Academy of Orthopaedic Surgeons 2015 Orthopaedic In-Training Examination


SECTION 4: Hip and Knee • 129

Question 165
Polyethylene irradiation during the sterilization process produces

1. longer shelf life.


2. a smoother surface.
3. decreased free radicals.
4. decreased cross-linking.
5. improved wear performance.

PREFERRED RESPONSE: 5

RECOMMENDED READINGS
Crowninshield RD, Muratoglu OK; Implant Wear Symposium 2007 Engineering Work Group. How have
new sterilization techniques and new forms of polyethylene influenced wear in total joint replacement? J
Am Acad Orthop Surg. 2008;16 Suppl 1:S80-5. PubMed PMID 18612021.

Faris PM, Ritter MA, Pierce AL, Davis KE, Faris GW. Polyethylene sterilization and production affects
wear in total hip arthroplasties. Clin Orthop Relat Res. 2006 Dec;453:305-8. PubMed PMID: 16906113.

© 2015 American Academy of Orthopaedic Surgeons 2015 Orthopaedic In-Training Examination


130 • American Academy of Orthopaedic Surgeons

Figure 177a Figure 177b

Question 177
Figures 177a and 177b are the radiographs of a patient who underwent a total knee arthroplasty (TKA) 1
year ago and now has persistent pain. What is the most appropriate next step?

1. Bone scan
2. Observation
3. CT scan
4. Revision knee arthroplasty
5. Complete blood count (CBC), erythrocyte sedimentation rate (ESR), and C-reactive
protein (CRP)

PREFERRED RESPONSE: 5

RECOMMENDED READINGS
Parvizi J, Ghanem E, Sharkey P, Aggarwal A, Burnett RS, Barrack RL. Diagnosis of infected total
knee: findings of a multicenter database. Clin Orthop Relat Res. 2008 Nov;466(11):2628-33. doi:
10.1007/s11999-008-0471-5. Epub 2008 Sep 10. PubMed PMID: 18781372; PubMed Central PMCID:
PMC2565043.

Rosenthall L, Lepanto L, Raymond F. Radiophosphate uptake in asymptomatic knee arthroplasty. J Nucl


Med. 1987 Oct;28(10):1546-9. PubMed PMID: 3655908.

© 2015 American Academy of Orthopaedic Surgeons 2015 Orthopaedic In-Training Examination


SECTION 4: Hip and Knee • 131

Question 191
Infection after total joint arthroplasty may be reduced with use of which protocol?

1. Staples for skin closure


2. Body-exhaust suits in the operating room
3. Removing the drain before discontinuing antibiotics
4. Minimizing use of allogenous blood transfusions
5. Universal presurgical screening for urinary tract infection

PREFERRED RESPONSE: 4

RECOMMENDED READINGS
Friedman R, Homering M, Holberg G, Berkowitz SD. Allogeneic blood transfusions and postoperative
infections after total hip or knee arthroplasty. J Bone Joint Surg Am. 2014 Feb 19;96(4):272-8. doi:
10.2106/JBJS.L.01268. PubMed PMID: 24553882.

Gómez-Lesmes SP, Tornero E, Martínez-Pastor JC, Pereira A, Marcos M, Soriano A. Length of storage
of transfused red blood cells and risk of prosthetic joint infection after primary knee arthroplasty. J
Arthroplasty. 2014 Oct;29(10):2016-20. doi: 10.1016/j.arth.2014.06.005. Epub 2014 Jun 9. PubMed
PMID: 25015758.

© 2015 American Academy of Orthopaedic Surgeons 2015 Orthopaedic In-Training Examination


132 • American Academy of Orthopaedic Surgeons

Figure 197a Figure 197b

Question 197
Figures 197a and 197b are the radiographs of a 26-year-old woman who has anterior and lateral hip pain
with all activities. Nonsurgical therapy has been exhausted. Based on these radiographic findings, what is
the preferred treatment at this time?

1. Hip resurfacing
2. Hip arthroscopy
3. Total hip arthroplasty (THA)
4. Periacetabular osteotomy
5. Proximal femoral osteotomy

PREFERRED RESPONSE: 4

RECOMMENDED READINGS
Ganz R, Klaue K, Vinh TS, Mast JW. A new periacetabular osteotomy for the treatment of hip dysplasias.
Technique and preliminary results. Clin Orthop Relat Res. 1988 Jul;(232):26-36. PubMed PMID:
3383491.

Parvizi J, Bican O, Bender B, Mortazavi SM, Purtill JJ, Erickson J, Peters C. Arthroscopy for labral tears
in patients with developmental dysplasia of the hip: a cautionary note. J Arthroplasty. 2009 Sep;24(6
Suppl):110-3. doi: 10.1016/j.arth.2009.05.021. PubMed PMID: 19596542.

Steppacher SD, Tannast M, Ganz R, Siebenrock KA. Mean 20-year followup of Bernese periacetabular
osteotomy. Clin Orthop Relat Res. 2008 Jul;466(7):1633-44. doi: 10.1007/s11999-008-0242-3. Epub 2008
May 1. PubMed PMID: 18449617; PubMed Central PMCID: PMC2505253. 

© 2015 American Academy of Orthopaedic Surgeons 2015 Orthopaedic In-Training Examination


SECTION 4: Hip and Knee • 133

Figure 206a Figure 206b

Question 206
Figures 206a and 206b are the radiographs of a 65-year-old man who underwent a left total hip
arthroplasty (THA). He has developed recurrent posterior instability. What is the most likely reason for his
recurrent dislocations?

1. Male gender
2. Failed capsular repair
3. Acetabular cup position
4. Insufficient femoral offset
5. Version of the femoral component

PREFERRED RESPONSE: 2

RECOMMENDED READINGS
Iorio R, Specht LM, Healy WL, Tilzey JF, Presutti AH. The effect of EPSTR and minimal incision surgery
on dislocation after THA. Clin Orthop Relat Res. 2006 Jun;447:39-42. PubMed PMID: 16741472.

Pellicci PM, Potter HG, Foo LF, Boettner F. MRI shows biologic restoration of posterior soft tissue repairs
after THA. Clin Orthop Relat Res. 2009 Apr;467(4):940-5. doi: 10.1007/s11999-008-0503-1. Epub 2008
Sep 24. PubMed PMID: 18813893; PubMed Central PMCID: PMC2650041.

Tsai SJ, Wang CT, Jiang CC. The effect of posterior capsule repair upon post-operative hip dislocation
following primary total hip arthroplasty. BMC Musculoskelet Disord. 2008 Feb 29;9:29. doi:
10.1186/1471-2474-9-29. PubMed PMID: 18307820; PubMed Central PMCID: PMC2292160. 

© 2015 American Academy of Orthopaedic Surgeons 2015 Orthopaedic In-Training Examination


134 • American Academy of Orthopaedic Surgeons

Figure 238a Figure 238b

Question 238
Presurgical planning for a left total hip arthroplasty is shown in Figures 238a and 238b When placing the
acetabular component in the position shown in Figure 238a, how will the effect differ vs using the method
shown in Figure 238b?

1. Increased hip offset


2. Increased joint reactive forces
3. No change in hip offset
4. No change in joint reactive forces
5. Decreased joint reactive forces

PREFERRED RESPONSE: 5

RECOMMENDED READINGS
Della Valle AG, Padgett DE, Salvati EA. Preoperative planning for primary total hip arthroplasty. J Am
Acad Orthop Surg. 2005 Nov;13(7):455-62. Review. PubMed PMID: 16272270.

McAuley JP, Ridgeway SR. Preoperative planning to prevent dislocation of the hip. Orthop Clin North
Am. 2001 Oct;32(4):579-86, viii. PubMed PMID: 11689371. 

© 2015 American Academy of Orthopaedic Surgeons 2015 Orthopaedic In-Training Examination


SECTION 4: Hip and Knee • 135

Question 248
According to the AAOS guideline, Treatment of Osteoarthritis of the Knee, 2nd Edition, what is the
recommended intervention for treatment of symptomatic knee osteoarthritis?

1. Intra-articular corticosteroid injection to the knee


2. Intra-articular viscosupplementation injection to the knee
3. Arthroscopic lavage and meniscal debridement of the knee
4. Valgus-directed force brace (medial compartment unloader)
5. Self-management program for strengthening and aerobic exercises

PREFERRED RESPONSE: 5

RECOMMENDED READINGS
Arden NK, Reading IC, Jordan KM, Thomas L, Platten H, Hassan A, Ledingham J. A randomised
controlled trial of tidal irrigation vs corticosteroid injection in knee osteoarthritis: the KIVIS Study.
Osteoarthritis Cartilage. 2008 Jun;16(6):733-9. Epub 2008 Feb 20. PubMed PMID: 18077189.

Coleman S, Briffa NK, Carroll G, Inderjeeth C, Cook N, McQuade J. A randomized controlled trial of a
self-management education program for osteoarthritis of the knee delivered by health care professionals.
Arthritis Res Ther. 2012 Jan 27;14(1):R21. doi: 10.1186/ar3703. PubMed PMID: 22284848; PubMed
Central PMCID: PMC3392814.

Kirkley A, Webster-Bogaert S, Litchfield R, Amendola A, MacDonald S, McCalden R, Fowler P. The


effect of bracing on varus gonarthrosis. J Bone Joint Surg Am. 1999 Apr;81(4):539-48. PubMed PMID:
10225800.

American Academy of Orthopaedic Surgeons: Treatment of Osteoarthritis of the Knee. Rosemont,


IL: American Academy of Orthopaedic Surgeons, 2013. Available at: http://www.aaos.org/research/
guidelines/guide.asp Accessed October 30, 2015.

© 2015 American Academy of Orthopaedic Surgeons 2015 Orthopaedic In-Training Examination


136 • American Academy of Orthopaedic Surgeons

Question 258
What factor is a contraindication to medial unicompartmental arthroplasty (UKA)?

1. Age older than 65


2. Rheumatoid arthritis
3. Osteochondral defect
4. Spontaneous osteonecrosis of the knee (SONK)
5. Previous anterior cruciate ligament reconstruction

PREFERRED RESPONSE: 2

RECOMMENDED READINGS
Glassman A, Lachiewicz P, Tanzer M, eds. Orthopaedic Knowledge Update: Hip and Knee
Reconstruction. Rosemont, IL: American Academy of Orthopaedic Surgeons; 2011.

Heyse TJ, Khefacha A, Fuchs-Winkelmann S, Cartier P. UKA after spontaneous osteonecrosis of the knee:
a retrospective analysis. Arch Orthop Trauma Surg. 2011 May;131(5):613-7. doi: 10.1007/s00402-010-
1177-1. Epub 2010 Aug 24. PubMed PMID: 20734200.

Question 264
What is the most common etiology of revision occurring within 2 years after total knee arthroplasty?

1. Infection
2. Loosening
3. Instability
4. Arthrofibrosis
5. Periprosthetic fracture

PREFERRED RESPONSE: 1

RECOMMENDED READINGS
Sharkey PF, Lichstein PM, Shen C, Tokarski AT, Parvizi J. Why are total knee arthroplasties failing
today--has anything changed after 10 years? J Arthroplasty. 2014 Sep;29(9):1774-8. doi: 10.1016/j.
arth.2013.07.024. Epub 2014 Jul 5. PubMed PMID: 25007726.

Bozic KJ, Kurtz SM, Lau E, Ong K, Chiu V, Vail TP, Rubash HE, Berry DJ. The epidemiology of
revision total knee arthroplasty in the United States. Clin Orthop Relat Res. 2010 Jan;468(1):45-51. doi:
10.1007/s11999-009-0945-0. Epub 2009 Jun 25. PubMed PMID: 19554385; PubMed Central PMCID:
PMC2795838.

© 2015 American Academy of Orthopaedic Surgeons 2015 Orthopaedic In-Training Examination


SECTION 5: Adult Spine • 137

SECTION 5: Adult Spine

Figure 7

Question 7
Figure 7 is the lumbar spine lateral radiograph of a 33-year-old woman who experienced worsening back
pain 4 months after undergoing lumbar surgery. What is the most appropriate treatment option?

1. Laminectomy
2. Facet injection
3. Posterior fusion
4. Revision arthroplasty
5. Immobilization in a rigid orthosis

PREFERRED RESPONSE: 3

RECOMMENDED READINGS
Harrison WD, Harrison DJ. The use of percutaneous lumbar fixation screws for bilateral pedicle fractures
with an associated dislocation of a lumbar disc prosthesis. Case Rep Orthop. 2013;2013:676017.
doi: 10.1155/2013/676017. Epub 2013 Nov 4. PubMed PMID: 24294533; PubMed Central PMCID:
PMC3835198.

Patel AA, Brodke DS, Pimenta L, Bono CM, Hilibrand AS, Harrop JS, Riew KD, Youssef JA,
Vaccaro AR. Revision strategies in lumbar total disc arthroplasty. Spine (Phila Pa 1976). 2008 May
15;33(11):1276-83. doi: 10.1097/BRS.0b013e3181714a1d. Review. PubMed PMID: 18469704. 

© 2015 American Academy of Orthopaedic Surgeons 2015 Orthopaedic In-Training Examination


138 • American Academy of Orthopaedic Surgeons

Figure 19

Question 19
Figure 19 is the lateral radiograph of the lumbosacral junction and pelvis of a 67-year-old woman with a
lumbar deformity who is being evaluated for surgery. Angle “x” refers to which radiographic parameter?

1. Plumb line
2. Sacral slope
3. Lumbar lordosis
4. Pelvic incidence
5. Pelvic tilt

PREFERRED RESPONSE: 4

RECOMMENDED READINGS
Labelle H, Mac-Thiong JM, Roussouly P. Spino-pelvic sagittal balance of spondylolisthesis: a review and
classification. Eur Spine J. 2011 Sep;20 Suppl 5:641-6. doi: 10.1007/s00586-011-1932-1. Epub 2011 Aug
2. Review. PubMed PMID: 21809015; PubMed Central PMCID: PMC3175928.

Mehta VA, Amin A, Omeis I, Gokaslan ZL, Gottfried ON. Implications of spinopelvic alignment for the
spine surgeon. Neurosurgery. 2012 Mar;70(3):707-21. doi: 10.1227/NEU.0b013e31823262ea. Review.
Erratum in: Neurosurgery. 2012 May;70(5):1324. PubMed PMID: 21937939. 

© 2015 American Academy of Orthopaedic Surgeons 2015 Orthopaedic In-Training Examination


SECTION 5: Adult Spine • 139

Figure 27a Figure 27b

Question 27
Figures 27a and 27b are the sagittal CT and T2-weighted MR images of the cervical spine of a 71-year-old
man who has neck pain after falling from a standing height. What is the best next step?

1. Halo orthosis
2. Physical therapy
3. Soft cervical collar
4. Anterior cervical fusion
5. Posterior cervical fusion

PREFERRED RESPONSE: 5

RECOMMENDED READINGS
Mathews M, Bolesta MJ. Treatment of spinal fractures in ankylosing spondylitis. Orthopedics. 2013
Sep;36(9):e1203-8. doi:10.3928/01477447-20130821-25. PubMed PMID: 24025014.

Whang PG, Goldberg G, Lawrence JP, Hong J, Harrop JS, Anderson DG, Albert TJ, Vaccaro AR. The
management of spinal injuries in patients with ankylosing spondylitis or diffuse idiopathic skeletal
hyperostosis: a comparison of treatment methods and clinical outcomes. J Spinal Disord Tech. 2009
Apr;22(2):77-85. doi: 10.1097/BSD.0b013e3181679bcb. PubMed PMID: 19342927. 

© 2015 American Academy of Orthopaedic Surgeons 2015 Orthopaedic In-Training Examination


140 • American Academy of Orthopaedic Surgeons

Figure 35

Question 35
Figure 35 is the sagittal T2-weighted MR image of the lumbar spine of a 56-year-old man who has
been experiencing back pain for several years. Which treatment has been shown to give rise to clinical
outcomes that are equivalent to those observed following fusion surgery?

1. Gabapentinoid administration
2. Chiropractic care
3. Epidural injections
4. Cognitive-behavioral therapy
5. Intradiskal electrothermal annuloplasty

PREFERRED RESPONSE: 4

RECOMMENDED READINGS
Mannion AF, Brox JI, Fairbank JC. Comparison of spinal fusion and nonoperative treatment in patients
with chronic low back pain: long-term follow-up of three randomized controlled trials. Spine J. 2013
Nov;13(11):1438-48. doi: 10.1016/j.spinee.2013.06.101. Epub 2013 Nov 5. PubMed PMID: 24200413.

Brox JI, Sørensen R, Friis A, Nygaard Ø, Indahl A, Keller A, Ingebrigtsen T, Eriksen HR, Holm I,
Koller AK, Riise R, Reikerås O. Randomized clinical trial of lumbar instrumented fusion and cognitive
intervention and exercises in patients with chronic low back pain and disc degeneration. Spine (Phila Pa
1976). 2003 Sep 1;28(17):1913-21. PubMed PMID: 12973134. 

© 2015 American Academy of Orthopaedic Surgeons 2015 Orthopaedic In-Training Examination


SECTION 5: Adult Spine • 141

Figure 45

Question 45
Figure 45 is the postsurgical lateral radiograph of a 72-year-old woman who undergoes lumbar fusion for
L5 spondylolysis. Immediately after surgery, one of her legs feels colder. This condition is most likely
caused by an intraoperative injury to which structure?

1. Sympathetic trunk
2. Cauda equina
3. Iliac vein
4. S1 nerve root
5. Superior hypogastric plexus

PREFERRED RESPONSE: 1

RECOMMENDED READINGS
Kasliwal MK, Deutsch H. Anhidrosis after anterior retroperitoneal approach for L4-L5 artificial disc
replacement. J Clin Neurosci. 2011 Jul;18(7):990-1. doi: 10.1016/j.jocn.2010.11.022. Epub 2011 May 12.
PubMed PMID: 21570302.

Schulte TL, Adolphs B, Oberdiek D, Osada N, Liljenqvist U, Filler TJ, Marziniak M, Bullmann V.
Approach-related lesions of the sympathetic chain in anterior correction and instrumentation of idiopathic
scoliosis. Eur Spine J. 2010 Sep;19(9):1558-68. doi: 10.1007/s00586-010-1455-1. Epub 2010 May 26.
PubMed PMID: 20502925; PubMed Central PMCID: PMC2989285. 

© 2015 American Academy of Orthopaedic Surgeons 2015 Orthopaedic In-Training Examination


142 • American Academy of Orthopaedic Surgeons

Question 55
A 27-year-old man with paraplegia is prescribed gabapentin for his neuropathic pain after sustaining a
thoracic-level spinal cord injury. What is the mechanism of action of this medication?

1. Disrupts production of arachidonic acid by inhibiting phospholipase A2


2. Inhibits the receptor activator of nuclear factor kappa beta ligand
3. Impedes postsynaptic sodium channels and prevents membrane depolarization
4. Binds to presynaptic calcium channels and limits the release of neurotransmitters
5. Reduces uptake of serotonin and norepinephrine by blocking transporter proteins

PREFERRED RESPONSE: 4

RECOMMENDED READINGS
Bennett MI, Simpson KH. Gabapentin in the treatment of neuropathic pain. Palliat Med. 2004 Jan;18(1):5-
11. Review. PubMed PMID: 14982201.

Mehta S, McIntyre A, Dijkers M, Loh E, Teasell RW. Gabapentinoids are effective in decreasing
neuropathic pain and other secondary outcomes after spinal cord injury: a meta-analysis. Arch Phys Med
Rehabil. 2014 Nov;95(11):2180-6. doi: 10.1016/j.apmr.2014.06.010. Epub 2014 Jun 30. Review. PubMed
PMID: 24992021.

Guy S, Mehta S, Leff L, Teasell R, Loh E. Anticonvulsant medication use for the management of
pain following spinal cord injury: systematic review and effectiveness analysis. Spinal Cord. 2014
Feb;52(2):89-96. doi: 10.1038/sc.2013.146. Epub 2013 Dec 3. Review. PubMed PMID: 24296804.

Question 68
A patient is positioned prone for a T10-S1 instrumented decompression and fusion for treatment of
degenerative scoliosis. After surgery the patient has numbness and tingling in the ring and small fingers.
Weakness is found in the hand intrinsic muscles. What is the most likely location for a neurologic injury
associated with positioning?

1. Ulnar nerve
2. Radial nerve
3. Median nerve
4. Cervical nerve root
5. Brachial plexus

PREFERRED RESPONSE: 1

© 2015 American Academy of Orthopaedic Surgeons 2015 Orthopaedic In-Training Examination


SECTION 5: Adult Spine • 143

RECOMMENDED READINGS
Labrom RD, Hoskins M, Reilly CW, Tredwell SJ, Wong PK. Clinical usefulness of somatosensory evoked
potentials for detection of brachial plexopathy secondary to malpositioning in scoliosis surgery. Spine
(Phila Pa 1976). 2005 Sep 15;30(18):2089-93. PubMed PMID: 16166901.

Schwartz DM, Sestokas AK, Hilibrand AS, Vaccaro AR, Bose B, Li M, Albert TJ. Neurophysiological
identification of position-induced neurologic injury during anterior cervical spine surgery. J Clin Monit
Comput. 2006 Dec;20(6):437-44. Epub 2006 Sep 8. PubMed PMID: 16960753.

© 2015 American Academy of Orthopaedic Surgeons 2015 Orthopaedic In-Training Examination


144 • American Academy of Orthopaedic Surgeons

Figure 77

Question 77
Figure 77 is the lateral thoracic radiograph of an 86-year-old woman who has acute midback pain after
a slip and fall at home. She denies lower extremity pain and has normal motor and sensory examination
findings. What is the best next step?

1. Open biopsy
2. Vertebroplasty
3. Osteoporosis evaluation
4. Posterior thoracic arthrodesis
5. Anterior corpectomy and arthrodesis

PREFERRED RESPONSE: 3

RECOMMENDED READINGS
Buchbinder R, Osborne RH, Ebeling PR, Wark JD, Mitchell P, Wriedt C, Graves S, Staples MP, Murphy
B. A randomized trial of vertebroplasty for painful osteoporotic vertebral fractures. N Engl J Med. 2009
Aug 6;361(6):557-68. doi: 10.1056/NEJMoa0900429. PubMed PMID: 19657121.

U.S. Preventive Services Task Force. Screening for osteoporosis: U.S. preventive services task force
recommendation statement. Ann Intern Med. 2011 Mar 1;154(5):356-64. doi: 10.7326/0003-4819-154-5-
201103010-00307. Epub 2011 Jan 17. PubMed PMID: 21242341.

© 2015 American Academy of Orthopaedic Surgeons 2015 Orthopaedic In-Training Examination


SECTION 5: Adult Spine • 145

Figure 94a Figure 94b

Question 94
Figures 94a and 94b are the MR images of a 72-year-old woman who has pain radiating into both legs
with ambulation. Her pain is improved with bending forward. Her neurologic examination reveals normal
strength and sensation. She has attempted nonsurgical treatment for 3 months and has experienced
progressive worsening pain. What is the most effective treatment option?

1. Facet injections
2. L4-S1 anterior and posterior fusion
3. Epidural steroid injections
4. Posterior L4-L5 lumbar decompression
5. Posterior L4-L5 lumbar decompression and arthrodesis

PREFERRED RESPONSE: 5

RECOMMENDED READINGS
Weinstein JN, Lurie JD, Tosteson TD, Zhao W, Blood EA, Tosteson AN, Birkmeyer N, Herkowitz H,
Longley M, Lenke L, Emery S, Hu SS. Surgical compared with nonoperative treatment for lumbar
degenerative spondylolisthesis. four-year results in the Spine Patient Outcomes Research Trial (SPORT)
randomized and observational cohorts. J Bone Joint Surg Am. 2009 Jun;91(6):1295-304. doi: 10.2106/
JBJS.H.00913. PubMed PMID: 19487505.

Abdu WA, Lurie JD, Spratt KF, Tosteson AN, Zhao W, Tosteson TD, Herkowitz H, Longely M, Boden
SD, Emery S, Weinstein JN. Degenerative spondylolisthesis: does fusion method influence outcome?
Four-year results of the spine patient outcomes research trial. Spine (Phila Pa 1976). 2009 Oct
1;34(21):2351-60. doi: 10.1097/BRS.0b013e3181b8a829. PubMed PMID: 19755935

© 2015 American Academy of Orthopaedic Surgeons 2015 Orthopaedic In-Training Examination


146 • American Academy of Orthopaedic Surgeons

Figure 110

Question 110
Figure 110 is the MR image of a 54-year-old man who has had severe neck pain, upper extremity pain,
and weakness for 6 months. He has progressive gait instability, and examination reveals bilateral weakness
in his biceps, triceps, and hand intrinsics. He has 3+ biceps, triceps, and quadriceps reflexes with sustained
ankle clonus. What is the chief posterior decompression contraindication?

1. Age
2. Severe neck pain
3. Cervical kyphosis
4. Multiple-level diseases
5. Duration of symptoms

PREFERRED RESPONSE: 3

RECOMMENDED READINGS
Suda K, Abumi K, Ito M, Shono Y, Kaneda K, Fujiya M. Local kyphosis reduces surgical outcomes of
expansive open-door laminoplasty for cervical spondylotic myelopathy. Spine (Phila Pa 1976). 2003 Jun
15;28(12):1258-62. PubMed PMID: 12811268.

Chiba K, Ogawa Y, Ishii K, Takaishi H, Nakamura M, Maruiwa H, Matsumoto M, Toyama Y. Long-term


results of expansive open-door laminoplasty for cervical myelopathy--average 14-year follow-up study.
Spine (Phila Pa 1976). 2006 Dec 15;31(26):2998-3005. PubMed PMID: 17172996.

© 2015 American Academy of Orthopaedic Surgeons 2015 Orthopaedic In-Training Examination


SECTION 5: Adult Spine • 147

Figure 124

Question 124
Figure 124 is the lateral radiograph of a 63-year-old man who has had a long-standing history of chronic
neck and right upper extremity pain. He recently reported progressive weakness in his right shoulder
and biceps that developed over the course of 90 days. He has a history of hepatitis C attributable to
intravenous (IV) drug use in the remote past. He denies any history of spinal trauma, and his temperature
in the office is 37°C. What is the most appropriate next step?

1. Dual-energy x-ray absorptiometry (DEXA) scan


2. CT scan with contrast
3. MR imaging without contrast
4. MR imaging with and without contrast
5. Flexion extension plain film images

PREFERRED RESPONSE: 4

RECOMMENDED READINGS
Schoenfeld AJ. Spine infections. In: Cannada LK, ed. Orthopaedic Knowledge Update 11. Rosemont, IL:
American Academy of Orthopaedic Surgeons; 2014:737-747.

Schoenfeld AJ, Wahlquist TC. Mortality, complication risk, and total charges after the treatment of
epidural abscess. Spine J. 2015 Feb 1;15(2):249-55. doi: 10.1016/j.spinee.2014.09.003. Epub 2014 Sep
21. PubMed PMID: 25241303.

© 2015 American Academy of Orthopaedic Surgeons 2015 Orthopaedic In-Training Examination


148 • American Academy of Orthopaedic Surgeons

Figure 141

Question 141
Figure 141 is the axial MR image of a 73-year-old man with a history of hypertension. He has had
bilateral buttock pain when walking more than 4 blocks for 4 weeks. A complete diagnostic workup
should include

1. examination of the peripheral pulses.


2. CT myelogram.
3. peripheral muscle biopsy.
4. laboratory evaluation including prothrombin time and international normalized ratio.
5. electromyography and nerve conduction studies.

PREFERRED RESPONSE: 1

RECOMMENDED READINGS
Issack PS, Cunningham ME, Pumberger M, Hughes AP, Cammisa FP Jr. Degenerative lumbar spinal
stenosis: evaluation and management. J Am Acad Orthop Surg. 2012 Aug;20(8):527-35. doi: 10.5435/
JAAOS-20-08-527. Review. PubMed PMID: 22855855.

Young IA, Hyman GS, Packia-Raj LN, Cole AJ. The use of lumbar epidural/transforaminal steroids for
managing spinal disease. J Am Acad Orthop Surg. 2007 Apr;15(4):228-38. Review. PubMed PMID:
17426294.

© 2015 American Academy of Orthopaedic Surgeons 2015 Orthopaedic In-Training Examination


SECTION 5: Adult Spine • 149

Figure 163

Question 163
Figure 163 is the MR image of a 43-year-old woman who has a 3-week history of neck pain radiating into
her left arm. She denies numbness or problems with balance. Examination reveals her pain is reproduced
with ipsilateral neck rotation. What is the best next step?

1. Radiofrequency ablation
2. Chiropractic manipulation
3. Posterior cervical foraminotomy
4. Anterior cervical diskectomy and fusion
5. Initiation of nonsteroidal anti-inflammatory drugs (NSAIDs)

PREFERRED RESPONSE: 5

RECOMMENDED READINGS
Rhee JM, Yoon T, Riew KD. Cervical radiculopathy. J Am Acad Orthop Surg. 2007 Aug;15(8):486-94.
Review. PubMed PMID: 17664368.

Dreyer SJ, Boden SD. Nonoperative treatment of neck and arm pain. Spine (Phila Pa 1976). 1998 Dec
15;23(24):2746-54. Review. PubMed PMID: 9879100.

© 2015 American Academy of Orthopaedic Surgeons 2015 Orthopaedic In-Training Examination


150 • American Academy of Orthopaedic Surgeons

Figure 180

Question 180
Figure 180 is the MR image of a 68-year-old woman who has experienced neck pain for several years.
Recently she began to notice gait imbalance and handwriting changes. What is the most appropriate next
step?

1. Cervical traction
2. Epidural steroid injections
3. Single-level anterior diskectomy and fusion
4. Multilevel posterior decompression and fusion
5. Evaluation by a neurologist for demyelinating disease

PREFERRED RESPONSE: 4

RECOMMENDED READINGS
Emery SE. Cervical spondylotic myelopathy: diagnosis and treatment. J Am Acad Orthop Surg. 2001
Nov-Dec;9(6):376-88. Review. PubMed PMID: 11767723.

Gerard CS, O’Toole JE. Current techniques in the management of cervical myelopathy and radiculopathy.
Neurosurg Clin N Am. 2014 Apr;25(2):261-70. doi: 10.1016/j.nec.2013.12.005. Epub 2014 Jan 31.
Review. PubMed PMID: 24703445. 

© 2015 American Academy of Orthopaedic Surgeons 2015 Orthopaedic In-Training Examination


SECTION 5: Adult Spine • 151

Figure 194

Question 194
Figure 194 is the lateral weight-bearing radiograph of a 72-year-old woman who has a 2-year history of
lower back and bilateral lower extremity pain with prolonged walking. Which surgical treatment will
provide the best long-term results?

1. Laminectomy
2. Laminectomy with instrumented fusion
3. Lumbar microdiskectomy
4. Interspinous process spacer
5. Anterior lumbar interbody fusion

PREFERRED RESPONSE: 2

RECOMMENDED READINGS
Majid K, Fischgrund JS. Degenerative lumbar spondylolisthesis: trends in management. J Am Acad
Orthop Surg. 2008 Apr;16(4):208-15. Review. PubMed PMID: 18390483.

Fischgrund JS, Mackay M, Herkowitz HN, Brower R, Montgomery DM, Kurz LT. 1997 Volvo Award
winner in clinical studies. Degenerative lumbar spondylolisthesis with spinal stenosis: a prospective,
randomized study comparing decompressive laminectomy and arthrodesis with and without spinal
instrumentation. Spine (Phila Pa 1976). 1997 Dec 15;22(24):2807-12. PubMed PMID: 9431616. 

© 2015 American Academy of Orthopaedic Surgeons 2015 Orthopaedic In-Training Examination


152 • American Academy of Orthopaedic Surgeons

Question 199
The mechanism of action of etanercept in the treatment of rheumatoid arthritis is based on its ability to act
as an

1. antagonist of tumor necrosis factor-alpha.


2. agonist of Interleukin-1 (IL-1).
3. antagonist of IL-1.
4. agonist of pyrimidine synthesis.
5. antagonist of cyclooxygenase-1.

PREFERRED RESPONSE: 1

RECOMMENDED READINGS
Howe CR, Gardner GC, Kadel NJ. Perioperative medication management for the patient with rheumatoid
arthritis. J Am Acad Orthop Surg. 2006 Sep;14(9):544-51. Review. PubMed PMID: 16959892.

Smolen JS, Nash P, Durez P, Hall S, Ilivanova E, Irazoque-Palazuelos F, Miranda P, Park MC, Pavelka K,
Pedersen R, Szumski A, Hammond C, Koenig AS, Vlahos B. Maintenance, reduction, or withdrawal of
etanercept after treatment with etanercept and methotrexate in patients with moderate rheumatoid arthritis
(PRESERVE): a randomised controlled trial. Lancet. 2013 Mar 16;381(9870):918-29. doi: 10.1016/
S0140-6736(12)61811-X. Epub 2013 Jan 17. PubMed PMID: 23332236. 

© 2015 American Academy of Orthopaedic Surgeons 2015 Orthopaedic In-Training Examination


SECTION 5: Adult Spine • 153

Figure 205

Question 205
The arrow in Figure 205 points to which structure?

1. Left L3 nerve root


2. Left L4 nerve root
3. Right L3 nerve root
4. Right L4 nerve root
5. Right segmental vessel

PREFERRED RESPONSE: 3

RECOMMENDED READINGS
An H. Diagnostic imaging of the spine. In: An H, ed. Principles and Techniques of Spine Surgery.
Baltimore, MD: Williams & Wilkins; 1998:102-125.

Bono CM, Wisneski R, Garfin SR. Lumbar disc herniations. In: Herkowitz HN, Garfin SR, Eismont FJ,
Bell GR, Balderston RA, eds. Rothman-Simeone’s The Spine, Vol 1. 5th ed. Philadelphia, PA: Saunders
Elsevier; 2006:967-991. 

© 2015 American Academy of Orthopaedic Surgeons 2015 Orthopaedic In-Training Examination


154 • American Academy of Orthopaedic Surgeons

Question 214
A 72-year-old woman recently started taking denosumab after sustaining a second osteoporotic vertebral
body compression fracture. What is the mechanism of action of this medication?

1. Increases serum calcium levels by activating vitamin D


2. Impairs the formation of the ruffled border of osteoclasts
3. Functions as a selective estrogen receptor modulator
4. Stimulates osteoblastic differentiation of pluripotential stem cells
5. Inhibits the receptor activator of nuclear factor kappa beta ligand (RANKL)

PREFERRED RESPONSE: 2

RECOMMENDED READINGS
Väänänen HK, Zhao H, Mulari M, Halleen JM. The cell biology of osteoclast
function. J Cell Sci. 2000 Feb;113 ( Pt 3):377-81. Review. PubMed PMID: 10639325.

Boyce BF. Advances in the regulation of osteoclasts and osteoclast functions. J Dent Res. 2013
Oct;92(10):860-7. doi: 10.1177/0022034513500306. Epub 2013 Aug 1. Review. PubMed PMID:
23906603; PubMed Central PMCID: PMC3775372.

Warriner AH, Saag KG. Osteoporosis diagnosis and medical treatment. Orthop Clin North Am. 2013
Apr;44(2):125-35. doi: 10.1016/j.ocl.2013.01.005. Epub 2013 Feb 26. Review. PubMed PMID: 23544819.

Cummings SR, San Martin J, McClung MR, Siris ES, Eastell R, Reid IR, Delmas P, Zoog HB, Austin
M, Wang A, Kutilek S, Adami S, Zanchetta J, Libanati C, Siddhanti S, Christiansen C; FREEDOM Trial.
Denosumab for prevention of fractures in postmenopausal women with osteoporosis. N Engl J Med. 2009
Aug 20;361(8):756-65. doi: 10.1056/NEJMoa0809493. Epub 2009 Aug 11. Erratum in: N Engl J Med.
2009 Nov 5;361(19):1914. PubMed PMID: 19671655. 

© 2015 American Academy of Orthopaedic Surgeons 2015 Orthopaedic In-Training Examination


SECTION 5: Adult Spine • 155

Question 220
An 83-year-old man has had an L1 osteoporotic burst fracture with progressive lower extremity weakness
for 4 days. He can no longer walk because of back and leg pain and leg weakness. Examination reveals
bilateral weakness in his hip flexors and quadriceps. MR imaging reveals severe canal stenosis and
compression on the conus medullaris and cauda equina. What is the best treatment option?

1. Kyphoplasty
2. Vertebroplasty
3. Laminectomy
4. Laminectomy and fusion
5. Thoracic lumbar sacral orthosis

PREFERRED RESPONSE: 4

RECOMMENDED READINGS
Ughwanogho E, Hu X, Lieberman IH. Vertebral compression fractures. In: Cannada LK, ed. Orthopaedic
Knowledge Update 11. Rosemont, IL: American Academy of Orthopaedic Surgeons; 2014:775-782.

Patel AA, Vaccaro AR. Thoracolumbar spine trauma classification. J Am Acad Orthop Surg. 2010
Feb;18(2):63-71. PubMed PMID: 20118323.

Esses SI, McGuire R, Jenkins J, Finkelstein J, Woodard E, Watters WC 3rd, Goldberg MJ, Keith M,
Turkelson CM, Wies JL, Sluka P, Boyer KM, Hitchcock K. The treatment of symptomatic osteoporotic
spinal compression fractures. J Am Acad Orthop Surg. 2011 Mar;19(3):176-82. PubMed PMID:
21368099.

© 2015 American Academy of Orthopaedic Surgeons 2015 Orthopaedic In-Training Examination


156 • American Academy of Orthopaedic Surgeons

Question 225
A 90-year-old woman has neck pain and an incomplete spinal cord injury associated with a cervical
spine fracture after sustaining a ground-level fall. This patient’s age puts her at heightened risk for which
complication?

1. Mortality
2. Chronic pain
3. Wound infection
4. Pulmonary embolism
5. Intracranial hemorrhage

PREFERRED RESPONSE: 1

RECOMMENDED READINGS
Fassett DR, Harrop JS, Maltenfort M, Jeyamohan SB, Ratliff JD, Anderson DG, Hilibrand AS, Albert TJ,
Vaccaro AR, Sharan AD. Mortality rates in geriatric patients with spinal cord injuries. J Neurosurg Spine.
2007 Sep;7(3):277-81. PubMed PMID: 17877260.

Irwin ZN, Arthur M, Mullins RJ, Hart RA. Variations in injury patterns, treatment, and outcome for spinal
fracture and paralysis in adult versus geriatric patients. Spine (Phila Pa 1976). 2004 Apr 1;29(7):796-802.
PubMed PMID: 15087803.

Question 233
When performing lateral transpsoas interbody fusion at L4-5, which nerve(s) may be injured when
dissecting through the abdominal wall?

1. Sciatic nerve
2. L4 and L5 nerve roots
3. Ilioinguinal and iliohypogastric
4. Lateral femoral cutaneous and obturator
5. Lateral femoral cutaneous and saphenous

PREFERRED RESPONSE: 3

RECOMMENDED READINGS
Ahmadian A, Deukmedjian AR, Abel N, Dakwar E, Uribe JS. Analysis of lumbar plexopathies and nerve
injury after lateral retroperitoneal transpsoas approach: diagnostic standardization. J Neurosurg Spine.
2013 Mar;18(3):289-97. doi: 10.3171/2012.11.SPINE12755. Epub 2012 Dec 21. Review. PubMed PMID:
23259543.

Uribe JS, Vale FL, Dakwar E. Electromyographic monitoring and its anatomical implications in minimally
invasive spine surgery. Spine (Phila Pa 1976). 2010 Doi: 15;35(26 Suppl):S368-74. doi: 10.1097/
BRS.0b013e3182027976. Review. PubMed PMID: 21160402.

© 2015 American Academy of Orthopaedic Surgeons 2015 Orthopaedic In-Training Examination
SECTION 5: Adult Spine • 157

Figure 243

Question 243
Figure 243 is the lateral radiograph of a 32-year-old basketball player who has had low-back and leg
pain for 4 years. An examination reveals exacerbation of back pain with extension. She has normal motor
examination findings and normal sensation. Which pelvic parameter correlates with this condition?

1. Pelvic tilt
2. Pelvic incidence
3. Sacral slope
4. Slip angle
5. Spinopelvic angle

PREFERRED RESPONSE: 2

RECOMMENDED READINGS
Labelle H, Roussouly P, Berthonnaud E, Dimnet J, O’Brien M. The importance of spino-pelvic balance in
L5-s1 developmental spondylolisthesis: a review of pertinent radiologic measurements. Spine (Phila Pa
1976). 2005 Mar 15;30(6 Suppl):S27-34. Review. PubMed PMID: 15767882.

Hanson DS, Bridwell KH, Rhee JM, Lenke LG. Correlation of pelvic incidence with low- and high-grade
isthmic spondylolisthesis. Spine (Phila Pa 1976). 2002 Sep 15;27(18):2026-9. PubMed PMID: 12634563. 

© 2015 American Academy of Orthopaedic Surgeons 2015 Orthopaedic In-Training Examination


158 • American Academy of Orthopaedic Surgeons

Figure 249a Figure 249b

Question 249
Figures 249a and 249b are the cervical angiogram images of a 56-year-old woman with a cervical spine
fracture resulting from a motor vehicle collision. Which fracture pattern is most commonly associated
with this type of injury?
1. C5 facet with subluxation
2. C3 transverse process
3. Type 2 odontoid
4. Hangman
5. Jefferson
PREFERRED RESPONSE: 1
RECOMMENDED READINGS
Lebl DR, Bono CM, Velmahos G, Metkar U, Nguyen J, Harris MB. Vertebral artery injury associated
with blunt cervical spine trauma: a multivariate regression analysis. Spine (Phila Pa 1976). 2013 Jul
15;38(16):1352-61. doi: 10.1097/BRS.0b013e318294bacb. PubMed PMID: 23574813.
Dreger T, Place H, Piper C, Mattingly T, Brechbuehler J. Analysis of Cervical Angiograms in Cervical
Spine Trauma Patients, Does it Make a Difference? J Spinal Disord Tech. 2014 Aug 1. [Epub ahead of
print] PubMed PMID: 25089671.

© 2015 American Academy of Orthopaedic Surgeons 2015 Orthopaedic In-Training Examination


SECTION 6: Foot and Ankle • 159

SECTION 6: Foot and Ankle

Figure 1a Figure 1b

Question 1
Figures 1a and 1b are the radiographs of a 28-year-old woman who has frequent, painless giving-way of
her ankle, which is causing her difficulty at work. Examination reveals severe laxity of both the anterior
talofibular ligament (ATFL) and calcaneofibular ligament (CFL). Her hindfoot alignment does not change
with Coleman block testing. Surgical treatment should include reconstruction of the ATFL, CFL, and

1. triple arthrodesis.
2. in situ subtalar arthrodesis.
3. realignment subtalar arthrodesis.
4. dorsiflexion osteotomy of the first metatarsal.
5. lateralizing osteotomy of the calcaneus tuberosity.

PREFERRED RESPONSE: 5

RECOMMENDED READINGS
Strauss JE, Forsberg JA, Lippert FG 3rd. Chronic lateral ankle instability and associated conditions: a
rationale for treatment. Foot Ankle Int. 2007 Oct;28(10): 1041-4. PubMed PMID: 17923051.

Vienne P, Schцniger R, Helmy N, Espinosa N. Hindfoot instability in cavovarus deformity: static and
dynamic balancing. Foot Ankle Int. 2007 Jan;28(1):96-102. PubMed PMID: 17257547.

© 2015 American Academy of Orthopaedic Surgeons 2015 Orthopaedic In-Training Examination


160 • American Academy of Orthopaedic Surgeons

Figure 16a Figure 16b Figure 16c

Question 16
Figures 16a through 16c are the weight-bearing radiographs of an active 34-year-old man with chronic
great toe pain who is 6-ft, 5-in tall and weighs 250 pounds. He underwent surgery 8 months ago and
reports persistent stiffness and pain with weight-bearing activity despite shoe modifications and a steel
shank insert. He denies fevers, chills, and a history of prior wound difficulties. In addition to component
explantation, what is the most appropriate treatment method?

1. Resection arthroplasty
2. Capsular interposition and phalanx osteotomy
3. Arthrodesis with structural bone graft
4. Conversion to total joint arthroplasty
5. In situ arthrodesis

PREFERRED RESPONSE: 3

RECOMMENDED READINGS
Garras DN, Durinka JB, Bercik M, Miller AG, Raikin SM. Conversion arthrodesis for failed
first metatarsophalangeal joint hemiarthroplasty. Foot Ankle Int. 2013 Sep;34(9):1227-32. doi:
10.1177/1071100713488093. Epub 2013 Apr 23. PubMed PMID: 23613329.

Brodsky JW, Ptaszek AJ, Morris SG. Salvage first MTP arthrodesis utilizing ICBG: clinical evaluation and
outcome. Foot Ankle Int. 2000 Apr;21(4):290-6. PubMed PMID: 10808968. 

© 2015 American Academy of Orthopaedic Surgeons 2015 Orthopaedic In-Training Examination


SECTION 6: Foot and Ankle • 161

Figure 31a Figure 31b Figure 31c

Question 31
Figures 31a through 31c are the MR images of an abnormal tendon that is most important during which
stage of the gait cycle?

1. Toe-off
2. Terminal swing
3. Heel strike
4. Midstance
5. Terminal stance

PREFERRED RESPONSE: 5

RECOMMENDED READINGS
Kaye RA, Jahss MH. Tibialis posterior: a review of anatomy and biomechanics in relation to support of
the medial longitudinal arch. Foot Ankle. 1991 Feb;11(4):244-7. Review. PubMed PMID: 1855713.

Semple R, Murley GS, Woodburn J, Turner DE. Tibialis posterior in health and disease: a review of
structure and function with specific reference to electromyographic studies. J Foot Ankle Res. 2009
Aug 19;2:24. doi: 10.1186/1757-1146-2-24. PubMed PMID: 19691828; PubMed Central PMCID:
PMC2739849.

© 2015 American Academy of Orthopaedic Surgeons 2015 Orthopaedic In-Training Examination


162 • American Academy of Orthopaedic Surgeons

Figure 46a Figure 46b

Question 46
Figures 46a and 46b are the current radiographs of a 42-year-old man who has severe left ankle and
hindfoot pain after sustaining an open ankle fracture 3 years ago. The pain is aggravated by weight bearing
and relieved by rest. He denies paresthesia and has had 7 surgeries on the ankle during the last 3 years,
including a tibiotalocalcaneal arthrodesis 15 months ago. What is the best next step?

1. Observation
2. Scar revision
3. Nerve exploration
4. Revision arthrodesis
5. Referral to a pain management clinic

PREFERRED RESPONSE: 4

RECOMMENDED READINGS
Hammett R, Hepple S, Forster B, Winson I. Tibiotalocalcaneal (hindfoot) arthrodesis by retrograde
intramedullary nailing using a curved locking nail. The results of 52 procedures. Foot Ankle Int. 2005
Oct;26(10):810-5. PubMed PMID: 16221452.

Chou LB, Mann RA, Yaszay B, Graves SC, McPeake WT 3rd, Dreeben SM, Horton GA, Katcherian
DA, Clanton TO, Miller RA, Van Manen JW. Tibiotalocalcaneal arthrodesis. Foot Ankle Int. 2000
Oct;21(10):804-8. PubMed PMID: 11128009. 

© 2015 American Academy of Orthopaedic Surgeons 2015 Orthopaedic In-Training Examination


SECTION 6: Foot and Ankle • 163

Question 54
A 63-year-old man with a 20-year history of noninsulin-dependent diabetes mellitus has a 6-month history
of a Wagner grade 1 plantar foot ulcer at the hallux interphalangeal (IP) joint. There is no erythema or
drainage. Pulses are palpable. Sensation to a 5.07 Semmes-Weinstein monofilament test is absent. A
Silfverskiöld test result is negative, and radiograph findings are normal. Total contact casting has been
unsuccessful. Surgical treatment at this point should consist of

1. Keller arthroplasty.
2. hallux IP joint fusion.
3. hallux disarticulation.
4. hallux metatarsophalangeal fusion.
5. debridement and skin graft.

PREFERRED RESPONSE: 1

RECOMMENDED READINGS
Pinzur MS, Slovenkai MP, Trepman E, Shields NN; Diabetes Committee of American Orthopaedic
Foot and Ankle Society. Guidelines for diabetic foot care: recommendations endorsed by the Diabetes
Committee of the American Orthopaedic Foot and Ankle Society. Foot Ankle Int. 2005 Jan;26(1):113-9.
PubMed PMID: 15680122.

Lin SS, Bono CM, Lee TH. Total contact casting and Keller arthoplasty for diabetic great toe ulceration
under the interphalangeal joint. Foot Ankle Int. 2000 Jul;21(7):588-93. PubMed PMID: 10919626. 

© 2015 American Academy of Orthopaedic Surgeons 2015 Orthopaedic In-Training Examination


164 • American Academy of Orthopaedic Surgeons

Figure 69a Figure 69b Figure 69c Figure 69d

Question 69
Figures 69a through 69d are the weight-bearing radiographs and CT scan of a 33-year-old woman with
persistent ankle pain 3 years after undergoing total ankle arthroplasty and subtalar arthrodesis. She
sustained an open talar neck fracture that was treated with open reduction and internal fixation 6 years
ago. She describes constant pain with weight-bearing activity despite supportive bracing and has noticed
substantial loss of ankle motion. She denies fevers, chills, and prior wound complications. What is the
most appropriate treatment method?

1. Total ankle arthroplasty revision


2. Ankle debridement and polyethylene exchange
3. Revision subtalar arthrodesis and polyethylene exchange
4. Component explantation, bone debridement, and tibiotalocalcaneal arthrodesis
5. Lateralizing calcaneal osteotomy and dorsiflexion metatarsal osteotomy

PREFERRED RESPONSE: 4

RECOMMENDED READINGS
Berkowitz MJ, Sanders RW, Walling AK. Salvage arthrodesis after failed ankle replacement: surgical
decision making. Foot Ankle Clin. 2012 Dec;17(4):725-40.Review. PubMed PMID: 23297436.

Berkowitz MJ, Clare MP, Walling AK, Sanders R. Salvage of failed total ankle arthroplasty with fusion
using structural allograft and internal fixation. Foot Ankle Int. 2011 May;32(5):S493-502. doi: 10.3113/
FAI.2011.0493. PubMed PMID: 21733457.

© 2015 American Academy of Orthopaedic Surgeons 2015 Orthopaedic In-Training Examination


SECTION 6: Foot and Ankle • 165

Figure 80

Question 80
Figure 80 is the radiograph of a 58-year-old woman who has a painful hallux. She has pronation of the
great toe upon standing, tightness of the adductor halluces, painless first metatarsophalangeal (MTP) range
of motion, and pain at the first tarsometatarsal (TMT) joint. In addition to any distal soft-tissue procedure,
the recommended surgical treatment should consist of

1. first TMT arthrodesis.


2. first MTP arthrodesis.
3. distal Chevron osteotomy.
4. proximal first metatarsal osteotomy.
5. Silastic implant arthroplasty of the first MTP joint.

PREFERRED RESPONSE: 1

RECOMMENDED READINGS
Thompson IM, Bohay DR, Anderson JG. Fusion rate of first tarsometatarsal arthrodesis in the modified
Lapidus procedure and flatfoot reconstruction. Foot Ankle Int. 2005 Sep;26(9):698-703. PubMed PMID:
16174499.

Scranton PE, Coetzee JC, Carreira D. Arthrodesis of the first metatarsocuneiform joint: a comparative
study of fixation methods. Foot Ankle Int. 2009 Apr;30(4):341-5. doi: 10.3113/FAI.2009.0341. PubMed
PMID: 19356359.

© 2015 American Academy of Orthopaedic Surgeons 2015 Orthopaedic In-Training Examination


166 • American Academy of Orthopaedic Surgeons

Figure 93a Figure 93b

Question 93
Figures 93a and 93b are the radiographs of a 34-year-old woman who has a 2-week history of first
metatarsophalangeal (MTP) joint pain after sustaining a hyperextension injury while falling down her
stairs. The hallux is stiff and mildly swollen. She has pain with resisted plantar flexion of the first joint.
Initial treatment should consist of

1. physical therapy.
2. stabilization taping.
3. a weight-bearing cast.
4. a corticosteroid injection.
5. manipulation under anesthesia.

PREFERRED RESPONSE: 2

RECOMMENDED READINGS
George E, Harris AH, Dragoo JL, Hunt KJ. Incidence and risk factors for turf toe injuries in intercollegiate
football: data from the national collegiate athletic association injury surveillance system. Foot Ankle Int.
2014 Feb;35(2):108-15. doi: 10.1177/1071100713514038. Epub 2013 Dec 11. PubMed PMID: 24334272.

Anderson RB, Hunt KJ, McCormick JJ. Management of common sports-related injuries about the foot and
ankle. J Am Acad Orthop Surg. 2010 Sep;18(9):546-56. Review. PubMed PMID: 20810936.

© 2015 American Academy of Orthopaedic Surgeons 2015 Orthopaedic In-Training Examination


SECTION 6: Foot and Ankle • 167

Figure 105a Figure 105b Figure 105c

Question 105
Figures 105a through 105c are the weight-bearing radiographs of a morbidly obese 38-year-old woman
with diabetic peripheral neuropathy who has a 1-year history of a neuropathic plantar heel wound
measuring 3 cm x 3 cm. There is a positive “probe-to-bone” test result, and peripheral pulses are audible
by Doppler examination. The wound has been refractory to serial wound debridements, hyperbaric
treatments, and local injection of growth factors. She denies fevers and chills, and her blood glucose levels
and insulin requirements have not changed substantially. What is the most appropriate next step?

1. Hindfoot arthrodesis
2. Transtibial amputation
3. Circular frame and fine-wire fixation
4. Calcaneal saucerization and tendon Achilles lengthening
5. Percutaneous Achilles lengthening and total contact casting

PREFERRED RESPONSE: 4

RECOMMENDED READINGS
Bollinger M, Thordarson DB. Partial calcanectomy: an alternative to below knee amputation. Foot Ankle
Int. 2002 Oct;23(10):927-32. PubMed PMID: 12398145.

Smith DG, Stuck RM, Ketner L, Sage RM, Pinzur MS. Partial calcanectomy for the treatment of large
ulcerations of the heel and calcaneal osteomyelitis. An amputation of the back of the foot. J Bone Joint
Surg Am. 1992 Apr;74(4):571-6. PubMed PMID: 1583052.

Crandall RC, Wagner FW Jr. Partial and total calcanectomy: a review of thirty-one consecutive cases over
a ten-year period. J Bone Joint Surg Am. 1981 Jan;63(1):152-5. PubMed PMID: 7451518.

© 2015 American Academy of Orthopaedic Surgeons 2015 Orthopaedic In-Training Examination


168 • American Academy of Orthopaedic Surgeons

Question 117
A 57-year-old woman developed flatfoot deformity 2 years ago. She has medial ankle pain and swelling.
She also has posterior calf achiness with prolonged walking. What are the pathomechanics of her calf
discomfort?

1. Increased external leg rotation


2. A more horizontal subtalar joint axis
3. Plastic deformation of the plantar fascia
4. Altered kinetics of the foot intrinsic muscles
5. Inability to “lock” the transverse tarsal joints

PREFERRED RESPONSE: 5

RECOMMENDED READINGS
Sarrafian SK. Functional anatomy of the foot and ankle. In: Sarrafian SK, ed. Anatomy of the Foot and
Ankle. Descriptive, Topographic, Functional, 2nd ed. Philadelphia, PA:Lippincott;1993:474-602.

Ringleb SI, Kavros SJ, Kotajarvi BR, Hansen DK, Kitaoka HB, Kaufman KR. Changes in gait associated
with acute stage II posterior tibial tendon dysfunction. Gait Posture. 2007 Apr;25(4):555-64. Epub 2006
Jul 28. PubMed PMID: 16876415.

© 2015 American Academy of Orthopaedic Surgeons 2015 Orthopaedic In-Training Examination


SECTION 6: Foot and Ankle • 169

Figure 130

Question 130
Figure 130 is a standing radiograph of a 32-year-old woman with painful hallux valgus who now has
pain over her medial eminence but no lesser-toe pain. She has failed nonsurgical treatment with shoe and
activity modifications. There is no first-ray instability. The radiograph shows a 1-2 intermetatarsal angle of
17 degrees. What is the most appropriate surgical treatment?

1. Akin osteotomy
2. Distal Chevron osteotomy
3. Keller resection arthroplasty
4. First metatarsophalangeal (MTP) joint arthrodesis
5. Proximal first metatarsal osteotomy with distal soft-tissue release

PREFERRED RESPONSE: 5

RECOMMENDED READINGS
Easley ME, Kiebzak GM, Davis WH, Anderson RB. Prospective, randomized comparison of proximal
crescentic and proximal Chevron osteotomies for correction of hallux valgus deformity. Foot Ankle Int.
1996 Jun;17(6):307-16. PubMed PMID: 8791076.

Dreeben S, Mann RA. Advanced hallux valgus deformity: long-term results utilizing the distal soft tissue
procedure and proximal metatarsal osteotomy. Foot Ankle Int. 1996 Mar;17(3):142-4. PubMed PMID:
8919617.  

© 2015 American Academy of Orthopaedic Surgeons 2015 Orthopaedic In-Training Examination


170 • American Academy of Orthopaedic Surgeons

Figure 147

Question 147
Figure 147 is the standing anteroposterior radiograph of a 22-year-old woman who has forefoot pain that
developed 6 weeks ago without any antecedent injury. There is associated swelling; however, she has not
had any fevers. Examination reveals tenderness along the third metatarsal shaft. What is the best next
step?

1. Observation
2. Third-ray resection
3. Protected weight-bearing
4. Curettage and bone grafting
5. 6 weeks of intravenous antibiotics

PREFERRED RESPONSE: 3

RECOMMENDED READINGS
Shindle MK, Endo Y, Warren RF, Lane JM, Helfet DL, Schwartz EN, Ellis SJ. Stress fractures about the
tibia, foot, and ankle. J Am Acad Orthop Surg. 2012 Mar;20(3):167-76. doi: 10.5435/JAAOS-20-03-167.
Review. PubMed PMID: 22382289.

Rongstad KM, Tueting J, Rongstad M, Garrels K, Meis R. Fourth metatarsal base stress fractures in
athletes: a case series. Foot Ankle Int. 2013 Jul;34(7): 962-8. doi: 10.1177/1071100713475613. Epub 2013
Feb 5. PubMed PMID: 23386752.

© 2015 American Academy of Orthopaedic Surgeons 2015 Orthopaedic In-Training Examination


SECTION 6: Foot and Ankle • 171

Figure 158a Figure 158b

Question 158
Figures 158a and 158b are the weight-bearing radiographs of a 32-year-old woman who has persistent
lateral ankle pain 6 months after sustaining a lateral ankle sprain. She attended a course of physical
therapy and continues to use a lace-up ankle brace. She has intermittent pain along the anterior ankle with
prolonged standing and walking. She has full ankle and hindfoot range of motion, generalized tenderness
along the anterior joint line of the ankle, and a symmetric end point with ligamentous testing about the
ankle. She has painless 4+ eversion strength when compared to the contralateral side. What is the most
appropriate next step?

1. CT scan
2. MR imaging
3. Cavus foot orthotic
4. Boot immobilization
5. More physical therapy

PREFERRED RESPONSE: 5

RECOMMENDED READINGS
Prado MP, Mendes AA, Amodio DT, Camanho GL, Smyth NA, Fernandes TD. A comparative,
prospective, and randomized study of two conservative treatment protocols for first-episode lateral ankle
ligament injuries. Foot Ankle Int. 2014 Mar;35(3):201-6. doi: 10.1177/1071100713519776. Epub 2014 Jan
13. PubMed PMID: 24419825.

Kemler E, van de Port I, Backx F, van Dijk CN. A systematic review on the treatment of acute ankle
sprain: brace versus other functional treatment types. Sports Med. 2011 Mar 1;41(3):185-97. doi:
10.2165/11584370-000000000-00000. Review. PubMed PMID: 21395362. 

© 2015 American Academy of Orthopaedic Surgeons 2015 Orthopaedic In-Training Examination


172 • American Academy of Orthopaedic Surgeons

Figure 179a Figure 179b Figure 179c

Question 179
Figures 179a through 179c are the radiographs of a 61-year-old woman with insulin-dependent diabetes
mellitus and peripheral neuropathy who sustained an ankle fracture-dislocation that was treated by open
reduction and internal fixation 4 weeks ago. The incision healed uneventfully. There is erythema that
decreases with elevation. What would have minimized risk for this complication?

1. Early ankle mobilization


2. Delayed internal fixation
3. Presurgical bisphosphonates
4. Postsurgical antibiotics for 2 weeks
5. Additional internal or external fixation

PREFERRED RESPONSE: 5

RECOMMENDED READINGS
Wukich DK, Kline AJ. The management of ankle fractures in patients with diabetes. J Bone Joint Surg
Am. 2008 Jul;90(7):1570-8. doi: 10.2106/JBJS.G.01673. Review. PubMed PMID: 18594108.

Jani MM, Ricci WM, Borrelli J Jr, Barrett SE, Johnson JE. A protocol for treatment of unstable ankle
fractures using transarticular fixation in patients with diabetes mellitus and loss of protective sensibility.
Foot Ankle Int. 2003 Nov;24(11):838-44. PubMed PMID: 14655888.

Perry MD, Taranow WS, Manoli A 2nd, Carr JB. Salvage of failed neuropathic ankle fractures:
use of large-fragment fibular plating and multiple syndesmotic screws. J Surg Orthop Adv. 2005
Summer;14(2):85-91. PubMed PMID: 16115434.

© 2015 American Academy of Orthopaedic Surgeons 2015 Orthopaedic In-Training Examination


SECTION 6: Foot and Ankle • 173

Figure 193a Figure 193b Figure 193c

Question 193
Figures 193a through 193c are the radiographs of a 53-year-old woman with diabetic peripheral
neuropathy who had surgical treatment of an ankle fracture 3 months ago. She has noticed increased
swelling and deformity during the last 3 weeks after initiation of weight-bearing activity despite boot
immobilization. She denies major shifts in insulin requirements. Previous incisions are healed, and there
are no wounds or areas of skin breakdown. What is the most appropriate next step?

1. Arthrodesis
2. Transtibial amputation
3. Circular frame stabilization
4. Revision open reduction and internal fixation (ORIF)
5. Surgical debridement and intravenous antibiotics

PREFERRED RESPONSE: 1

RECOMMENDED READINGS
Rammelt S, Pyrc J, Agren PH, Hartsock LA, Cronier P, Friscia DA, Hansen ST, Schaser K, Ljungqvist J,
Sands AK. Tibiotalocalcaneal fusion using the hindfoot arthrodesis nail: a multicenter study. Foot Ankle
Int. 2013 Sep;34(9):1245-55. doi: 10.1177/1071100713487526. Epub 2013 Apr 23. PubMed PMID:
23613330.

Thomas RL, Sathe V, Habib SI. The use of intramedullary nails in tibiotalocalcaneal arthrodesis. J Am
Acad Orthop Surg. 2012 Jan;20(1):1-7. doi: 10.5435/JAAOS-20-01-001. PubMed PMID: 22207513. 

© 2015 American Academy of Orthopaedic Surgeons 2015 Orthopaedic In-Training Examination


174 • American Academy of Orthopaedic Surgeons

Figure 200a Figure 200b

Question 200 (Item Deleted)


Figures 200a and 200b are the radiographs of a 63-year-old woman with a long-standing bunion deformity
that has become painful during the last year. Wide-toe-box shoes no longer provide relief, and she desires
surgical correction. In addition to possible distal soft-tissue release, which procedure most effectively
achieves correction?

1. Distal metatarsal osteotomy and soft-tissue release


2. Distal metatarsal and proximal phalanx osteotomies
3. Proximal metatarsal osteotomy and soft-tissue release
4. Proximal metatarsal and proximal phalanx osteotomies
5. First tarsometatarsal realignment arthrodesis and soft-tissue release

PREFERRED RESPONSE: 2

RECOMMENDED READINGS
Coughlin MJ, Saltzman CL, Anderson RB: Mann’s Surgery of the Foot and Ankle, 9th ed. Philadelphia, PA:
Saunders 2014;155-321.

Jung HG, Kim TH, Park JT, Shin MH, Lee SH. Proximal reverse chevron metatarsal osteotomy, lateral
soft tissue release, and akin osteotomy through a single medial incision for hallux valgus. Foot Ankle Int.
2014 Apr;35(4):368-73. doi: 10.1177/1071100713517099. Epub 2013 Dec 18. PubMed PMID: 24351657.

© 2015 American Academy of Orthopaedic Surgeons 2015 Orthopaedic In-Training Examination


SECTION 6: Foot and Ankle • 175

Question 211
A 46-year-old woman has an 18-month history of plantar heel pain. She describes start-up symptoms that
persist with activity throughout the day. She has failed night splinting, custom inserts, cortisone injections,
and physical therapy. She has neutral-slight valgus hindfoot alignment. There is point tenderness over the
plantar medial heel, a negative Tinel sign result, and a plantar heel spur as seen on radiographs. Ankle
dorsiflexion is 15 degrees less than neutral with the knee in extension and 10 degrees with the knee in
flexion. In addition to treatment of the plantar fascia, what is the most appropriate next step?

1. Heel spur removal


2. Gastrocnemius recession
3. Tarsal tunnel release
4. Achilles tendon lengthening
5. Anterior ankle decompression

PREFERRED RESPONSE: 2

RECOMMENDED READINGS
Anderson JG, Bohay DR, Eller EB, Witt BL. Gastrocnemius recession. Foot Ankle Clin. 2014
Dec;19(4):767-86. doi: 10.1016/j.fcl.2014.09.001. Epub 2014 Nov 25. Review. PubMed PMID:
25456721.

Maskill JD, Bohay DR, Anderson JG. Gastrocnemius recession to treat isolated foot pain. Foot Ankle Int.
2010 Jan;31(1):19-23. doi: 10.3113/FAI.2010.0019. PubMed PMID: 20067718.

Question 216 (Item Deleted)


The initial nonsurgical treatment of posterior tibialis tendon insufficiency with flexible pes planovalgus
deformity and forefoot adduction consists of

1. physical therapy.
2. cast immobilization.
3. an ankle-foot orthosis.
4. a custom foot orthoses with lateral hindfoot posting.
5. a custom foot orthoses with medial hindfoot posting.

PREFERRED RESPONSE: 3

RECOMMENDED READINGS
Chao W, Wapner KL, Lee TH, Adams J, Hecht PJ. Nonoperative management of posterior tibial tendon
dysfunction. Foot Ankle Int. 1996 Dec;17(12):736-41. PubMed PMID: 8973895.

Augustin JF, Lin SS, Berberian WS, Johnson JE. Nonoperative treatment of adult acquired flat foot with
the Arizona brace. Foot Ankle Clin. 2003 Sep;8(3):491-502. PubMed PMID: 14560901. 

© 2015 American Academy of Orthopaedic Surgeons 2015 Orthopaedic In-Training Examination


176 • American Academy of Orthopaedic Surgeons

Figure 222a Figure 222b Figure 222c

Question 222
Figures 222a through 222c are the weight-bearing radiographs of a 57-year-old man with a history of
flatfoot who has persistent foot pain. He underwent a subtalar arthrodesis 1 year ago. He states that his
foot still “feels flat” compared to the contralateral foot. He is tender in the lateral subfibular area but is
nontender in the sinus tarsi. What is the most appropriate next step?

1. Lateral wall exostectomy


2. Medial column arthrodesis
3. Medializing calcaneal osteotomy and lateral column lengthening
4. Extension of arthrodesis to triple arthrodesis
5. Arthrodesis takedown with revision arthrodesis

PREFERRED RESPONSE: 5

RECOMMENDED READINGS
Johnson JE, Yu JR. Arthrodesis techniques in the management of stage II and III acquired adult flatfoot
deformity. Instr Course Lect. 2006;55:531-42. Review. PubMed PMID: 16958486.

Kadakia AR, Haddad SL. Hindfoot arthrodesis for the adult acquired flat foot. Foot Ankle Clin. 2003
Sep;8(3):569-94, x. Review. PubMed PMID: 14560906.

© 2015 American Academy of Orthopaedic Surgeons 2015 Orthopaedic In-Training Examination


SECTION 6: Foot and Ankle • 177

Question 232
A 57-year-old man with diabetes mellitus has a plantar foot ulcer at the fifth metatarsal head. He has
experienced no healing after 3 months of total contact casting. There is no erythema or drainage and his
pedal pulses are palpable. Laboratory results reveal his white blood cell (WBC) count is within defined
limits. His erythrocyte sedimentation rate is 21 mm/h (reference range [rr], 0-20 mm/h), and his C-reactive
protein level is 2.9 mg/L (rr, 0.08-3.1 mg/L). Plain radiographs suggest only decreased mineralization of
the metatarsal heads. Which diagnostic test is most appropriate at this time to help guide treatment?

1. MR imaging
2. CT scan
3. Gallium scan
4. Indium-111 WBC scan
5. Technetium-99m bone scan

PREFERRED RESPONSE: 1

RECOMMENDED READINGS
Ertugrul BM, Lipsky BA, Savk O. Osteomyelitis or Charcot neuro-osteoarthropathy? Differentiating these
disorders in diabetic patients with a foot problem. Diabet Foot Ankle. 2013 Nov 5;4. doi: 10.3402/dfa.
v4i0.21855. Review. PubMed PMID: 24205433; PubMed Central PMCID: PMC3819473.

Fujii M, Armstrong DG, Terashi H. Efficacy of magnetic resonance imaging in diagnosing diabetic foot
osteomyelitis in the presence of ischemia. J Foot Ankle Surg. 2013 Nov-Dec;52(6):717-23. doi: 10.1053/j.
jfas.2013.07.009. Erratum in: J Foot Ankle Surg. 2015 May-Jun;54(3):520. Armsrong, David G [corrected
to Armstrong, David G]. PubMed PMID: 24160720.

© 2015 American Academy of Orthopaedic Surgeons 2015 Orthopaedic In-Training Examination


178 • American Academy of Orthopaedic Surgeons

Figure 245a Figure 245b

Question 245
Figures 245a and 245b are radiographs taken 4 weeks after a total ankle arthroplasty performed through
an anteromedial approach. Which technical consideration may have most effectively prevented the
complication shown here?

1. Deltoid ligament lengthening


2. A thinner polyethylene component
3. A smaller press-fit tibial component
4. A more proximal tibial cut
5. A more lateral position of the tibial jig

PREFERRED RESPONSE: 5

RECOMMENDED READINGS
Manegold S, Haas NP, Tsitsilonis S, Springer A, Märdian S, Schaser KD. Periprosthetic fractures in
total ankle replacement: classification system and treatment algorithm. J Bone Joint Surg Am. 2013 May
1;95(9):815-20, S1-3. doi: 10.2106/JBJS.L.00572. PubMed PMID: 23636188.

McGarvey WC, Clanton TO, Lunz D. Malleolar fracture after total ankle arthroplasty: a comparison of
two designs. Clin Orthop Relat Res. 2004 Jul;(424):104-10. Review. PubMed PMID: 15241150. 

© 2015 American Academy of Orthopaedic Surgeons 2015 Orthopaedic In-Training Examination


SECTION 6: Foot and Ankle • 179

Question 251
Which soft-tissue procedure is necessary to prevent deformity after partial foot amputation through the
transverse tarsal joints?

1. Achilles tendon lengthening


2. Anterior tibialis transfer to the talar head
3. Peroneus brevis transfer to the calcaneus
4. Posterior tibialis transfer to the cuboid
5. Extensor hallucis longus transfer to the cuboid

PREFERRED RESPONSE: 1

RECOMMENDED READINGS
Dillon MP, Barker TM. Preservation of residual foot length in partial foot amputation: a biomechanical
analysis. Foot Ankle Int. 2006 Feb;27(2):110-6.PubMed PMID: 16487463.

Chang BB, Jacobs RL, Darling RC 3rd, Leather RP, Shah DM. Foot amputations. Surg Clin North Am.
1995 Aug;75(4):773-82. Review. PubMed PMID: 7638721.

Question 260
Which type of strengthening exercise is most successful in the setting of Achilles tendinopathy?

1. Isometric open-chain
2. Eccentric open-chain
3. Eccentric closed-chain
4. Concentric open-chain
5. Concentric closed-chain

PREFERRED RESPONSE: 3

RECOMMENDED READINGS
Verrall G, Schofield S, Brustad T. Chronic Achilles tendinopathy treated with eccentric stretching
program. Foot Ankle Int. 2011 Sep;32(9):843-9. PubMed PMID: 22097158.

Shalabi A, Kristoffersen-Wilberg M, Svensson L, Aspelin P, Movin T. Eccentric training of the


gastrocnemius-soleus complex in chronic Achilles tendinopathy results in decreased tendon volume and
intratendinous signal as evaluated by MRI. Am J Sports Med. 2004 Jul-Aug;32(5):1286-96. Epub 2004
May 18. PubMed PMID: 15262655.

© 2015 American Academy of Orthopaedic Surgeons 2015 Orthopaedic In-Training Examination


180 • American Academy of Orthopaedic Surgeons

Question 268
What is the most common complication associated with ankle arthroscopy?

1. Neurapraxia
2. Arterial injury
3. Deep infection
4. Superficial wound infection
5. Stiffness from joint distraction

PREFERRED RESPONSE: 1

RECOMMENDED READINGS
Carlson MJ, Ferkel RD. Complications in ankle and foot arthroscopy. Sports Med Arthrosc. 2013
Jun;21(2):135-9. doi: 10.1097/JSA.0b013e31828e5c6c. Review. PubMed PMID: 23649162.

Zengerink M, van Dijk CN. Complications in ankle arthroscopy. Knee Surg Sports Traumatol Arthrosc.
2012 Aug;20(8):1420-31. doi: 10.1007/s00167-012-2063-x. Epub 2012 Jun 5. PubMed PMID: 22669362;
PubMed Central PMCID: PMC3402678.

© 2015 American Academy of Orthopaedic Surgeons 2015 Orthopaedic In-Training Examination


SECTION 7: Sports Medicine • 181

SECTION 7: Sports Medicine

Question 11
A professional basketball player underwent anterior cruciate ligament reconstruction and is going through
an uneventful postsurgical rehabilitation. He would like to know how his return to play will be evaluated.
The ideal method with which to determine the timing of his return should involve which factors?

1. Time since his reconstruction


2. Consultation with the team owner
3. The surgeon’s rehabilitation protocol
4. Discussion with the team’s athletic trainer and physical therapist
5. Stipulations in the player’s contract regarding the percentage of games played

PREFERRED RESPONSE: 4

RECOMMENDED READINGS
Shrier I, Safai P, Charland L. Return to play following injury: whose decision should it be? Br J Sports
Med. 2014 Mar;48(5):394-401. doi: 10.1136/bjsports-2013-092492. Epub 2013 Sep 5. PubMed PMID:
24009011.

Clover J, Wall J. Return-to-play criteria following sports injury. Clin Sports Med. 2010 Jan;29(1):169-75,
table of contents. doi: 10.1016/j.csm.2009.09.008. PubMed PMID: 19945592.

© 2015 American Academy of Orthopaedic Surgeons 2015 Orthopaedic In-Training Examination


182 • American Academy of Orthopaedic Surgeons

Figure 23a Figure 23b Figure 23c

Question 23
Figures 23a through 23c are the clinical photographs of a 52-year-old tennis player who has lateral
shoulder pain with activity and difficulty with his serve. Examination reveals pain resolution during a
scapular assistance test. What is the most appropriate initial treatment of this patient’s condition?

1. Scapulothoracic fusion
2. Transfer of the levator scapulae and rhomboid muscles
3. Exploration of the spinal accessory nerve
4. Decompression of the long thoracic nerve
5. Pectoralis stretching and strengthening of rhomboids, serratus, and trapezius

PREFERRED RESPONSE: 5

RECOMMENDED READINGS
Meininger AK, Figuerres BF, Goldberg BA. Scapular winging: an update. J Am Acad Orthop Surg. 2011
Aug;19(8):453-62. Review. PubMed PMID: 21807913.

Kibler WB, Sciascia A, Wilkes T. Scapular dyskinesis and its relation to shoulder injury. J Am Acad
Orthop Surg. 2012 Jun;20(6):364-72. doi: 10.5435/JAAOS-20-06-364. Review. PubMed PMID:
22661566. 

© 2015 American Academy of Orthopaedic Surgeons 2015 Orthopaedic In-Training Examination


SECTION 7: Sports Medicine • 183

Video 42a (scan to view) Video 42b (scan to view)

Question 42
Figures 42a and 42b are the MR images of an 18-year-old collegiate football player who landed
awkwardly after jumping to defend a pass. He reported a buckling sensation in his left knee and required
assistance off the field. Knee examination reveals an increase in translation during a Lachman test but
a feeling of an end point. He has mild varus laxity without a palpable lateral collateral ligament (LCL).
What is the most appropriate treatment option?

1. LCL repair
2. LCL repair with augmentation
3. Anterior cruciate ligament (ACL) reconstruction with an Achilles allograft
4. ACL reconstruction and posterolateral corner reconstruction
5. Rehabilitation and bracing to continue play with surgical treatment after the season

PREFERRED RESPONSE: 4

RECOMMENDED READINGS
Ranawat A, Baker CL 3rd, Henry S, Harner CD. Posterolateral corner injury of the knee: evaluation and
management. J Am Acad Orthop Surg. 2008 Sep;16(9):506-18. Review. PubMed PMID: 18768708.

Engelman GH, Carry PM, Hitt KG, Polousky JD, Vidal AF. Comparison of allograft versus autograft
anterior cruciate ligament reconstruction graft survival in an active adolescent cohort. Am J Sports Med.
2014 Oct;42(10):2311-8. doi: 10.1177/0363546514541935. Epub 2014 Jul 31. PubMed PMID: 25081312. 

© 2015 American Academy of Orthopaedic Surgeons 2015 Orthopaedic In-Training Examination


184 • American Academy of Orthopaedic Surgeons

Figure 51a Figure 51b Figure 51c

Figure 51d Figure 51e

Question 51
Figures 51a through 51e are the radiographs, MR image, and CT scans of a 25-year-old man who has
had right shoulder instability for 6 years. He had an initial episode while playing basketball and a second
episode a few years later (also while playing basketball). Both injuries were anterior glenohumeral
dislocations that necessitated reduction. Currently he feels instability with simple maneuvers and
overhead activities. Examination reveals apprehension with abduction and external rotation and relief
with posterior-directed force on the proximal humerus in this position. A strengthening program has not
provided adequate stability. What is the best treatment option?

1. Shoulder arthroscopy with rotator cuff repair


2. Shoulder arthroscopy with superior labral repair
3. Shoulder arthroscopy with anterior labral repair and capsulorrhaphy
4. Shoulder stabilization procedure to address glenoid bone loss
5. Continued physical therapy and a shoulder stabilization exercise program

PREFERRED RESPONSE: 4

© 2015 American Academy of Orthopaedic Surgeons 2015 Orthopaedic In-Training Examination


SECTION 7: Sports Medicine • 185

RECOMMENDED READINGS
Burkhart SS, De Beer JF. Traumatic glenohumeral bone defects and their relationship to failure of
arthroscopic Bankart repairs: significance of the inverted-pear glenoid and the humeral engaging Hill-
Sachs lesion. Arthroscopy. 2000 Oct;16(7):677-94. PubMed PMID: 11027751.
Itoi E, Lee SB, Berglund LJ, Berge LL, An KN. The effect of a glenoid defect on anteroinferior stability of
the shoulder after Bankart repair: a cadaveric study. J Bone Joint Surg Am. 2000 Jan;82(1):35-46. PubMed
PMID: 10653082.
Burkhart SS, De Beer JF, Barth JR, Cresswell T, Roberts C, Richards DP. Results of modified Latarjet
reconstruction in patients with anteroinferior instability and significant bone loss. Arthroscopy. 2007
Oct;23(10):1033-41. Erratum in: Arthroscopy. 2007 Dec;23(12):A16. Criswell, Tim [corrected to
Cresswell, Tim]. PubMed PMID: 17916467. 

© 2015 American Academy of Orthopaedic Surgeons 2015 Orthopaedic In-Training Examination


186 • American Academy of Orthopaedic Surgeons

Question 62
A 19-year-old recreational runner sustains an inversion injury to her right ankle during a cross country run
and is unable to bear weight. Emergency department radiographs show no fracture. A splint is placed, she
is told to not place weight, and she is sent to you for follow-up 4 days later. Examination reveals swelling
and ecchymosis of her lateral ankle, tenderness over the lateral ankle ligaments, and positive talar drawer
and tilt test findings. What is the best next step?

1. Lateral ligament repair


2. MR imaging
3. Functional rehabilitation
4. Casting for 6 weeks followed by rehabilitation
5. Immobilization for 7 to 10 days followed by rehabilitation

PREFERRED RESPONSE: 5

RECOMMENDED READINGS
Petersen W, Rembitzki IV, Koppenburg AG, Ellermann A, Liebau C, Brüggemann GP, Best R. Treatment
of acute ankle ligament injuries: a systematic review. Arch Orthop Trauma Surg. 2013 Aug;133(8):1129-
41. doi: 10.1007/s00402-013-1742-5. Epub 2013 May 28. Review. PubMed PMID: 23712708.

Kaminski TW, Hertel J, Amendola N, Docherty CL, Dolan MG, Hopkins JT, Nussbaum E, Poppy W,
Richie D; National Athletic Trainers’ Association. National Athletic Trainers’ Association position
statement: conservative management and prevention of ankle sprains in athletes. J Athl Train. 2013 Jul-
Aug;48(4):528-45. doi: 10.4085/1062-6050-48.4.02. Review. PubMed PMID: 23855363.

Pihlajamдki H, Hietaniemi K, Paavola M, Visuri T, Mattila VM. Surgical versus functional treatment
for acute ruptures of the lateral ligament complex of the ankle in young men: a randomized controlled
trial. J Bone Joint Surg Am. 2010 Oct 20;92(14):2367-74. doi: 10.2106/JBJS.I.01176. Epub 2010 Sep 10.
PubMed PMID: 20833874.

Lamb SE, Marsh JL, Hutton JL, Nakash R, Cooke MW; Collaborative Ankle Support Trial (CAST
Group). Mechanical supports for acute, severe ankle sprain: a pragmatic, multicentre, randomised
controlled trial. Lancet. 2009 Feb 14;373(9663):575-81. doi: 10.1016/S0140-6736(09)60206-3. PubMed
PMID: 19217992.

van Dijk CN, Lim LS, Bossuyt PM, Marti RK. Physical examination is sufficient for the diagnosis of
sprained ankles. J Bone Joint Surg Br. 1996 Nov;78(6):958-62. PubMed PMID: 8951015.

© 2015 American Academy of Orthopaedic Surgeons 2015 Orthopaedic In-Training Examination


SECTION 7: Sports Medicine • 187

Question 83
A 17-year-old football player is hit during the course of play. He lies down on the field as the training staff
enters the field to assist. By the time they arrive to the player, he is sitting up. He quickly stands and is
walked to the sideline. The player experiences initial confusion when questioned on the sideline, but this
quickly passes. He has no memory loss; is alert and oriented to person, place, and time; and has a mild
headache. He wants to return to the game and the coach asks if he can play. What is the best next step?

1. No return to play this game


2. Immediate return to the game
3. Obtain an immediate head CT scan
4. Return to the game only if the team’s needs necessitate it
5. Return to the game when his headache symptoms resolve

PREFERRED RESPONSE: 1

RECOMMENDED READINGS
McCrory P, Meeuwisse WH, Aubry M, Cantu B, Dvorák J, Echemendia RJ, Engebretsen L, Johnston
K, Kutcher JS, Raftery M, Sills A, Benson BW, Davis GA, Ellenbogen RG, Guskiewicz K, Herring
SA, Iverson GL, Jordan BD, Kissick J, McCrea M, McIntosh AS, Maddocks D, Makdissi M, Purcell
L, Putukian M, Schneider K, Tator CH, Turner M. Consensus statement on concussion in sport: the 4th
International Conference on Concussion in Sport held in Zurich, November 2012. Br J Sports Med. 2013
Apr;47(5):250-8. doi: 10.1136/bjsports-2013-092313. PubMed PMID: 23479479.

Putukian M. The acute symptoms of sport-related concussion: diagnosis and on-field management.
Clin Sports Med. 2011 Jan;30(1):49-61, viii. doi: 10.1016/j.csm.2010.09.005. Review. PubMed PMID:
21074081.

© 2015 American Academy of Orthopaedic Surgeons 2015 Orthopaedic In-Training Examination


188 • American Academy of Orthopaedic Surgeons

Question 99
A 26-year-old professional football player is experiencing sharp stabbing pain that radiates from his
neck to his left thumb. The pain began acutely after a tackle. An MR image of the cervical spine reveals
a lateral disk herniation with foraminal encroachment but no abutment of the cervical cord or central
stenosis. Which treatment most likely will allow an expedient return to play?

1. Cervical foraminotomy
2. Cervical disk replacement
3. Oral methylprednisolone
4. Chiropractic manipulation
5. Anterior cervical decompression and fusion

PREFERRED RESPONSE: 3

RECOMMENDED READINGS
Meredith DS, Jones KJ, Barnes R, Rodeo SA, Cammisa FP, Warren RF. Operative and nonoperative
treatment of cervical disc herniation in National Football League athletes. Am J Sports Med. 2013
Sep;41(9):2054-8. doi: 10.1177/0363546513493247. Epub 2013 Jun 20. PubMed PMID: 23788681.

Wong JJ, Côté P, Quesnele JJ, Stern PJ, Mior SA. The course and prognostic factors of symptomatic
cervical disc herniation with radiculopathy: a systematic review of the literature. Spine J. 2014 Aug
1;14(8):1781-9. doi: 10.1016/j.spinee.2014.02.032. Epub 2014 Mar 12. PubMed PMID: 24614255.

Question 112
At the request of his parents, a 12-year-old Little League player is being evaluated for shoulder and elbow
pain in his pitching arm. He plays baseball through the spring, summer, and fall. When he is not playing
for multiple teams, he works with a pitching coach, throwing 3 to 4 days a week. He throws fastballs, a
change-up, and recently began throwing a curveball. With regard to his shoulder and elbow pain, what is
the most appropriate advice?

1. Throwing the curveball causes his pain, so be sure to inform his pitching coach.
2. Not pitch for at least 4 months out of the year.
3. Increase the time he spends with his pitching coach.
4. Begin a weight-lifting program for his shoulder and elbow.
5. Ulnar collateral ligament (UCL) reconstruction to address his elbow mechanics.

PREFERRED RESPONSE: 2

© 2015 American Academy of Orthopaedic Surgeons 2015 Orthopaedic In-Training Examination


SECTION 7: Sports Medicine • 189

RECOMMENDED READINGS
Olsen SJ 2nd, Fleisig GS, Dun S, Loftice J, Andrews JR. Risk factors for shoulder and elbow injuries in
adolescent baseball pitchers. Am J Sports Med. 2006 Jun;34(6):905-12. Epub 2006 Feb 1. PubMed PMID:
16452269.

Bruce JR, Andrews JR. Ulnar collateral ligament injuries in the throwing athlete. J Am Acad Orthop Surg.
2014 May;22(5):315-25. doi: 10.5435/JAAOS-22-05-315. Review. PubMed PMID: 24788447.

Question 125
A 43-year-old marathon runner has pain deep in her buttocks. She has pain with running and difficulty
sitting on hard surfaces because of her pain at the ischium. She slipped on ice almost 1 year ago and
believes this was when her pain started. An MR image of the hip reveals a partial avulsion of the proximal
hamstring origin. She has been doing appropriate physical therapy since her injury occurred but has not
experienced symptom relief. What is the most appropriate treatment option?

1. Percutaneous tenotomy
2. Cessation of distance running
3. Use of a foam roller in therapy
4. Open debridement and repair
5. Administration of an oral corticosteroid

PREFERRED RESPONSE: 4

RECOMMENDED READINGS
Bowman KF Jr, Cohen SB, Bradley JP. Operative management of partial-thickness tears of
the proximal hamstring muscles in athletes. Am J Sports Med. 2013 Jun;41(6):1363-71. doi:
10.1177/0363546513482717. Epub 2013 Apr 10. PubMed PMID: 23576684.

Hofmann KJ, Paggi A, Connors D, Miller SL. Complete Avulsion of the Proximal Hamstring Insertion:
Functional Outcomes After Nonsurgical Treatment. J Bone Joint Surg Am. 2014 Jun 18;96(12):1022-1025.
[Epub ahead of print] PubMed PMID: 24951738.

© 2015 American Academy of Orthopaedic Surgeons 2015 Orthopaedic In-Training Examination


190 • American Academy of Orthopaedic Surgeons

Question 146
A 20-year-old collegiate football player is seen after season completion with midfoot pain and bone scan
findings consistent with a navicular stress fracture. What is the most effective treatment option?

1. Rest from running, but no immobilization


2. Placement in a cast with no weight bearing for 2 weeks
3. Placement in a cast with no weight bearing for 6 weeks
4. Placement in a fracture boot with weight bearing as tolerated
5. Open debridement of the fracture site with internal fixation

PREFERRED RESPONSE: 3

RECOMMENDED READINGS
Torg JS, Moyer J, Gaughan JP, Boden BP. Management of tarsal navicular stress fractures: conservative
versus surgical treatment: a meta-analysis. Am J Sports Med. 2010 May; 38(5):1048-53. doi:
10.1177/0363546509355408. Epub 2010 Mar 2. PubMed PMID: 20197494.

Burne SG, Mahoney CM, Forster BB, Koehle MS, Taunton JE, Khan KM. Tarsal navicular stress injury:
long-term outcome and clinicoradiological correlation using both computed tomography and magnetic
resonance imaging. Am J Sports Med. 2005 Dec;33(12):1875-81. Epub 2005 Sep 12. PubMed PMID:
16157855.

Question 159
An 18-year-old freshman is seen for her preparticipation lacrosse physical. She reports a history of asthma
but says that the albuterol inhaler that she was prescribed does not effectively clear her symptoms. She
has difficulty with breathing only during conditioning workouts, practices, and games. What is the most
appropriate recommendation for this athlete?

1. Refer her for confirmation of her asthma diagnosis


2. Initiate a course of oral steroids
3. Add a steroid inhaler
4. Add nebulizer treatments
5. Disqualify her from participation

PREFERRED RESPONSE: 1

RECOMMENDED READINGS
Nielsen EW, Hull JH, Backer V. High prevalence of exercise-induced laryngeal obstruction in athletes.
Med Sci Sports Exerc. 2013 Nov;45(11):2030-5. doi: 10.1249/MSS.0b013e318298b19a. PubMed PMID:
23657163.

Eichenberger PA, Diener SN, Kofmehl R, Spengler CM. Effects of exercise training on airway
hyperreactivity in asthma: a systematic review and meta-analysis. Sports Med. 2013 Nov;43(11):1157-70.
doi: 10.1007/s40279-013-0077-2. Review. PubMed PMID: 23846823.

© 2015 American Academy of Orthopaedic Surgeons 2015 Orthopaedic In-Training Examination
SECTION 7: Sports Medicine • 191

Question 171
What is the best reason to use an autograft (rather than an allograft) for anterior cruciate ligament (ACL)
reconstruction in a young athlete?

1. Lower infection risk


2. Lower graft rupture rate
3. Lower long-term risk for arthritis
4. Lack of donor-site morbidity
5. Better incorporation of the graft material

PREFERRED RESPONSE: 2

RECOMMENDED READINGS
Kaeding CC, Aros B, Pedroza A, Pifel E, Amendola A, Andrish JT, Dunn WR, Marx RG, McCarty
EC, Parker RD, Wright RW, Spindler KP. Allograft Versus Autograft Anterior Cruciate Ligament
Reconstruction: Predictors of Failure From a MOON Prospective Longitudinal Cohort. Sports Health.
2011 Jan;3(1):73-81. PubMed PMID: 23015994; PubMed Central PMCID: PMC3445196.

Krych AJ, Jackson JD, Hoskin TL, Dahm DL. A meta-analysis of patellar tendon autograft versus patellar
tendon allograft in anterior cruciate ligament reconstruction. Arthroscopy. 2008 Mar;24(3):292-8. doi:
10.1016/j.arthro.2007.08.029. Epub 2007 Nov 5. Review. PubMed PMID: 18308180.

© 2015 American Academy of Orthopaedic Surgeons 2015 Orthopaedic In-Training Examination


192 • American Academy of Orthopaedic Surgeons

Figure 181a Figure 181b

Question 181
Figures 181a and 181b are the arthroscopic views of an 18-year-old collegiate basketball player who has
recurrent effusions 9 months after his fourth patella dislocation. He has had bracing and physical therapy
since the previous dislocation. Radiographs reveal lateral congruence. MR imaging shows articular
cartilage loss in the inferolateral patella. Lateral tibial tubercle offset relative to the trochlea groove is 19
mm. Diagnostic arthroscopy figures show the patella before and after debridement. An articular cartilage
biopsy is obtained. Reimplantation of articular cartilage should be undertaken in conjunction with which
other procedure(s)?

1. Galeazzi realignment
2. Anteromedial tibial tubercle transfer
3. Vastus medialis oblique (VMO) advancement
4. VMO advancement and lateral release
5. Medial tibial tubercle transfer and lateral release

PREFERRED RESPONSE: 2

RECOMMENDED READINGS
Beck PR, Thomas AL, Farr J, Lewis PB, Cole BJ. Trochlear contact pressures after anteromedialization
of the tibial tubercle. Am J Sports Med. 2005 Nov;33(11):1710-5. Epub 2005 Aug 10. PubMed PMID:
16093531.

Strauss EJ, Galos DK. The evaluation and management of cartilage lesions affecting the patellofemoral
joint. Curr Rev Musculoskelet Med. 2013 Jun;6(2):141-9. doi: 10.1007/s12178-013-9157-z. PubMed
PMID: 23392780; PubMed Central PMCID: PMC3702778.

© 2015 American Academy of Orthopaedic Surgeons 2015 Orthopaedic In-Training Examination


SECTION 7: Sports Medicine • 193

Question 184
Which effect does initiation of early eccentric strengthening at 3 weeks from surgery have in rehabilitation
of anterior cruciate ligament (ACL) reconstruction compared to traditional initiation at 3 months?

1. Improved control of postsurgical effusion


2. Increased pain in the surgical extremity
3. Increased muscle mass of the quadriceps and hamstrings
4. Increased risk for graft loosening because the tunnels have not healed
5. Decreased risk for rupture of the contralateral ACL

PREFERRED RESPONSE: 3

RECOMMENDED READINGS
Kruse LM, Gray B, Wright RW. Rehabilitation after anterior cruciate ligament reconstruction: a systematic
review. J Bone Joint Surg Am. 2012 Oct 3;94(19):1737-48. doi: 10.2106/JBJS.K.01246. Review. PubMed
PMID: 23032584; PubMed Central PMCID: PMC3448301.

van Grinsven S, van Cingel RE, Holla CJ, van Loon CJ. Evidence-based rehabilitation following anterior
cruciate ligament reconstruction. Knee Surg Sports Traumatol Arthrosc. 2010 Aug;18(8):1128-44. doi:
10.1007/s00167-009-1027-2. Epub 2010 Jan 13. Review. PubMed PMID: 20069277.

Gerber JP, Marcus RL, Dibble LE, Greis PE, Burks RT, Lastayo PC. Safety, feasibility, and efficacy of
negative work exercise via eccentric muscle activity following anterior cruciate ligament reconstruction. J
Orthop Sports Phys Ther. 2007 Jan;37(1):10-8. PubMed PMID: 17286094. 

© 2015 American Academy of Orthopaedic Surgeons 2015 Orthopaedic In-Training Examination


194 • American Academy of Orthopaedic Surgeons

Question 188
A 20-year-old dancer has atraumatic onset of midfoot pain. Radiographic findings are normal. Her body
mass index is 18.5, and she has had 5 menstrual cycles during the past year. What is the long-term risk of
no treatment?

1. Secondary infertility
2. Functional hyperthyroidism
3. Rebound uterine hypertrophy
4. Secondary calcium deficiency
5. Irreversible loss of bone mineral density

PREFERRED RESPONSE: 5

RECOMMENDED READINGS
Nazem TG, Ackerman KE. The female athlete triad. Sports Health. 2012 Jul;4(4):302-11. PubMed PMID:
23016101; PubMed Central PMCID: PMC3435916.

Nattiv A, Loucks AB, Manore MM, Sanborn CF, Sundgot-Borgen J, Warren MP; American College of
Sports Medicine. American College of Sports Medicine position stand. The female athlete triad. Med Sci
Sports Exerc. 2007 Oct;39(10):1867-82. PubMed PMID: 17909417.

© 2015 American Academy of Orthopaedic Surgeons 2015 Orthopaedic In-Training Examination


SECTION 7: Sports Medicine • 195

Figure 201a Figure 201b Figure 201c

Figure 201d Figure 201e Figure 201f

Question 201
Figures 201a through 201f are the radiographs and MR images of a 64-year-old woman who has left
shoulder pain. She has had the pain “on and off,” and it mostly bothers her at night. She has been taking
anti-inflammatory pain medications and has experienced some relief. Examination reveals mildly
diminished range of motion in elevation and external rotation, and she reports moderate pain with forced
abduction of the shoulder. Her strength in elevation is 4/5, which is limited by pain, and her external
rotation strength is 5/5. She has a negative belly press sign. What is the most appropriate next step?

1. Injection of platelet-rich plasma to the subacromial space


2. A 6-week course of physical therapy
3. Arthroscopic capsular release with manipulation under anesthesia
4. Arthroscopic subacromial decompression with coracoacromial ligament release
5. Arthroscopic rotator cuff repair

PREFERRED RESPONSE: 2
© 2015 American Academy of Orthopaedic Surgeons 2015 Orthopaedic In-Training Examination
196 • American Academy of Orthopaedic Surgeons

RECOMMENDED READINGS
Kesikburun S, Tan AK, Yilmaz B, Yaşar E, Yazicioğlu K. Platelet-rich plasma injections in the treatment
of chronic rotator cuff tendinopathy: a randomized controlled trial with 1-year follow-up. Am J Sports
Med. 2013 Nov;41(11):2609-16. doi: 10.1177/0363546513496542. Epub 2013 Jul 26. PubMed PMID:
23893418.

Hall MP, Band PA, Meislin RJ, Jazrawi LM, Cardone DA. Platelet-rich plasma: current concepts and
application in sports medicine. J Am Acad Orthop Surg. 2009 ct;17(10):602-8. Review. Erratum in: J Am
Acad Orthop Surg. 2010 Jan;18(1):17A. PubMed PMID: 19794217.

Ketola S, Lehtinen J, Arnala I, Nissinen M, Westenius H, Sintonen H, Aronen P, Konttinen YT,


Malmivaara A, Rousi T. Does arthroscopic acromioplasty provide any additional value in the treatment
of shoulder impingement syndrome?: a two-year randomised controlled trial. J Bone Joint Surg Br. 2009
Oct;91(10):1326-34. doi: 10.1302/0301-620X.91B10.22094. PubMed PMID: 19794168.

Pedowitz RA, Yamaguchi K, Ahmad CS, Burks RT, Flatow EL, Green A, Iannotti JP, Miller BS, Tashjian
RZ, Watters WC 3rd, Weber K, Turkelson CM, Wies JL, Anderson S, St Andre J, Boyer K, Raymond
L, Sluka P, McGowan R; American Academy of Orthopaedic Surgeons. Optimizing the management of
rotator cuff problems. J Am Acad Orthop Surg. 2011 Jun;19(6):368-79. PubMed PMID: 21628648. 

© 2015 American Academy of Orthopaedic Surgeons 2015 Orthopaedic In-Training Examination


SECTION 7: Sports Medicine • 197

Figure 209

Question 209 (Item Deleted)


Figure 209 is the clinical photograph of a 39-year-old right-hand-dominant woman who had surgery on
her lateral elbow 3 months ago. Despite rehabilitation, she has continued lateral elbow pain, “popping”
when she attempts to lift anything heavy, and episodic wound drainage. Examination reveals an incision
over her lateral epicondyle that appears to have a recently closed sinus tract. She is tender over the lateral
epicondyle, has full elbow range of motion, and guards with any other testing maneuver. The figure
reveals the findings upon opening the incision. What are the best next steps?

1. Irrigation and debridement, wound closure, and an infectious disease consultation


2. Irrigation and debridement, packing the wound open, and an infectious disease consultation
3. Repair of the extensor carpi radialis brevis and immobilization for several weeks
4. Repair of the lateral ulnar collateral ligament and immobilization for several weeks
5. Reconstruction of the lateral ulnar collateral ligament with autologous palmaris longus tendon

PREFERRED RESPONSE: 5

RECOMMENDED READINGS
Parsons BO, Ramsey ML Elbow instability and reconstruction. In: Flynn JM, ed. Orthopaedic Knowledge
Update 10. Rosemont, IL: American Academy of Orthopaedic Surgeons; 2011: 343-350.

Kelly EW, Morrey BF, O’Driscoll SW. Complications of elbow arthroscopy. J Bone Joint Surg Am. 2001
Jan;83-A(1):25-34. PubMed PMID: 11205854.

Calfee RP, Patel A, DaSilva MF, Akelman E. Management of lateral epicondylitis: current concepts. J Am
Acad Orthop Surg. 2008 Jan;16(1):19-29. Review. PubMed PMID: 18180389.

© 2015 American Academy of Orthopaedic Surgeons 2015 Orthopaedic In-Training Examination


198 • American Academy of Orthopaedic Surgeons

Question 231
In midseason, a collegiate football player has fatigue, fevers, and “swollen glands.” Examination reveals
an enlarged spleen. Laboratory studies confirm the diagnosis of mononucleosis. When should he be
cleared to return to play?

1. 1 week after starting antibiotic treatment


2. 2 weeks after symptom onset
3. 3 weeks after he is asymptomatic
4. He should not return for the current season
5. As soon as the fatigue and fevers cease

PREFERRED RESPONSE: 3

RECOMMENDED READINGS
Feeley BT, Behera S, Luke AC. Medical issues for the athlete. In: Cannada LK, ed. Orthopaedic
Knowledge Update 11. Rosemont, IL: American Academy of Orthopaedic Surgeons; 2014:115-127.

Putukian M, O’Connor FG, Stricker P, McGrew C, Hosey RG, Gordon SM, Kinderknecht J, Kriss V,
Landry G. Mononucleosis and athletic participation: an evidence-based subject review. Clin J Sport Med.
2008 Jul;18(4):309-15. doi: 10.1097/JSM.0b013e31817e34f8. Review. PubMed PMID: 18614881.

Jaworski CA, Donohue B, Kluetz J. Infectious disease. Clin Sports Med. 2011 Jul;30(3):575-90. doi:
10.1016/j.csm.2011.03.006. PubMed PMID: 21658549.

© 2015 American Academy of Orthopaedic Surgeons 2015 Orthopaedic In-Training Examination


SECTION 7: Sports Medicine • 199

Question 236
When evaluating accelerated vs nonaccelerated rehabilitation programs after anterior cruciate ligament
reconstruction (ACL), outcomes are

1. similar with each rehabilitation method.


2. better with accelerated rehabilitation because of earlier return to activity.
3. better with nonaccelerated rehabilitation because of increased graft strength.
4. worse with accelerated rehabilitation because of graft loosening.
5. worse with nonaccelerated rehabilitation because of higher frequency of knee stiffness.

PREFERRED RESPONSE: 1

RECOMMENDED READINGS
Kruse LM, Gray B, Wright RW. Rehabilitation after anterior cruciate ligament reconstruction: a systematic
review. J Bone Joint Surg Am. 2012 Oct 3;94(19):1737-48. doi: 10.2106/JBJS.K.01246. Review. PubMed
PMID: 23032584; PubMed Central PMCID: PMC3448301.

Beynnon BD, Johnson RJ, Naud S, Fleming BC, Abate JA, Brattbakk B, Nichols CE. Accelerated versus
nonaccelerated rehabilitation after anterior cruciate ligament reconstruction: a prospective, randomized,
double-blind investigation evaluating knee joint laxity using roentgen stereophotogrammetric analysis.
Am J Sports Med. 2011 Dec;39(12):2536-48. doi: 10.1177/0363546511422349. Epub 2011 Sep 27.
PubMed PMID: 21952714.

Question 244
Which intra-articular injury is most commonly associated with an anterior cruciate ligament (ACL) tear?

1. Medial meniscus tear


2. Lateral meniscus tear
3. Lateral tibial plateau fracture
4. Articular cartilage lesion of the lateral femoral condyle
5. Articular cartilage lesion of the medial femoral condyle

PREFERRED RESPONSE: 2

RECOMMENDED READINGS
Piasecki DP, Spindler KP, Warren TA, Andrish JT, Parker RD. Intraarticular injuries associated with
anterior cruciate ligament tear: findings at ligament reconstruction in high school and recreational athletes.
An analysis of sex-based differences. Am J Sports Med. 2003 Jul-Aug;31(4):601-5. PubMed PMID:
12860552.

Spindler KP, Schils JP, Bergfeld JA, Andrish JT, Weiker GG, Anderson TE, Piraino DW, Richmond
BJ, Medendorp SV. Prospective study of osseous, articular, and meniscal lesions in recent anterior
cruciate ligament tears by magnetic resonance imaging and arthroscopy. Am J Sports Med. 1993 Jul-
Aug;21(4):551-7. PubMed PMID: 8368416.

© 2015 American Academy of Orthopaedic Surgeons 2015 Orthopaedic In-Training Examination
200 • American Academy of Orthopaedic Surgeons

Figure 252a Figure 252b Figure 252c

Question 252
Figures 252a through 252c are the radiographs of a 27-year-old woman who lacks range of motion of her
left elbow. She has always had limited range of motion in this elbow; however, this did not bother her
until she gave birth. She cannot feed the child using her left arm secondary to the lack of motion, but she
denies pain. Examination reveals her left elbow has full and equivalent flexion and extension with a fixed
20 degrees of pronation. No further pronation or supination is present actively or passively. No pain is
elicited during the examination. Her right elbow has full flexion and extension as well as full pronation
and supination. What is the underlying cause of her condition?

1. Posttraumatic event
2. Formation failure
3. Segmentation failure
4. Autosomal recessive trait
5. Disuse

PREFERRED RESPONSE: 3

RECOMMENDED READINGS
Elliott AM, Kibria L, Reed MH. The developmental spectrum of proximal radioulnar synostosis. Skeletal
Radiol. 2010 Jan;39(1):49-54. doi: 10.1007/s00256-009-0762-2. PubMed PMID: 19669136.

Cleary JE, Omer GE Jr. Congenital proximal radio-ulnar synostosis. Natural history and functional
assessment. J Bone Joint Surg Am. 1985 Apr;67(4):539-45. PubMed PMID: 3980498.

© 2015 American Academy of Orthopaedic Surgeons 2015 Orthopaedic In-Training Examination


SECTION 7: Sports Medicine • 201

Question 275
A small fracture of the lateral tibial plateau known as a Segond fracture often is associated with anterior
cruciate ligament (ACL) injury and represents an avulsion of the

1. anterolateral ligament.
2. tibial ACL attachment.
3. femoral ACL attachment.
4. lateral capsule.
5. lateral collateral ligament (LCL).

PREFERRED RESPONSE: 1

RECOMMENDED READINGS
Dodds AL, Halewood C, Gupte CM, Williams A, Amis AA. The anterolateralligament: Anatomy,
length changes and association with the Segond fracture. Bone Joint J. 2014 Mar;96-B(3):325-31. doi:
10.1302/0301-620X.96B3.33033. PubMed PMID:24589786.

Claes S, Luyckx T, Vereecke E, Bellemans J. The Segond fracture: a bony injury of the anterolateral
ligament of the knee. Arthroscopy. 2014 Nov;30(11):1475-82. doi: 10.1016/j.arthro.2014.05.039. Epub
2014 Aug 8. PubMed PMID: 25124480.

Claes S, Vereecke E, Maes M, Victor J, Verdonk P, Bellemans J. Anatomy of the anterolateral ligament
of the knee. J Anat. 2013 Oct;223(4):321-8. doi: 10.1111/joa.12087. Epub 2013 Aug 1. PubMed PMID:
23906341.

© 2015 American Academy of Orthopaedic Surgeons 2015 Orthopaedic In-Training Examination


202 • American Academy of Orthopaedic Surgeons

SECTION 8: Shoulder and Elbow

Question 12
A 72-year-old man with cuff tear arthropathy is scheduled for a reverse total shoulder arthroplasty. Which
examination finding most strongly indicates the need for a concomitant latissimus dorsi transfer?

1. Pseudoparalysis
2. Anterosuperior escape
3. Positive drop arm sign
4. Positive Hornblower’s test result
5. Passive external rotation to 0 degrees

PREFERRED RESPONSE: 4

RECOMMENDED READINGS
Walch G, Boulahia A, Calderone S, Robinson AH. The ‘dropping’ and ‘hornblower’s signs in evaluation
of rotator-cuff tears. J Bone Joint Surg Br. 1998 Jul;80(4):624-8. PubMed PMID: 9699824.

Puskas GJ, Catanzaro S, Gerber C. Clinical outcome of reverse total shoulder arthroplasty combined with
latissimus dorsi transfer for the treatment of chronic combined pseudoparesis of elevation and external
rotation of the shoulder. J Shoulder Elbow Surg. 2014 Jan;23(1):49-57. doi: 10.1016/j.jse.2013.04.008.
Epub 2013 Jun 18. PubMed PMID: 23790326. 

© 2015 American Academy of Orthopaedic Surgeons 2015 Orthopaedic In-Training Examination


SECTION 8: Shoulder and Elbow • 203

Figure 26a Figure 26b Figure 26c

Figure 26d Figure 26e Figure 26f

Question 26
An 82-year-old right-hand-dominant man has progressive pain in his left shoulder and loss of overhead
function. He underwent a reverse total shoulder arthroplasty for rotator cuff–deficient arthritis 30 months
ago. Radiographs taken at the current visit are shown in Figures 26a through 26c, and select images from a
CT scan are shown in Figures 26d through 26f. Infection workup findings are negative. Examination reveals
70 degrees of forward elevation with pain with 20 degrees of external rotation. Axillary nerve function is
intact. Definitive treatment should consist of

1. retention of the glenoid component with a reverse humeral stem revision.


2. removal of both glenoid and humeral components (“resection arthroplasty”).
3. removal of the glenoid component with humeral stem revision to hemiarthroplasty.
4. in situ placement of the larger glenosphere component with revision reverse humeral stem.
5. revision glenoid baseplate with superiorly placed bone graft with revision reverse humeral stem.

PREFERRED RESPONSE: 5

© 2015 American Academy of Orthopaedic Surgeons 2015 Orthopaedic In-Training Examination


204 • American Academy of Orthopaedic Surgeons

RECOMMENDED READINGS
Boileau P, Melis B, Duperron D, Moineau G, Rumian AP, Han Y. Revision surgery of reverse shoulder
arthroplasty. J Shoulder Elbow Surg. 2013 Oct;22(10):1359-70. doi: 10.1016/j.jse.2013.02.004. Epub
2013 May 22. PubMed PMID: 23706884.

Holcomb JO, Cuff D, Petersen SA, Pupello DR, Frankle MA. Revision reverse shoulder arthroplasty for
glenoid baseplate failure after primary reverse shoulder arthroplasty. J Shoulder Elbow Surg. 2009 Sep-
Oct;18(5):717-23. doi: 10.1016/j.jse.2008.11.017. Epub 2009 Mar 17. PubMed PMID: 19278872. 

Figure 34a Figure 34b

Question 34
Figures 34a and 34b are the radiographs of a 22-year-old woman who sustained an elbow injury after a
fall. In the emergency department, her skin is noted to be intact. There is no tenderness over the medial
elbow or the distal radioulnar joint. What is the best treatment option?

1. Radial head arthroplasty


2. Long-arm cast for 3 weeks
3. Excision of the fracture fragments
4. Open reduction and internal fixation (ORIF)
5. Early motion with a functional brace

PREFERRED RESPONSE: 4

© 2015 American Academy of Orthopaedic Surgeons 2015 Orthopaedic In-Training Examination


SECTION 8: Shoulder and Elbow • 205

RECOMMENDED READINGS
Pike JM, Grewal R, Athwal GS, Faber KJ, King GJ. Open reduction and internal fixation of radial
head fractures: do outcomes differ between simple and complex injuries? Clin Orthop Relat Res. 2014
Jul;472(7):2120-7. doi: 10.1007/s11999-014-3519-8. PubMed PMID: 24585322; PubMed Central
PMCID: PMC4048425.

Yoon A, King GJ, Grewal R. Is ORIF superior to nonoperative treatment in isolated displaced partial
articular fractures of the radial head? Clin Orthop Relat Res. 2014 Jul;472(7):2105-12. doi: 10.1007/
s11999-014-3541-x. PubMed PMID: 24577616; PubMed Central PMCID: PMC4048435. 

© 2015 American Academy of Orthopaedic Surgeons 2015 Orthopaedic In-Training Examination


206 • American Academy of Orthopaedic Surgeons

Video 41 (scan to view)

Question 41
Video 41 is viewed from the posterior portal during shoulder arthroscopy. The instrument is introduced
through the anterior portal. Based on these video findings, which examination maneuver would most likely
elicit an abnormality of the indicated structure?

1. Speed test
2. O’Brien test
3. Belly press test
4. Hornblower’s test
5. Cross-body adduction

PREFERRED RESPONSE: 3

RECOMMENDED READINGS
Faruqui S, Wijdicks C, Foad A. Sensitivity of physical examination versus arthroscopy in diagnosing
subscapularis tendon injury. Orthopedics. 2014 Jan;37(1):e29-33. PubMed PMID: 24683653.

Yoon JP, Chung SW, Kim SH, Oh JH. Diagnostic value of four clinical tests for the evaluation of
subscapularis integrity. J Shoulder Elbow Surg. 2013 Sep;22(9):1186-92. doi: 10.1016/j.jse.2012.12.002.
Epub 2013 Feb 20. PubMed PMID:23434234. 

© 2015 American Academy of Orthopaedic Surgeons 2015 Orthopaedic In-Training Examination


SECTION 8: Shoulder and Elbow • 207

Figure 53a Figure 53b Figure 53c

Figure 53d Figure 53e

Question 53
Figures 53a through 53e are the radiographs and CT scans of a 78-year-old man who is seen in the
emergency department with persistent left shoulder pain 3 weeks after a fall. He had full function of his
left shoulder prior to the fall. He lives alone, and his medical history includes diabetes and hypertension.
Examination reveals pain with any attempted shoulder motion. He is neurovascularly intact in the left
upper extremity. What is the best next step?

1. Hemiarthroplasty with biceps tenodesis


2. Unconstrained total shoulder arthroplasty
3. Reverse shoulder arthroplasty with tuberosity repair
4. Percutaneous pinning
5. Open reduction and internal fixation with locked plating

PREFERRED RESPONSE: 3

© 2015 American Academy of Orthopaedic Surgeons 2015 Orthopaedic In-Training Examination


208 • American Academy of Orthopaedic Surgeons

RECOMMENDED READINGS
Bufquin T, Hersan A, Hubert L, Massin P. Reverse shoulder arthroplasty for the treatment of three- and
four-part fractures of the proximal humerus in the elderly: a prospective review of 43 cases with a short-
term follow-up. J Bone Joint Surg Br. 2007 Apr;89(4):516-20. PubMed PMID: 17463122.

Jobin CM, Galdi B, Anakwenze OA, Ahmad CS, Levine WN. Reverse shoulder arthroplasty for the
management of proximal humerus fractures. J Am Acad Orthop Surg. 2015 Mar;23(3):190-201. doi:
10.5435/JAAOS-D-13-00190. Epub 2015 Jan 28. Review. PubMed PMID: 25630370.

Anakwenze OA, Zoller S, Ahmad CS, Levine WN. Reverse shoulder arthroplasty for acute proximal
humerus fractures: a systematic review. J Shoulder Elbow Surg. 2014 Apr;23(4):e73-80. doi: 10.1016/j.
jse.2013.09.012. Epub 2014 Jan 7. Review. PubMed PMID: 24406120.

© 2015 American Academy of Orthopaedic Surgeons 2015 Orthopaedic In-Training Examination


SECTION 8: Shoulder and Elbow • 209

Figure 66
Acevedo DC, Paxton ES, Kukelyansky I, Abboud J, Ramsey M. Radial Head Arthroplasty: State of the Art, J Am Acad Orthop Surg. 2014;633-642.

Question 66
A 24-year-old man underwent closed reduction for an elbow dislocation 3 months ago. Now, he continues
to experience elbow discomfort and feelings of instability. When his forearm is supinated and a valgus
force is applied, his pain is recreated as the elbow is brought from flexion to extension. He also has
difficulty getting out of a chair with his forearms supinated. He has completed a course of physical
therapy without experiencing improvement. Which soft-tissue structure shown in Figure 66 requires
reconstruction?

1. A
2. B
3. C
4. D
5. E

PREFERRED RESPONSE: 5

RECOMMENDED READINGS
O’Driscoll SW, Bell DF, Morrey BF. Posterolateral rotatory instability of the elbow. J Bone Joint Surg
Am. 1991 Mar;73(3):440-6. PubMed PMID: 2002081.

Sanchez-Sotelo J, Morrey BF, O’Driscoll SW. Ligamentous repair and reconstruction for posterolateral
rotatory instability of the elbow. J Bone Joint Surg Br. 2005 Jan;87(1):54-61. PubMed PMID: 15686238.

© 2015 American Academy of Orthopaedic Surgeons 2015 Orthopaedic In-Training Examination


210 • American Academy of Orthopaedic Surgeons

Figure 85a Figure 85b

Question 85
Figures 85a and 85b are the radiographs of a 32-year-old man who had an arthroscopic superior labral
repair 5 months ago. Three-suture anchors were used to complete the repair, and all were placed through
an anterior rotator interval portal. He has severe posterior shoulder pain with limited overhead elevation
and “weakness.” Examination reveals active forward elevation limited to 120 degrees by pain, but he
has full passive range of motion, and the belly press test finding is within normal limits. The patient has
weakness of external rotation by the side. What is the most likely cause of his symptoms?

1. Rotator cuff tear


2. Adhesive capsulitis
3. Axillary nerve injury
4. Suprascapular nerve injury
5. Posttraumatic arthritis

PREFERRED RESPONSE: 4

RECOMMENDED READINGS
Morgan RT, Henn RF 3rd, Paryavi E, Dreese J. Injury to the suprascapular nerve during superior labrum
anterior and posterior repair: is a rotator interval portal safer than an anterosuperior portal? Arthroscopy.
2014 Nov;30(11):1418-23. doi: 10.1016/j.arthro.2014.06.006. Epub 2014 Aug 12. PubMed PMID:
25125380.

Koh KH, Park WH, Lim TK, Yoo JC. Medial perforation of the glenoid neck following SLAP repair
places the suprascapular nerve at risk: a cadaveric study. J Shoulder Elbow Surg. 2011 Mar;20(2):245-50.
doi: 10.1016/j.jse.2010.06.004. Epub 2010 Oct 16. PubMed PMID: 20951610.

© 2015 American Academy of Orthopaedic Surgeons 2015 Orthopaedic In-Training Examination


SECTION 8: Shoulder and Elbow • 211

Figure 89

Question 89
Figure 89 is the clinical photograph of a 42-year-old woman who has had gradual-onset atraumatic
elbow pain for 2 months. She has pain with daily activities and especially when her elbow is extended.
Her elbow is tender to palpation in the area shown in the photograph, but elbow radiograph findings are
normal. What is the best next step?

1. Elbow immobilization
2. Cortisone injection
3. Ligament reconstruction
4. Tendon repair
5. Tendon gliding exercises

PREFERRED RESPONSE: 5

RECOMMENDED READINGS
Coombes BK, Bisset L, Brooks P, Khan A, Vicenzino B. Effect of corticosteroid injection, physiotherapy,
or both on clinical outcomes in patients with unilateral lateral epicondylalgia: a randomized controlled
trial. JAMA. 2013 Feb 6;309(5):461-9. doi: 10.1001/jama.2013.129. PubMed PMID: 23385272.

Dines JS, Bedi A, Williams PN, Dodson CC, Ellenbecker TS, Altchek DW, Windler G, Dines DM. Tennis
Injuries: Epidemiology, Pathophysiology, and Treatment. J Am Acad Orthop Surg. 2015 Mar;23(3):181-
189. Epub 2015 Feb 9. Review. PubMed PMID: 25667400.

Boyer MI, Hastings H 2nd. Lateral tennis elbow: “Is there any science out there?” J Shoulder Elbow Surg.
1999 Sep-Oct;8(5):481-91. Review. PubMed PMID: 10543604.

© 2015 American Academy of Orthopaedic Surgeons 2015 Orthopaedic In-Training Examination


212 • American Academy of Orthopaedic Surgeons

Question 101
A 52-year-old otherwise healthy woman elects to undergo an arthroscopic right shoulder rotator cuff repair
with decompression in the beach chair position. She receives an uncomplicated interscalene regional block
using nerve stimulation localization that is supplemented with laryngeal mask airway. Twenty minutes into
surgery, the anesthesiologist informs you that the patient has become hypotensive (mean arterial pressure
40-60 mm Hg) and tachycardic despite repeated use of vasopressors. Decreased breath sounds are noted in
the upper lung fields on the right. In addition to halting the surgery, the most appropriate next step is a

1. CT angiogram.
2. venous duplex ultrasound.
3. portable chest radiograph.
4. critical care consultation.
5. transesophageal echocardiogram.

PREFERRED RESPONSE: 3

RECOMMENDED READINGS
Bryan NA, Swenson JD, Greis PE, Burks RT. Indwelling interscalene catheter use in an outpatient
setting for shoulder surgery: technique, efficacy, and complications. J Shoulder Elbow Surg. 2007 Jul-
Aug;16(4):388-95. Epub 2007 May 15. PubMed PMID: 1750724.

Mandim BL, Alves RR, Almeida R, Pontes JP, Arantes LJ, Morais FP. Pneumothorax post brachial plexus
block guided by ultrasound: a case report. Rev Bras Anestesiol. 2012 Sep-Oct;62(5):741-7. doi: 10.1016/
S0034-7094(12)70173-3. PubMed PMID: 22999407.

Lee HC, Dewan N, Crosby L. Subcutaneous emphysema, pneumomediastinum, and potentially life-
threatening tension pneumothorax. Pulmonary complications from arthroscopic shoulder decompression.
Chest. 1992 May;101(5):1265-7. PubMed PMID: 1582282.

© 2015 American Academy of Orthopaedic Surgeons 2015 Orthopaedic In-Training Examination


SECTION 8: Shoulder and Elbow • 213

Figure 116

Question 116
During arthroscopic evaluation of a 44-year-old mechanic’s shoulder, the pathology noted in Figure 116 is
seen. What is the best treatment option?

1. Labral repair
2. Biceps tenodesis
3. Rotator cuff repair
4. Rotator cuff debridement
5. Subacromial decompression

PREFERRED RESPONSE: 3

RECOMMENDED READINGS
Cordasco FA, Backer M, Craig EV, Klein D, Warren RF. The partial-thickness rotator cuff tear: is
acromioplasty without repair sufficient? Am J Sports Med. 2002 Mar-Apr;30(2):257-60. PubMed PMID:
11912097.

Payne LZ, Altchek DW, Craig EV, Warren RF. Arthroscopic treatment of partial rotator cuff tears in young
athletes. A preliminary report. Am J Sports Med. 1997 May-Jun;25(3):299-305. PubMed PMID: 9167807.

© 2015 American Academy of Orthopaedic Surgeons 2015 Orthopaedic In-Training Examination


214 • American Academy of Orthopaedic Surgeons

Video 131

Question 131
The examination finding demonstrated in Video 131 is consistent with a tear of which structure?

1. Infraspinatus
2. Subscapularis
3. Pectoralis major
4. Superior labrum
5. Long head of the biceps

PREFERRED RESPONSE: 1

RECOMMENDED READINGS
Castoldi F, Blonna D, Hertel R. External rotation lag sign revisited: accuracy for diagnosis of full
thickness supraspinatus tear. J Shoulder Elbow Surg. 2009 Jul-Aug;18(4):529-34. doi: 10.1016/j.
jse.2008.11.007. Epub 2009 Feb 14. PMID: 19223205.

Walch G, Boulahia A, Calderone S, Robinson AH. The ‘dropping’ and ‘hornblower’s’ signs in evaluation
of rotator-cuff tears. J Bone Joint Surg Br. 1998 Jul;80(4):624-8. PMID: 9699824.

© 2015 American Academy of Orthopaedic Surgeons 2015 Orthopaedic In-Training Examination


SECTION 8: Shoulder and Elbow • 215

Figure 149a Figure 149b Figure 149c

Figure 149d Figure 149e

Question 149
Figures 149a through 149e are the radiographs and MR images of a 78-year-old woman who has pain in
her left shoulder. The pain is worse at night and seems to “come and go”. She has functional restrictions
for simple activities of daily living. Her treatment to date has consisted of occasional acetaminophen,
which temporarily eases the pain. Examination reveals 140 degrees of overhead elevation and 30 degrees
of external rotation with a 10-degree external rotation lag. There is moderate pain at the extremes of
motion and weakness to resistance. What is the best next step?

1. Injection of corticosteroids followed by physical therapy


2. Trial of immobilization with referral to pain management
3. Reverse total shoulder arthroplasty with latissimus dorsi transfer
4. Arthroscopic capsular release with manipulation under anesthesia
5. Arthroscopic partial rotator cuff repair with concomitant interpositional allograft

PREFERRED RESPONSE: 1

© 2015 American Academy of Orthopaedic Surgeons 2015 Orthopaedic In-Training Examination


216 • American Academy of Orthopaedic Surgeons

RECOMMENDED READINGS
Laudicina L, D’Ambrosia R. Management of irreparable rotator cuff tears and glenohumeral arthritis.
Orthopedics. 2005 Apr;28(4):382-8; quiz 389-90. Review. PubMed PMID: 15887585.

Izquierdo R, Voloshin I, Edwards S, Freehill MQ, Stanwood W, Wiater JM, Watters WC 3rd, Goldberg
MJ, Keith M, Turkelson CM, Wies JL, Anderson S, Boyer K, Raymond L, Sluka P; American Academy
of Orthopaedic Surgeons. Treatment of glenohumeral osteoarthritis. J Am Acad Orthop Surg. 2010
Jun;18(6):375-82. PubMed PMID: 20511443.

Figure 167

Question 167
Figure 167 is an intraoperative view from the accessory anterolateral portal of a 31-year-old man who
underwent arthroscopic surgery on his right shoulder. The surgery was performed in the lateral decubitus
position. During the first 3 weeks after surgery, the repair will be protected by avoiding repetitive passive

1. abduction to 90 degrees in the scapular plane.


2. internal rotation with the shoulder adducted.
3. adduction with the shoulder flexed at shoulder level.
4. external rotation with the shoulder adducted.
5. external rotation to 45 degrees with the shoulder abducted.

PREFERRED RESPONSE: 3

RECOMMENDED READINGS
Kim SH, Ha KI, Park JH, Kim YM, Lee YS, Lee JY, Yoo JC. Arthroscopic posterior labral repair and
capsular shift for traumatic unidirectional recurrent posterior subluxation of the shoulder. J Bone Joint
Surg Am. 2003 Aug;85-A(8):1479-87. PubMed PMID: 12925627.

Millett PJ, Clavert P, Hatch GF 3rd, Warner JJ. Recurrent posterior shoulder instability. J Am Acad Orthop
Surg. 2006 Aug;14(8):464-76. Review. PubMed PMID: 16885478. 

© 2015 American Academy of Orthopaedic Surgeons 2015 Orthopaedic In-Training Examination


SECTION 8: Shoulder and Elbow • 217

Figure 178a Figure 178b

Figure 178c Figure 178d

Question 178
Figures 178a and 178b are the plain radiographs of a 36-year-old woman who has right elbow stiffness.
She sustained an injury to her right elbow 7 months ago. Injury radiographs are shown in Figures 178c
and 178d. She underwent a protracted course of occupational therapy that included use of a static splint.
Examination reveals elbow range of motion from 15 degrees of extension and flexion of 90 degrees. There
is no pain in the midrange of her motion arc, but there is pain at the extremes. During an open release of
her elbow to restore elbow flexion, release of which structure is necessary?

1. Posterior oblique band of the medial collateral ligament


2. Triceps tendon insertion
3. Lateral ulnar collateral ligament
4. Anterior capsule
5. Anterior band of the medial collateral ligament

PREFERRED RESPONSE: 1

© 2015 American Academy of Orthopaedic Surgeons 2015 Orthopaedic In-Training Examination


218 • American Academy of Orthopaedic Surgeons

RECOMMENDED READINGS
Morrey BF, An KN. Articular and ligamentous contributions to the stability of the elbow joint. Am J
Sports Med. 1983 Sep-Oct;11(5):315-9. PubMed PMID: 6638246.

Regan WD, Korinek SL, Morrey BF, An KN. Biomechanical study of ligaments around the elbow joint.
Clin Orthop Relat Res. 1991 Oct;(271):170-9. PubMed PMID: 1914292.

Wada T, Ishii S, Usui M, Miyano S. The medial approach for operative release of post-traumatic
contracture of the elbow. J Bone Joint Surg Br. 2000 Jan;82(1):68-73. PubMed PMID: 10697317. 

© 2015 American Academy of Orthopaedic Surgeons 2015 Orthopaedic In-Training Examination


SECTION 8: Shoulder and Elbow • 219

Figure 185a Figure 185b

Figure 185c Figure 185d

Question 185
Figures 185a through 185d are the radiographs and MR image of a 55-year-old man who had arthroscopic
repair of an acute, traumatic, massive 2-tendon rotator cuff tear involving the supraspinatus and
infraspinatus 4 months ago. He did well initially with formal physical therapy, but during the last month
has noted increasing shoulder pain and decreasing range of motion. Examination reveals he has active
forward elevation of about 100 degrees. He has well-preserved passive range of motion, weakness of
external rotation by his side, and positive impingement signs. Hornblower’s and belly press test findings
are normal, and an infection workup is negative. What is the best next step?

1. Revision rotator cuff repair


2. Continued physical therapy
3. Hemiarthroplasty
4. Latissimus dorsi transfer
5. Reverse total shoulder arthroplasty

PREFERRED RESPONSE: 1

© 2015 American Academy of Orthopaedic Surgeons 2015 Orthopaedic In-Training Examination


220 • American Academy of Orthopaedic Surgeons

RECOMMENDED READINGS
Denard PJ, Burkhart SS. Arthroscopic revision rotator cuff repair. J Am Acad Orthop Surg. 2011
Nov;19(11):657-66. Review. PubMed PMID: 22052642.

Lambers Heerspink FO, Dorrestijn O, van Raay JJ, Diercks RL. Specific patient-related prognostic
factors for rotator cuff repair: a systematic review. J Shoulder Elbow Surg. 2014 Jul;23(7):1073-80. doi:
10.1016/j.jse.2014.01.001. Epub 2014 Apr 13. Review. PubMed PMID: 24725900. 

Question 202
An active 65-year-old woman slipped on ice, fell down 2 steps, and injured her right shoulder. Prior to
the injury her shoulder “was normal,” and she denies any history of shoulder pain or injury. She sustained
a distal radius fracture 1 year ago after sustaining a simple mechanical fall, which was treated with cast
immobilization. Examination reveals that her skin is intact and the deltoid contracts well. Radiographs
demonstrate a displaced 4-part proximal humeral fracture with a fracture through the anatomic neck that
has a 2-mm posteromedial metaphyseal extension. The head fragment is in 90 degrees of varus. What is
the best treatment option?

1. Hemiarthroplasty
2. Percutaneous pinning
3. Unconstrained total shoulder arthroplasty
4. Open reduction and internal fixation (ORIF)
5. Sling immobilization and physical therapy

PREFERRED RESPONSE: 1

RECOMMENDED READINGS
Solberg BD, Moon CN, Franco DP, Paiement GD. Surgical treatment of three and four-part proximal
humeral fractures. J Bone Joint Surg Am. 2009 Jul;91(7):1689-97. doi: 10.2106/JBJS.H.00133. PubMed
PMID: 19571092.

Kontakis G, Koutras C, Tosounidis T, Giannoudis P. Early management of proximal humeral fractures


with hemiarthroplasty: a systematic review. J Bone Joint Surg Br. 2008 Nov;90(11):1407-13. doi:
10.1302/0301-620X.90B11.21070. Review. PubMed PMID: 18978256. 

© 2015 American Academy of Orthopaedic Surgeons 2015 Orthopaedic In-Training Examination


SECTION 8: Shoulder and Elbow • 221

Question 213
Which diagnosis most favorably influences implant survivorship among patients who undergo total elbow
arthroplasty?

1. Rheumatoid arthritis
2. Posttraumatic arthritis
3. Primary osteoarthritis
4. Distal humeral nonunion
5. Acute distal humeral fracture

PREFERRED RESPONSE: 1

RECOMMENDED READINGS
Aldridge JM 3rd, Lightdale NR, Mallon WJ, Coonrad RW. Total elbow arthroplasty with the Coonrad/
Coonrad-Morrey prosthesis. A 10- to 31-year survival analysis. J Bone Joint Surg Br. 2006 Apr;88(4):509-
14. PubMed PMID: 16567787.

Skyttä ET, Eskelinen A, Paavolainen P, Ikävalko M, Remes V. Total elbow arthroplasty in rheumatoid
arthritis: a population-based study from the Finnish Arthroplasty Register. Acta Orthop. 2009
Aug;80(4):472-7. doi: 10.3109/17453670903110642. PubMed PMID: 19562563; PubMed Central
PMCID: PMC2823192.

Cil A, Veillette CJ, Sanchez-Sotelo J, Morrey BF. Linked elbow replacement: a salvage procedure for
distal humeral nonunion. J Bone Joint Surg Am. 2008 Sep;90(9):1939-50. doi: 10.2106/JBJS.G.00690.
PubMed PMID: 18762655.

Schneeberger AG, Adams R, Morrey BF. Semiconstrained total elbow replacement for the treatment of
post-traumatic osteoarthrosis. J Bone Joint Surg Am. 1997 Aug;79(8):1211-22. PubMed PMID: 9278082.

Kozak TK, Adams RA, Morrey BF. Total elbow arthroplasty in primary osteoarthritis of the elbow. J
Arthroplasty 1998;13(7):837-842. PMID: 9802675.

Kamineni S, Morrey BF. Distal humeral fractures treated with noncustom total elbow replacement.
Surgical technique. J Bone Joint Surg Am 2005;87 Suppl 1(Pt 1):41-50. PMID:15743846.

© 2015 American Academy of Orthopaedic Surgeons 2015 Orthopaedic In-Training Examination


222 • American Academy of Orthopaedic Surgeons

Question 215
Under the Physician Payments Sunshine Act, what is the annual monetary value limit for items given to
physicians by orthopaedic implant companies? (These companies must report this type of activity to the
Centers for Medicare and Medicaid Services.)

1. $10
2. $25
3. $50
4. $100
5. $200

PREFERRED RESPONSE: 1

RECOMMEND READINGS
Peterson RN. What the sunshine law brings to light. AAOS NOW March 2013, page 1,

http://federalregister.gov/a/2013-02572 published 2/8/2014


Question 221
A 55-year-old woman has had pain in her left shoulder for 3 months with no preceding trauma.
Examination reveals substantial reduction in her left shoulder motion, both actively and passively. MR
imaging reveals space reduction in the axillary recess. Which comorbidity is associated with a much worse
prognosis and poorer outcomes of both nonsurgical and surgical treatment?

1. Diabetes mellitus
2. Crohn’s disease
3. Renal disease
4. Congestive obstructive pulmonary disease
5. Breast cancer

PREFERRED RESPONSE: 1

RECOMMENDED READINGS
Neviaser AS, Neviaser RJ. Adhesive capsulitis of the shoulder. J Am Acad Orthop Surg. 2011
Sep;19(9):536-42. Review. PubMed PMID: 21885699.

Arkkila PE, Kantola IM, Viikari JS, Rönnemaa T. Shoulder capsulitis in type I and II diabetic patients:
association with diabetic complications and related diseases. Ann Rheum Dis. 1996 Dec;55(12):907-14.
PubMed PMID: 9014585; PubMed Central PMCID: PMC1010343.

© 2015 American Academy of Orthopaedic Surgeons 2015 Orthopaedic In-Training Examination


SECTION 8: Shoulder and Elbow • 223

Question 234
A 55-year-old woman had successful reduction of an anterior shoulder dislocation 2 weeks ago.
Examination reveals she is unable to actively elevate her arm. Her passive range of motion is equal to
uninjured shoulder motion. What is the function of the structure that is most likely injured?

1. Scapular protraction and inferior rotation


2. Scapular elevation and superior rotation
3. Flexion and supination of the forearm
4. Extension of the humerus
5. Compression of the humeral head against the glenoid

PREFERRED RESPONSE: 5

RECOMMENDED READINGS
Neviaser RJ, Neviaser TJ, Neviaser JS. Concurrent rupture of the rotator cuff and anterior dislocation
of the shoulder in the older patient. J Bone Joint Surg Am. 1988 Oct;70(9):1308-11. PubMed PMID:
3182884.

Jouve F, Graveleau N, Nové-Josserand L, Walch G. [Recurrent anterior instability of the shoulder


associated with full thickness rotator cuff tear: results of surgical treatment]. Rev Chir Orthop Reparatrice
Appar Mot. 2008 Nov;94(7):659-69. doi: 10.1016/j.rco.2008.03.032. Epub 2008 May 12. French. PubMed
PMID: 18984123.

© 2015 American Academy of Orthopaedic Surgeons 2015 Orthopaedic In-Training Examination


224 • American Academy of Orthopaedic Surgeons

Question 242
A 45-year-old man has a painful shoulder after a fall and difficulty elevating his arm above waist level.
Shoulder radiograph findings are normal. His belly press sign is positive, but there is no external rotation
lag. His deltoid contracts strongly. Which other examination finding would most likely be positive?

1. Neer test
2. Lift-off test
3. Apprehension test
4. Hornblower’s test
5. Increased passive internal rotation

PREFERRED RESPONSE: 2

RECOMMENDED READINGS
Gerber C, Krushell RJ. Isolated rupture of the tendon of the subscapularis muscle. Clinical features in 16
cases. J Bone Joint Surg Br. 1991 May;73(3):389-94. PubMed PMID: 1670434.

Barth J, Audebert S, Toussaint B, Charousset C, Godeneche A, Graveleau N, Joudet T, Lefebvre Y, Nove-


Josserand L, Petroff E, Solignac N, Scymanski C, Pitermann M, Thelu CE; French Arthroscopy Society.
Diagnosis of subscapularis tendon tears: are available diagnostic tests pertinent for a positive diagnosis?
Orthop Traumatol Surg Res. 2012 Dec;98(8 Suppl):S178-85. doi:10.1016/j.otsr.2012.09.008. Epub 2012
Nov 9. PubMed PMID: 23142297.

Question 253
What is the most likely risk factor for recurrent shoulder instability after arthroscopic anterior shoulder
stabilization?

1. Female gender
2. Athletic throwing activity
3. Age older than 20 years
4. Unidirectional instability
5. Attritional glenoid bone loss

PREFERRED RESPONSE: 5

RECOMMENDED READINGS
Ahmed I, Ashton F, Robinson CM. Arthroscopic Bankart repair and capsular shift for recurrent anterior
shoulder instability: functional outcomes and identification of risk factors for recurrence. J Bone Joint
Surg Am. 2012 Jul 18;94(14):1308-15. doi: 10.2106/JBJS.J.01983. PubMed PMID: 22810402.

Balg F, Boileau P. The instability severity index score. A simple pre-operative score to select patients
for arthroscopic or open shoulder stabilisation. J Bone Joint Surg Br. 2007 Nov;89(11):1470-7. PubMed
PMID: 17998184.

© 2015 American Academy of Orthopaedic Surgeons 2015 Orthopaedic In-Training Examination
SECTION 8: Shoulder and Elbow • 225

Question 272
When performing a primary anatomic total shoulder arthroplasty for glenohumeral osteoarthritis, selecting
an appropriately sized humeral head component is important to avoid either overstuffing the joint or
leaving it too lax. Which relationship between thickness of the humeral head and radius of the curvature
most closely replicates normal anatomy?

1. Approximately 30% radius of curvature


2. Approximately 40% radius of curvature
3. Approximately 50% radius of curvature
4. Approximately 70% radius of curvature
5. Approximately 90% radius of curvature

PREFERRED RESPONSE: 4

RECOMMENDED READINGS
Boileau P, Walch G. The three-dimensional geometry of the proximal humerus. Implications for surgical
technique and prosthetic design. J Bone Joint Surg Br. 1997 Sep;79(5):857-65. PubMed PMID: 9331050.

Robertson DD, Yuan J, Bigliani LU, Flatow EL, Yamaguchi K. Three-dimensional analysis
of the proximal part of the humerus: relevance to arthroplasty. J Bone Joint Surg Am. 2000
Nov;82-A(11):1594-602. PubMed PMID: 11097450.

© 2015 American Academy of Orthopaedic Surgeons 2015 Orthopaedic In-Training Examination


226 • American Academy of Orthopaedic Surgeons

SECTION 9: Oncology

Figure 4a Figure 4b Figure 4c

Figure 4d Figure 4e

Question 4
A 16-year-old boy has had progressive leg pain for 6 weeks. Radiographs and a gadolinium-enhanced MR
image are shown in Figures 4a through 4c. Figures 4d and 4e reveal low- and high-power pathology. What
is the best next step?

1. Wide resection
2. Wide resection and radiotherapy
3. Neoadjuvant chemotherapy, wide resection, and radiotherapy
4. Neoadjuvant chemotherapy, wide resection, and adjuvant chemotherapy
5. Curetting and bone grafting

PREFERRED RESPONSE: 4
© 2015 American Academy of Orthopaedic Surgeons 2015 Orthopaedic In-Training Examination
SECTION 9: Oncology • 227

RECOMMENDED READINGS
Chou AJ, Malek F. Osteosarcoma of bone. In: Biermann JS, ed. Orthopaedic Knowledge Update:
Musculoskeletal Tumors 3. Rosemont, IL: American Academy of Orthopaedic Surgeons; 2014: 159-170.

Messerschmitt PJ, Garcia RM, Abdul-Karim FW, Greenfield EM, Getty PJ. Osteosarcoma. J Am Acad
Orthop Surg. 2009 Aug;17(8):515-27. Review. PubMed PMID: 19652033. 

© 2015 American Academy of Orthopaedic Surgeons 2015 Orthopaedic In-Training Examination


228 • American Academy of Orthopaedic Surgeons

Figure 17a Figure 17b Figure 17c

Figure 17d Figure 17e

Question 17
A 35-year-old man has had a painless mass for 4 months. A radiograph and T1-weighted, T2-weighted,
and contrast-enhanced axial MR images are shown in Figures 17a through 17d. Histopathology is shown
in Figure 17e. Treatment recommendations should include

1. clinical and radiologic surveillance.


2. marginal excision.
3. wide excision.
4. wide excision and radiotherapy.
5. radiotherapy.

PREFERRED RESPONSE: 4

© 2015 American Academy of Orthopaedic Surgeons 2015 Orthopaedic In-Training Examination


SECTION 9: Oncology • 229

RECOMMENDED READINGS
Esther RJ. Soft-tissue sarcomas. In: Biermann JS, ed. Orthopaedic Knowledge Update: Musculoskeletal
Tumors 3. Rosemont, IL: American Academy of Orthopaedic Surgeons; 2014: 295-306.

Bird JE. Surgical management of soft-tissue sarcomas. In: Biermann JS, ed. Orthopaedic Knowledge
Update: Musculoskeletal Tumors 3. Rosemont, IL: American Academy of Orthopaedic Surgeons; 2014:
307-315.

Gilbert NF, Cannon CP, Lin PP, Lewis VO. Soft-tissue sarcoma. J Am Acad Orthop Surg. 2009
Jan;17(1):40-7. Review. PubMed PMID: 19136426. 

Figure 25a Figure 25b Figure 25c

Question 25
Figures 25a through 25c are the anteroposterior and lateral radiographs and high-power histopathology
of a 26-year-old man who has had a progressive growth on his nondominant wrist for 3 months. Which
additional study should be recommended in the evaluation of this patient?

1. Positron emission tomography (PET) scan


2. Whole-body bone scan
3. Thin-cut (2-mm) CT scan
4. Chest radiograph
5. CT of the chest, abdomen, and pelvis

PREFERRED RESPONSE: 4

© 2015 American Academy of Orthopaedic Surgeons 2015 Orthopaedic In-Training Examination


230 • American Academy of Orthopaedic Surgeons

RECOMMENDED READINGS
Cheong D, Letson GD. Giant cell tumor of bone. In: Biermann JS, ed. Orthopaedic Knowledge Update:
Musculoskeletal Tumors 3. Rosemont, IL: American Academy of Orthopaedic Surgeons; 2014: 133-146.

Raskin KA, Schwab JH, Mankin HJ, Springfield DS, Hornicek FJ. Giant cell tumor of bone. J Am
Acad Orthop Surg. 2013 Feb;21(2):118-26. doi: 10.5435/JAAOS-21-02-118. Review. PubMed PMID:
23378375. 

Figure 37a Figure 37b Figure 37c

Question 37
Figures 37a through 37c are the MR images of an enlarged thigh and high-power pathology of a 55-year-
old woman who noted asymmetry of her thighs. A positive result for which additional pathologic study
will help to determine the definitive pathologic diagnosis?

1. t(X; 18)
2. t(11;22)
3. HMB-45
4. MDM2 amplification
5. Mutation in the neurofibromin 2 (NF2) gene

PREFERRED RESPONSE: 4

RECOMMENDED READINGS
McGarry SV. Lipoma and other benign lipomatous tumors. In: Biermann JS, ed. Orthopaedic Knowledge
Update: Musculoskeletal Tumors 3. Rosemont, IL: American Academy of Orthopaedic Surgeons; 2014:
257-261.

Weiss SW, Golsblum JR. Enzinger and Weiss’s Soft Tissue Tumors. 5th ed. Philadelphia, PA: Elsevier;
2008.

Jones KB. The molecular biology of musculoskeletal neoplasia. In: Biermann JS, ed. Orthopaedic
Knowledge Update: Musculoskeletal Tumors 3. Rosemont, IL: American Academy of Orthopaedic
Surgeons; 2014: 31-38.
© 2015 American Academy of Orthopaedic Surgeons 2015 Orthopaedic In-Training Examination
SECTION 9: Oncology • 231

Figure 47a Figure 47b

Question 47 (Item Deleted)


Figures 47a and 47b are the MR images of a 50-year-old man who has a slow-growing mass in his right
thigh that was removed in a piecemeal manner and subsequently diagnosed as an intermediate-grade
liposarcoma. The figures show the mass prior to surgical excision. A CT scan of his chest, abdomen, and
pelvis reveals no evidence of metastatic disease. What is the best next step?

1. Clinical and radiologic surveillance every 3 months


2. MR imaging of the extremity
3. Tumor bed excision
4. Transfemoral amputation
5. Radiation

PREFERRED RESPONSE: 2

RECOMMENDED READINGS
Umer HM, Umer M, Qadir I, Abbasi N, Masood N. Impact of unplanned excision on prognosis of patients
with extremity soft tissue sarcoma. Sarcoma. 2013;2013:498604. doi: 10.1155/2013/498604. Epub 2013
Apr 30. PubMed PMID: 23737702; PubMed Central PMCID: PMC3659434.

Potter BK, Adams SC, Pitcher JD Jr, Temple HT. Local recurrence of disease after unplanned excisions
of high-grade soft tissue sarcomas. Clin Orthop Relat Res. 2008 Dec;466(12):3093-100. doi: 10.1007/
s11999-008-0529-4. Epub 2008 Sep 26. PubMed PMID: 18818981; PubMed Central PMCID:
PMC2628223. 

© 2015 American Academy of Orthopaedic Surgeons 2015 Orthopaedic In-Training Examination


232 • American Academy of Orthopaedic Surgeons

Question 60
Radiographs, MR imaging, and needle biopsy of the right distal femur were used to diagnose a high-grade
osteosarcoma in a 12-year-old girl. In addition to a chest CT scan, comprehensive staging and initial
treatment should include a

1. whole-body bone scan and referral to pediatric oncology.


2. whole-body bone scan and referral to pediatric oncology and radiation oncology.
3. whole-body bone scan, bone marrow biopsy, and referral to pediatric oncology and
radiation oncology.
4. CT scan of the abdomen and pelvis, whole-body bone scan, and referral to pediatric oncology.
5. CT scan of the abdomen and pelvis, whole-body bone scan, bone marrow biopsy, and referral
to pediatric oncology.

PREFERRED RESPONSE: 1

RECOMMENDED READINGS
Messerschmitt PJ, Garcia RM, Abdul-Karim FW, Greenfield EM, Getty PJ. Osteosarcoma. J Am Acad
Orthop Surg. 2009 Aug;17(8):515-27. Review. PubMed PMID: 19652033.

Chou AJ, Malek F. Osteosarcoma of bone. In: Biermann JS, ed. Orthopaedic Knowledge Update 3:
Musculoskeletal Tumors. Rosemont, IL: American Academy of Orthopaedic Surgeons; 2014:159-170.

© 2015 American Academy of Orthopaedic Surgeons 2015 Orthopaedic In-Training Examination


SECTION 9: Oncology • 233

Figure 71a Figure 71b

Question 71
Figures 71a and 71b are the radiographs of a 73-year-old woman with a history of nonmetastatic breast
cancer and ambulatory thigh pain for 1 month. A whole-body bone scan reveals radiotracer uptake
localized to the proximal femur. A needle biopsy confirms metastatic breast carcinoma. What is the most
appropriate treatment strategy?

1. Radiation
2. Prophylactic internal fixation
3. Prophylactic internal fixation and radiation
4. Proximal femur resection
5. Proximal femur resection and radiation

PREFERRED RESPONSE: 3
-
RECOMMENDED READINGS
Bickels J, Dadia S, Lidar Z. Surgical management of metastatic bone disease. J Bone Joint Surg Am. 2009
Jun;91(6):1503-16. doi: 10.2106/JBJS.H.00175. Review. PubMed PMID: 19487532.

Beebe KS: Prediction of impending pathologic fractures and treatment considerations in patients with
metastatic bone disease. In: Biermann JS, ed. Orthopaedic Knowledge Update 3: Musculoskeletal Tumors.
Rosemont, IL: American Academy of Orthopaedic Surgeons; 2014:331-338.

Goodman HJ, Patterson FR. Surgical management of lower extremity metastatic disease. In: Biermann
JS, ed. Orthopaedic Knowledge Update 3: Musculoskeletal Tumors. Rosemont, IL: American Academy of
Orthopaedic Surgeons; 2014:345-353.

© 2015 American Academy of Orthopaedic Surgeons 2015 Orthopaedic In-Training Examination


234 • American Academy of Orthopaedic Surgeons

Question 79
A 70-year-old woman with a history of breast cancer has left groin pain with ambulation. Radiographs
reveal a destructive lesion in her left proximal femur without fracture. A bone scan reveals a solitary lesion
in the left femoral neck. Following a thorough history and examination, the best next step is

1. referral to radiation oncology for radiotherapy.


2. surgical prophylaxis.
3. surgical prophylaxis followed by radiotherapy.
4. laboratory evaluation (complete blood count [CBC], chemistry panel, urinalysis) followed by
surgical prophylaxis.
5. laboratory evaluation (CBC, chemistry panel, and urinalysis); CT scan of the chest, abdomen,
and pelvis; and biopsy of the femoral lesion.

PREFERRED RESPONSE: 5

RECOMMENDED READINGS
Weber KL. Evaluation of the adult patient (aged >40 years) with a destructivebone lesion. J Am Acad
Orthop Surg. 2010 Mar;18(3):169-79. Review. PubMedPMID: 20190107.

Rougraff BT. Evaluation of the patient with carcinoma of unknown originmetastatic to bone. Clin Orthop
Relat Res. 2003 Oct;(415 Suppl):S105-9.Review. PubMed PMID: 14600599.

Beebe KS: Prediction of impending pathologic fractures and treatment considerations in patients with
metastatic bone disease. In: Biermann JS, ed. Orthopaedic Knowledge Update 3: Musculoskeletal Tumors.
Rosemont, IL: American Academy of Orthopaedic Surgeons; 2014:331-338.

© 2015 American Academy of Orthopaedic Surgeons 2015 Orthopaedic In-Training Examination


SECTION 9: Oncology • 235

Figure 90

Question 90
Figure 90 is the biopsy specimen of a 30-year-old man who has a 4-month history of a painless,
slow-growing mass in his medial thigh. Local imaging (orthogonal radiographs and MR images with
intravenous gadolinium) reveals a 6- x 5- x 7-cm heterogeneous mass in the adductor magnus muscle.
Additional analysis confirms the presence of translocation t(X; 18). What are the appropriate staging steps
to establish a diagnosis?

1. Chest CT scan and whole-body bone scan


2. Chest radiographs and chest CT scan
3. Chest radiographs, chest CT scan, and whole-body bone scan
4. Chest radiographs, chest CT scan, and examination of locoregional lymph nodes
5. Chest radiographs, chest CT scan, whole-body bone scan, and examination of locoregional
lymph nodes

PREFERRED RESPONSE: 4

RECOMMENDED READINGS
Gilbert NF, Cannon CP, Lin PP, Lewis VO. Soft-tissue sarcoma. J Am Acad Orthop Surg. 2009
Jan;17(1):40-7. Review. PubMed PMID: 19136426.

Esther RJ. Soft tissue sarcomas. In: Biermann JS, ed. Orthopaedic Knowledge Update 3: Musculoskeletal
Tumors. Rosemont, IL: American Academy of Orthopaedic Surgeons; 2014:295-306.

© 2015 American Academy of Orthopaedic Surgeons 2015 Orthopaedic In-Training Examination


236 • American Academy of Orthopaedic Surgeons

Figure 109

Question 109
Figure 109 is the postsurgical radiograph of a 60-year-old woman who sustained a right pertrochanteric
fracture through a solitary osseous metastasis of renal cell cancer origin. She is a community ambulatory
who fell as the result of an episode of dizziness that has been attributed to multiple small-brain metastases.
She was treated with proximal femoral resection and the reconstruction shown in the figure through a
posterolateral approach. What is the most appropriate immediate rehabilitation plan?

1. Nonweight-bearing activity with walker assistance and posterior hip precautions


2. Toe-touch weight bearing with cane assistance and posterior hip precautions
3. Toe-touch weight bearing with cane assistance and posterior and anterior hip precautions
4. Full weight bearing with walker assistance and posterior hip precautions
5. Full weight bearing with cane assistance and anterior and posterior hip precautions

PREFERRED RESPONSE: 4

RECOMMENDED READINGS
Faruqui SR, Jaeblon T. Ambulatory assistive devices in orthopaedics: uses and modifications. J Am Acad
Orthop Surg. 1 2010 Jan;18(1):41-50. Review. PubMed PMID: 20044491.

Goodman HJ, Patterson FR: Surgical management of lower extremity metastatic disease. In: Biermann
JS, ed. Orthopaedic Knowledge Update 3: Musculoskeletal Tumors. Rosemont, IL: American Academy of
Orthopaedic Surgeons; 2014:345-353.

© 2015 American Academy of Orthopaedic Surgeons 2015 Orthopaedic In-Training Examination


SECTION 9: Oncology • 237

Question 119
A 65-year-old woman sustained a pathologic hip fracture through a femoral neck lesion. She has breast
cancer and multifocal metastatic bone disease. In addition to estrogen receptor status, which other
pathologic order(s) will assist your medical oncologist in planning this patient’s treatment?

1. Human epidermal growth factor receptor 2 (HER2)/neu receptor status only


2. HER2/neu and vascular endothelial growth factor (VEGF) receptors
3. Progesterone receptor status only
4. Progesterone and HER2/neu receptors
5. Progesterone, VEGF, and HER2/neu receptors

PREFERRED RESPONSE: 4

RECOMMENDED READINGS
Quinn RH, Rajani R. Disease-specific considerations in metastatic bone disease. In: Biermann JS, ed.
Orthopaedic Knowledge Update: Musculoskeletal Tumors 3. Rosemont, IL: American Academy of
Orthopaedic Surgeons; 2014: 365-369.

Buzdar AU. Role of biologic therapy and chemotherapy in hormone receptor- and HER2-positive breast
cancer. Ann Oncol. 2009 Jun;20(6):993-9. doi: 10.1093/annonc/mdn739. Epub 2009 Jan 15. Review.
PubMed PMID: 19150946.

Question 127
Several cancers directly produce receptor activator of nuclear factor kappa beta ligand (RANKL), an
important molecule in

1. osteoclastogenesis.
2. osteoblastogenesis.
3. stem cell differentiation.
4. interstitial macrophages.
5. osteopontin production.

PREFERRED RESPONSE: 1

RECOMMENDED READINGS
Fitzhugh VA. The pathophysiology of bone metastasis. In: Biermann JS, ed. Orthopaedic Knowledge
Update 3: Musculoskeletal Tumors. Rosemont, IL: American Academy of Orthopaedic Surgeons;
2014:319-329.

Lynch CC. Matrix metalloproteinases as master regulators of the vicious cycle of bone metastasis. Bone.
2011 Jan;48(1):44-53. doi: 10.1016/j.bone.2010.06.007. Epub 2010 Jun 16. Review. PubMed PMID:
20601294.

© 2015 American Academy of Orthopaedic Surgeons 2015 Orthopaedic In-Training Examination


238 • American Academy of Orthopaedic Surgeons

Figure 151a Figure 151b

Question 151
Figures 151a and 151b are the short tau inversion recovery and postcontrast MR images of a 27-year-old
man who has mild, progressive pain and swelling overlying his upper thoracic spine. What is the best next
step?

1. Biopsy
2. Wide resection
3. Marginal resection
4. Radiation therapy
5. Repeat imaging in 3 months

PREFERRED RESPONSE: 1

RECOMMENDED READINGS
Damron TA, Beauchamp CP, Rougraff BT, Ward WG Sr. Soft-tissue lumps and bumps. Instr Course Lect.
2004;53:625-37. Review. PubMed PMID: 15116652.

Escobar C, Munker R, Thomas JO, Li BD, Burton GV. Update on desmoid tumors. Ann Oncol. 2012
Mar;23(3):562-9. doi: 10.1093/annonc/mdr386. Epub 2011 Aug 22.

© 2015 American Academy of Orthopaedic Surgeons 2015 Orthopaedic In-Training Examination


SECTION 9: Oncology • 239

Question 156
Radiotherapy and chemotherapy may be used as part of definitive treatment for

1. osteosarcoma.
2. Ewing sarcoma.
3. chondrosarcoma.
4. adamantinoma.
5. schwannoma.

PREFERRED RESPONSE: 2

RECOMMENDED READINGS
Baldini EH. Radiation therapy. In: Biermann JS, ed. Orthopaedic Knowledge Update 3: Musculoskeletal
Tumors. Rosemont, IL: American Academy of Orthopaedic Surgeons; 2014:77-84.

Wright J. Chemotherapy. In: Biermann JS, ed. Orthopaedic Knowledge Update 3: Musculoskeletal
Tumors. Rosemont, IL: American Academy of Orthopaedic Surgeons; 2014:53-63.

Question 172
Painful symptoms caused by osteoid osteomas are mediated by

1. Cyclooxygenase-2 (COX-2).
2. endorphins.
3. prostaglandin E2.
4. prostaglandin H2.
5. prostaglandin I2.

PREFERRED RESPONSE: 3

RECOMMENDED READINGS
Henshaw RM, Carmody Soni EE. Benign bone-forming tumors. In: Biermann JS, ed. Orthopaedic
Knowledge Update 3: Musculoskeletal Tumors. Rosemont, IL: American Academy of Orthopaedic
Surgeons; 2014:107-121.

Makley JT, Dunn MJ. Prostaglandin synthesis by osteoid osteoma. Lancet. 1982 Jul 3;2(8288):42.
PubMed PMID: 6123769. 

© 2015 American Academy of Orthopaedic Surgeons 2015 Orthopaedic In-Training Examination


240 • American Academy of Orthopaedic Surgeons

Question 187
Osteogenesis imperfecta is a metabolic bone disease characterized by which inheritance pattern(s)?

1. Autosomal dominant and autosomal recessive


2. X-linked recessive
3. Autosomal recessive and X-linked recessive
4. Autosomal dominant
5. Autosomal dominant and X-linked recessive

PREFERRED RESPONSE: 1

RECOMMENDED READINGS
Burnei G, Vlad C, Georgescu I, Gavriliu TS, Dan D. Osteogenesis imperfecta:diagnosis and treatment. J
Am Acad Orthop Surg. 2008 Jun;16(6):356-66.Review. PubMed PMID: 18524987.

Krakow D. Skeletal dysplasias. Clin Perinatol. 2015 Jun;42(2):301-19, viii. doi: 10.1016/j.
clp.2015.03.003. Epub 2015 Apr 8. Review. PubMed PMID: 26042906; PubMed Central PMCID:
PMC4456691.

Harrington J, Sochett E, Howard A. Update on the evaluation and treatment of osteogenesis imperfecta.
Pediatr Clin North Am. 2014 Dec;61(6):1243-57. doi: 10.1016/j.pcl.2014.08.010. Epub 2014 Sep 22.
Review. PubMed PMID: 25439022. 

Question 217
A bone tumor staged as IIA is defined by the Enneking Surgical Staging System as

1. low-grade intracompartmental.
2. low-grade extracompartmental.
3. high-grade intracompartmental.
4. high-grade extracompartmental.
5. high-grade regional or distant metastasis.

PREFERRED RESPONSE: 3

RECOMMENDED READINGS
Simon AE. Staging systems. In: Simon MA, Springfield D, eds. Surgery for Bone and Soft-Tissue Tumors.
Philadelphia, PA: Lippincott-Raven; 1998: 47-50.

Cummings JE, Seidel MJ. Clinical presentations and staging of bone tumors. In: Biermann JS, ed.
Orthopaedic Knowledge Update 3: Musculoskeletal Tumors. Rosemont, IL: American Academy of
Orthopaedic Surgeons; 2014: 3-11.

Enneking WF, Spanier SS, Goodman MA. A system for the surgical staging of musculoskeletal sarcoma.
Clin Orthop Relat Res. 1980 Nov-Dec;(153):106-20. PubMed PMID: 7449206. 
© 2015 American Academy of Orthopaedic Surgeons 2015 Orthopaedic In-Training Examination
SECTION 9: Oncology • 241

Question 241
Giant-cell tumor of bone (GCTB) is most commonly seen in which structure?

1. Distal femur
2. Distal radius
3. Proximal tibia
4. Proximal femur
5. Proximal humerus

PREFERRED RESPONSE: 1

RECOMMENDED READINGS
Cheong D, Letson GD. Giant cell tumor of bone. In: Biermann JS, ed. Orthopaedic Knowledge Update 3:
Musculoskeletal Tumors. Rosemont, IL: American Academy of Orthopaedic Surgeons; 2014:133-146.

McDonald DJ, Weber KL. Giant cell tumor of bone. In: Schwartz HS, ed. Orthopaedic Knowledge Update
2: Musculoskeletal Tumors. Rosemont, IL: American Academy of Orthopaedic Surgeons; 2007:134-135.

Question 254
Local recurrence of primary malignant bone tumors is directly related to

1. tumor size.
2. surgical margin.
3. histologic grade.
4. anatomic location.
5. neoadjuvant therapy.

PREFERRED RESPONSE: 2

RECOMMENDED READINGS
Kawaguchi S, Lewis VO. Surgical management of malignant primary bone tumors. In: Biermann JS,
ed. Orthopaedic Knowledge Update 3: Musculoskeletal Tumors. Rosemont, IL: American Academy of
Orthopaedic Surgeons; 2014:219-230.

Enneking WF. Surgical procedures. In: Enneking WF, ed. Musculoskeletal Tumor Surgery. New York, NY:
Churchill Livingstone; 1983:99.

© 2015 American Academy of Orthopaedic Surgeons 2015 Orthopaedic In-Training Examination


242 • American Academy of Orthopaedic Surgeons

Figure 257a Figure 257b

Question 257
Figures 257a and 257b are the radiographs of a 23-year-old man who has multiple bony masses that are
intermittently symptomatic. Which gene is most likely associated with his condition?

1. NF1
2. SYT
3. EWS
4. EXT1
5. COL1A

PREFERRED RESPONSE: 4

RECOMMENDED READINGS
Muthusamy S, Conway SA, Temple HT. Five polyostotic conditions that general orthopedic surgeons
should recognize (or should not miss). Orthop Clin North Am. 2014 Jul;45(3):417-29. doi: 10.1016/j.
ocl.2014.04.004. Review. PubMed PMID: 24975767.

Steffner R. Benign bone tumors. Cancer Treat Res. 2014;162:31-63. doi: 10.1007/978-3-319-07323-1_3.
Review. PubMed PMID: 25070230.

© 2015 American Academy of Orthopaedic Surgeons 2015 Orthopaedic In-Training Examination


SECTION 9: Oncology • 243

Figure 262a Figure 262b

Figure 262c Figure 262d

Question 262
Figures 262a through 262d are the radiographs and MR images of a 10-year-old girl who has activity-
related bilateral knee pain. Examination reveals diffuse anterior knee tenderness. What is the best next step
in treatment of this lesion?

1. Biopsy
2. Observation
3. CT scan of the chest
4. Curettage and grafting
5. Radiofrequency ablation

PREFERRED RESPONSE: 2

© 2015 American Academy of Orthopaedic Surgeons 2015 Orthopaedic In-Training Examination


244 • American Academy of Orthopaedic Surgeons

RECOMMENDED READINGS
Steffner R. Benign bone tumors. Cancer Treat Res. 2014;162:31-63. doi: 10.1007/978-3-319-07323-1_3.
Review. PubMed PMID: 25070230.

Smith SE, Kransdorf MJ. Primary musculoskeletal tumors of fibrous origin. Semin Musculoskelet Radiol.
2000;4(1):73-88. Review. PubMed PMID: 11061693.

© 2015 American Academy of Orthopaedic Surgeons 2015 Orthopaedic In-Training Examination


SECTION 9: Oncology • 245

Figure 270a Figure 270b Figure 270c

Figure 270d Figure 270e

Question 270
Figures 270a through 270e are the radiographs, MR image, and CT scans of a 19-year-old woman who has
had progressive right hip pain for 1 year. She denies any antecedent trauma or inciting event. Examination
reveals diffuse tenderness around the hip and pain with attempted range of motion. What is the best next
step?

1. Biopsy
2. CT scan of the chest
3. Antibiotic treatment
4. Radiofrequency ablation
5. Protected weight bearing

PREFERRED RESPONSE: 4

RECOMMENDED READINGS
Gibbs CP, Lewis VO, Peabody T. Beyond bone grafting: techniques in the surgical management of benign
bone tumors. Instr Course Lect. 2005;54:497-503. Review. PubMed PMID: 15948474

Bourgault C, Vervoort T, Szymanski C, Chastanet P, Maynou C. Percutaneous CT-guided radiofrequency


thermocoagulation in the treatment of osteoid osteoma: a 87 patient series. Orthop Traumatol Surg Res.
2014 May;100(3):323-7. doi: 10.1016/j.otsr.2014.02.001. Epub 2014 Mar 25. PubMed PMID: 24679368.

© 2015 American Academy of Orthopaedic Surgeons 2015 Orthopaedic In-Training Examination
246 • American Academy of Orthopaedic Surgeons

Figure 273a Figure 273b Figure 273c Figure 273d

Figure 273e Figure 273f

Question 273
Figures 273a through 273f are the radiographs, MR images, and low- and high-power biopsy specimens
of a 15-year-old girl who has progressive knee pain and a limp. The pain began insidiously and has
progressed to pain at night. Examination reveals diffuse tenderness around the knee but no mass. What is
the best next step?

1. Embolization
2. Chemotherapy
3. Wide resection
4. Radiation therapy
5. Curettage and grafting

PREFERRED RESPONSE: 2

RECOMMENDED READINGS
Lewis VO, Morris CD, Parsons TW 3rd. Malignant and benign bone tumors that you are likely to see.
Instr Course Lect. 2013;62:535-49. PubMed PMID: 23395056.

Weber K, Damron TA, Frassica FJ, Sim FH. Malignant bone tumors. Instr Course Lect. 2008;57:673-88.
Review. PubMed PMID: 18399615. 
© 2015 American Academy of Orthopaedic Surgeons 2015 Orthopaedic In-Training Examination
SECTION 10: Basic Science • 247

SECTION 10: Basic Science

Question 9
Increased levels of peroxisome proliferator-activated receptor gamma 2 (PPARy2) result in increased

1. bone.
2. adipose.
3. cartilage.
4. muscle.
5. tendon or ligament.

PREFERRED RESPONSE: 2

RECOMMENDED READINGS
Lee FY, Drissi MH, Zuscik MJ, Chen D, Nizami S, Goto H. Molecular and cell biology in orthopaedics.
In: O’Keefe RJ, Jacobs JJ, Chu CR, Einhorn TA, eds. Orthopaedic Basic Science Foundations of Clinical
Practice. 4th ed. Rosemont, IL: American Academy of Orthopaedic Surgeons; 2013:3-42.

Takada I, Suzawa M, Matsumoto K, Kato S. Suppression of PPAR transactivation switches cell fate of
bone marrow stem cells from adipocytes into osteoblasts. Ann N Y Acad Sci. 2007 Nov;1116:182-95.
Epub 2007 Jul 26. Review. PubMed PMID: 17656564. 

Question 22
What is the mechanism of the anticoagulation effect of heparin?

1. Inhibition of Factor Xa
2. Activation of antithrombin (AT) III
3. Degradation of serine proteases
4. Interference with Factors Va and VIIIa
5. Blocking of vitamin K epoxide reductase (VKOR)

PREFERRED RESPONSE: 2

RECOMMENDED READINGS
Pellegrini VD. Thrombolic disease and fat embolism syndrome. In: O’Keefe RJ, Jacobs JJ, Chu CR,
Einhorn TA. eds. Orthopaedic Basic Science: Foundations of Clinical Practice. 4th ed. Rosemont, IL;
American Academy of Orthopaedic Surgeons. 2013:117-133.

Roehrig S, Straub A, Pohlmann J, Lampe T, Pernerstorfer J, Schlemmer KH, Reinemer P, Perzborn E.


Discovery of the novel antithrombotic agent 5-chloro-N-({(5S)-2-oxo-3-[4-(3-oxomorpholin-4-yl)phenyl]-
1,3-oxazolidin-5-yl}methyl)thiophene-2-carboxamide (BAY 59-7939): an oral, direct factor Xa inhibitor. J
Med Chem.2005 Sep 22;48(19):5900-8. PubMed PMID: 16161994. 

© 2015 American Academy of Orthopaedic Surgeons 2015 Orthopaedic In-Training Examination


248 • American Academy of Orthopaedic Surgeons

Question 24
What is the most likely cause of a pseudotumor in a well-positioned total hip arthroplasty?

1. Loosening of the cup at the bone interface


2. Fretting and corrosion reaction from the taper
3. Adhesive wear products from polyethylene
4. Backside wear of the polyethylene liner
5. Catastrophic polyethylene failure with metal-on-metal contact

PREFERRED RESPONSE: 2

RECOMMENDED READINGS
Cook RB, Bolland BJ, Wharton JA, Tilley S, Latham JM, Wood RJ. Pseudotumour formation due to
tribocorrosion at the taper interface of large diameter metal on polymer modular total hip replacements.
J Arthroplasty. 2013 Sep;28(8):1430-6. doi: 10.1016/j.arth.2013.02.009. Epub 2013 Mar 23. PubMed
PMID: 23528556.

Lieberman JR ed. AAOS Comprehensive Orthopaedic Review. Rosemont, IL; American Academy of
Orthopaedic Surgeons. 2009: 21.

Question 36
Which gait pattern is most likely associated with knee osteoarthritis progression?

1. Abductor lurch
2. Ankle-flexed short stance
3. Increased adductor moment
4. Impulse loading after heel strike
5. Anterior cruciate ligament (ACL) protective gait

PREFERRED RESPONSE: 3

RECOMMENDED READINGS
Li J, Hosseini A, Gadikota HR, Li G. Kinesiology of the knee joint. In: O’Keefe RJ, Jacobs JJ, Chu CR,
Einhorn TA, eds. Orthopaedic Basic Science Foundations of Clinical Practice. 4th ed. Rosemont, IL:
American Academy of Orthopaedic Surgeons; 2013:261-278.

Chehab EF, Favre J, Erhart-Hledik JC, Andriacchi TP. Baseline knee adduction and flexion moments
during walking are both associated with 5 year cartilage changes in patients with medial knee
osteoarthritis. Osteoarthritis Cartilage. 2014 Nov;22(11):1833-9. doi: 10.1016/j.joca.2014.08.009. Epub
2014 Aug 27. PubMed PMID: 25211281; PubMed Central PMCID: PMC4369510. 

© 2015 American Academy of Orthopaedic Surgeons 2015 Orthopaedic In-Training Examination


SECTION 10: Basic Science • 249

Question 52
What is a major controlling molecule for tendon collagen fiber size?

1. Elastin
2. Fibrillin
3. Decorin
4. Biglycan
5. Fibronectin

PREFERRED RESPONSE: 3

RECOMMENDED READINGS
Reuther KE, Gray CF, Soslowsky LJ. Form and function of tendon and ligament. In: O’Keefe RJ, Jacobs
JJ, Chu CR, Einhorn TA. eds. Orthopaedic Basic Science: Foundations of Clinical Practice. 4th ed.
Rosemont, IL; American Academy of Orthopaedic Surgeons; 2013: 213-228.

Zhang G, Ezura Y, Chervoneva I, Robinson PS, Beason DP, Carine ET, Soslowsky LJ, Iozzo RV, Birk DE.
Decorin regulates assembly of collagen fibrils and acquisition of biomechanical properties during tendon
development. J Cell Biochem. 2006 Aug 15;98(6):1436-49. PubMed PMID: 16518859. 

Question 61
What is the principal cause of age-related increase in articular cartilage brittleness?

1. More rapid cleavage of collagen


2. Increase in advanced glycation end products
3. Increased association of type IX collagen with type II collagen
4. Loss of matrilin 3 association with collagen fibrils
5. Type II collagen replacement with type III collagen

PREFERRED RESPONSE: 2

RECOMMENDED READINGS
Chubinskaya S, Malfait A-M, Wimmer MA. Form and function of articular cartilage. In: O’Keefe RJ,
Jacobs JJ, Chu CR, Einhorn TA. eds. Orthopaedic Basic Science: Foundations of Clinical Practice. 4th ed.
Rosemont, IL: American Academy of Orthopaedic Surgeons; 2013:183-197.

Shane Anderson A, Loeser RF. Why is osteoarthritis an age-related disease? Best Pract Res Clin
Rheumatol. 2010 Feb;24(1):15-26. doi: 10.1016/j.berh.2009.08.006. Review. PubMed PMID: 20129196.

© 2015 American Academy of Orthopaedic Surgeons 2015 Orthopaedic In-Training Examination


250 • American Academy of Orthopaedic Surgeons

Question 67
The muscle energy source for a marathon athlete is predominantly

1. glycogen.
2. glycogen and fatty acids.
3. glycogen and lactic acids.
4. Adenosine triphosphate (ATP) and creatine phosphate.
5. ATP, creatine phosphate, and glycogen.

PREFERRED RESPONSE: 2

RECOMMENDED READINGS
Wright A, Gharaibeh B, Huard J. Form and function of skeletal muscle. In: O’Keefe RJ, Jacobs JJ, Chu
CR, Einhorn TA, eds. Orthopaedic Basic Science Foundations of Clinical Practice. 4th ed. Rosemont, IL:
American Academy of Orthopaedic Surgeons; 2013:229-237.

Garrett WE Jr, Best TM. Anatomy, physiology, mechanics of skeletal muscle. In: Buckwalter JA, Einhorn
TA, Simon SR, eds. Orthopaedic Basic Science: Biology and Biomechanics of the Musculoskeletal
System. Rosemont, IL: American Academy of Orthopaedic Surgeons; 2000:693-716.

Question 72
Which molecule is most responsible for the hydration of the intervertebral disk?

1. Decorin
2. Versican
3. Aggrecan
4. Type I collagen
5. Type II collagen

PREFERRED RESPONSE: 3

RECOMMENDED READINGS
Moss IL, An HS. Form and function of the intervertebral disk. In: O’Keefe RJ, Jacobs JJ, Chu CR,
Einhorn TA, eds. Orthopaedic Basic Science: Foundations of Clinical Practice. 4th ed. Rosemont, IL:
American Academy of Orthopaedic Surgeons; 2013:253-260.

Roughley PJ. Biology of intervertebral disc aging and degeneration: involvement of the extracellular
matrix. Spine (Phila Pa 1976). 2004 Dec 1;29(23):2691-9. Review. PubMed PMID: 15564918.

© 2015 American Academy of Orthopaedic Surgeons 2015 Orthopaedic In-Training Examination


SECTION 10: Basic Science • 251

Question 84
What is the mechanism of action for denosumab in the treatment of osteoporosis?

1. Blocking the parathyroid receptor


2. Blocking the carbonic anhydrase receptor
3. Attachment to osteoprotegerin (OPG)
4. Attachment to receptor activator of nuclear factor kappa beta (RANK)
5. Inhibition of RANK ligand (RANKL)

PREFERRED RESPONSE: 5

RECOMMENDED READINGS
Bukata SV, Tyler WK. Metabolic bone disease. In: O’Keefe RJ, Jacobs JJ, Chu CR, Einhorn TA, eds.
Orthopaedic Basic Science: Foundations of Clinical Practice. 4th ed. Rosemont, IL: American Academy
of Orthopaedic Surgeons; 2013:353-364.

Yasuda H. RANKL, a necessary chance for clinical application to osteoporosis and cancer-related bone
diseases. World J Orthop. 2013 Oct 18;4(4):207-17. doi: 10.5312/wjo.v4.i4.207. Review. PubMed PMID:
24147256.

Capozzi A, Lello S, Pontecorvi A. The inhibition of RANK-ligand in the management of postmenopausal


osteoporosis and related fractures: the role of denosumab. Gynecol Endocrinol. 2014 Jun;30(6):403-8. doi:
10.3109/09513590.2014.892067. Epub 2014 Mar 5. PubMed PMID: 24592987.

Question 87
What is the function of sclerostin in adult bone homeostasis?

1. Decreases bone formation by inhibiting osteoblastogenesis


2. Decreases bone formation by promoting osteoclastogenesis
3. Decreases bone formation by inhibiting osteocytes
4. Increases bone formation by promoting osteoblastogenesis
5. Increases bone formation by inhibiting osteoclastogenesis

PREFERRED RESPONSE: 1

RECOMMENDED READINGS
Suen PK, He YX, Chow DH, Huang L, Li C, Ke HZ, Ominsky MS, Qin L. Sclerostin monoclonal
antibody enhanced bone fracture healing in an open osteotomy model in rats. J Orthop Res. 2014
Aug;32(8):997-1005. doi: 10.1002/jor.22636. Epub 2014 Apr 30. PMID: 24782158.

Bukata SV, Tyler WK. Metabolic bone disease. In: O’Keefe RJ, Jacobs JJ, Chu CR, Einhorn TA, eds.
Orthopaedic Basic Science: Foundations of Clinical Practice. 4th ed. Rosemont, IL: American Academy
of Orthopaedic Surgeons; 2013:353-364.


© 2015 American Academy of Orthopaedic Surgeons 2015 Orthopaedic In-Training Examination
252 • American Academy of Orthopaedic Surgeons

Question 100
After initial arthroplasty surgery, infection that occurs within how many days is considered a surgical-site
infection?

1. 30 days
2. 45 days
3. 60 days
4. 90 days
5. 365 days

PREFERRED RESPONSE: 5

RECOMMENDED READINGS
Salava JK, Springer BD. Orthopaedic infections. In: Cannada LK, ed. Orthopaedic Knowledge Update 11.
Rosemont, IL: American Academy of Orthopaedic Surgeons; 2014:287-306.

Mangram AJ, Horan TC, Pearson ML, Silver LC, Jarvis WR. Guideline for Prevention of Surgical Site
Infection, 1999. Centers for Disease Control and Prevention (CDC) Hospital Infection Control Practices
Advisory Committee. Am J Infect Control. 1999 Apr;27(2):97-132; quiz 133-4; discussion 96. PubMed
PMID: 10196487.

Question 106
A 5-year-old boy has progressive muscle weakness. Examination reveals pseudohypertrophy of the calf
and deltoid musculature and a positive Gower sign. How is this disorder most commonly inherited?

1. Non-Mendelian
2. Autosomal recessive
3. Autosomal dominant
4. X-linked dominant
5. X-linked recessive

PREFERRED RESPONSE: 5

RECOMMENDED READINGS
Babis GC, Sakellariou VI. Muscle disorders. In: Cannada LK, ed. Orthopaedic Knowledge Update 11.
Rosemont, IL: American Academy of Orthopaedic Surgeons; 2014:237-252.

Shieh PB. Muscular dystrophies and other genetic myopathies. Neurol Clin. 2013 Nov;31(4):1009-29. doi:
10.1016/j.ncl.2013.04.004. Review. PubMed PMID: 24176421.

© 2015 American Academy of Orthopaedic Surgeons 2015 Orthopaedic In-Training Examination


SECTION 10: Basic Science • 253

Question 115
Which factor is most commonly used to stimulate cartilage regeneration in vitro?

1. Tumor necrosis factor (TNF)-alpha


2. Transforming growth factor (TGF)-beta
3. Platelet-derived growth factor (PDGF)
4. Fibroblast growth factor (FGF)
5. Insulin-like growth factor-1 (IGF-1)

PREFERRED RESPONSE: 2

RECOMMENDED READINGS
Klatt BA, Chen A, Tuan R. Arthritis and other cartilage disorders. In: Cannada LK, ed. Orthopaedic
Knowledge Update 11. Rosemont, IL: American Academy of Orthopaedic Surgeons; 2014:207-222.

Fortier LA, Barker JU, Strauss EJ, McCarrel TM, Cole BJ. The role of growth factors in cartilage repair.
Clin Orthop Relat Res. 2011 Oct;469(10):2706-15. doi: 10.1007/s11999-011-1857-3. Review. PubMed
PMID: 21403984; PubMed Central PMCID: PMC3171543.

Question 118
Which immune cell type is involved in metal allergy associated with orthopaedic implants?

1. Mast cells
2. Dendritic cells
3. Natural killer cells
4. B lymphocytes
5. T lymphocytes

PREFERRED RESPONSE: 5

RECOMMENDED READINGS
Jacobs JJ, Urban RM, Hallab NJ, Skipor AK, Fischer A, Wimmer MA. Metal-on-metal bearing surfaces. J
Am Acad Orthop Surg. 2009 Feb;17(2):69-76. PubMed PMID: 19202120.

O’Keefe RJ, Jacobs JJ, Chu CR, Einhorn TA, eds. Orthopaedic Basic Science: Foundations of Clinical
Practice. 4th ed. Rosemont, IL; American Academy of Orthopaedic Surgeons: 2013.

© 2015 American Academy of Orthopaedic Surgeons 2015 Orthopaedic In-Training Examination


254 • American Academy of Orthopaedic Surgeons

Question 126
Which ion and ion channel are responsible for generation of a nerve action potential?

1. Sodium ion through a voltage-gated channel


2. Sodium ion through a transmitter-gated channel
3. Potassium ion through a voltage-gated channel
4. Potassium ion through a mechanically-gated channel
5. Chloride ion through a mechanically-gated channel

PREFERRED RESPONSE: 1

RECOMMENDED READINGS
O’Keefe RJ, Jacobs JJ, Chu CR, Einhorn TA, eds. Orthopaedic Basic Science: Foundations of Clinical
Practice. 4th ed. Rosemont, IL; American Academy of Orthopaedic Surgeons: 2013.

Lee DH, Claussen GC, Oh S. Clinical nerve conduction and needle electromyography studies. J Am Acad
Orthop Surg. 2004 Jul-Aug;12(4):276-87. Review. PubMed PMID: 15473679.

Catterall WA. Structure and function of voltage-gated sodium channels at atomic resolution. Exp Physiol.
2014 Jan;99(1):35-51. doi: 10.1113/expphysiol.2013.071969. Epub 2013 Oct 4. Review. PubMed PMID:
24097157; PubMed Central PMCID: PMC3885250.

© 2015 American Academy of Orthopaedic Surgeons 2015 Orthopaedic In-Training Examination


SECTION 10: Basic Science • 255

Figure 134

Question 134
Figure 134 is the radiograph of a 50-year-old man. Laboratory tests were performed based on these
findings, and results demonstrated an elevated alkaline phosphatase level. Which cell type is abnormally
altered in the underlying pathology of this disorder?

1. Osteoclasts
2. Osteoblasts
3. Osteocytes
4. Osteoprogenitor cells
5. Mesenchymal stem cells

PREFERRED RESPONSE: 1

RECOMMENDED READINGS
Ralston SH, Langston AL, Reid IR. Pathogenesis and management of Paget’s disease of bone. Lancet.
2008 Jul 12;372(9633):155-63. doi: 10.1016/S0140-6736(08)61035-1. Review. PubMed PMID:
18620951.

Kaplan FS, Singer FR. Paget’s Disease of Bone: Pathophysiology, Diagnosis, and Management. J Am
Acad Orthop Surg. 1995 Nov;3(6):336-344. PubMed PMID: 10790672.

© 2015 American Academy of Orthopaedic Surgeons 2015 Orthopaedic In-Training Examination


256 • American Academy of Orthopaedic Surgeons

Question 142
A 46-year-old laborer has elbow pain for 1 year. The pain is located over the lateral aspect, and he denies
any trauma. Examination reveals pain to palpation just distal to the lateral epicondyle and pain with
resisted wrist extension with the elbow in extension. Where does the primary muscle involved in this
disorder insert?

1. Radial styloid
2. Base of the second metacarpal
3. Base of the third metacarpal
4. Base of the fourth metacarpal
5. Base of the fifth metacarpal

PREFERRED RESPONSE: 3

RECOMMENDED READINGS
Brummel J, Baker CL 3rd, Hopkins R, Baker CL Jr. Epicondylitis: lateral. Sports Med Arthrosc. 2014
Sep;22(3):e1-6. doi: 10.1097/JSA.0000000000000024. Review. PubMed PMID: 25077751.

Bunata RE, Brown DS, Capelo R. Anatomic factors related to the cause of tennis elbow. J Bone Joint Surg
Am. 2007 Sep;89(9):1955-63. PubMed PMID: 17768192.

© 2015 American Academy of Orthopaedic Surgeons 2015 Orthopaedic In-Training Examination


SECTION 10: Basic Science • 257

Figure 148a Figure 148b

Question 148
Which physical examination finding is consistent with the lesion seen on the MR images shown in Figures
148a and 148b?

1. Saddle anesthesia
2. Weak great toe extension
3. Decreased patellar tendon reflex
4. Decreased Achilles tendon reflex
5. Diminished sensation over the dorsum of the foot

PREFERRED RESPONSE: 4

RECOMMENDED READINGS
O’Keefe RJ, Jacobs JJ, Chu CR, Einhorn TA, eds. Orthopaedic Basic Science: Foundations of Clinical
Practice. 4th ed. Rosemont, IL; American Academy of Orthopaedic Surgeons: 2013.

Hoppenfeld S, Thomas H. Physical Examination of the Spine and Extremities 1st ed. East Norwalk, CT,
Appleton-Century Crofts, 1976.

© 2015 American Academy of Orthopaedic Surgeons 2015 Orthopaedic In-Training Examination


258 • American Academy of Orthopaedic Surgeons

Question 153
What happens to the size and cellularity of the nucleus pulposus proportional to the intervertebral disk
after birth?

1. Increase in size and cellularity


2. Increase in size and decrease in cellularity
3. Decrease in size and cellularity
4. Decrease in size and increase in cellularity
5. Size and cellularity remain constant

PREFERRED RESPONSE: 3

RECOMMENDED READINGS
O’Keefe RJ, Jacobs JJ, Chu CR, Einhorn TA, eds. Orthopaedic Basic Science: Foundations of Clinical
Practice. 4th ed. Rosemont, IL; American Academy of Orthopaedic Surgeons: 2013.

Urban JPG, Roberts S, Ralphs JR. The Nucleus of the Intervertebral Disc from Development to
Degeneration. Amer Zool. 2000;40:53-61.

© 2015 American Academy of Orthopaedic Surgeons 2015 Orthopaedic In-Training Examination


SECTION 10: Basic Science • 259

Figure 155

Question 155
What is the most common fiber type in the knee structure shown in the arthroscopic image in Figure 155?

1. Proteoglycans
2. Elastin
3. Sharpey fibers
4. Collagen type I
5. Collagen type II

PREFERRED RESPONSE: 4

RECOMMENDED READINGS
Duthon VB, Barea C, Abrassart S, Fasel JH, Fritschy D, Ménétrey J. Anatomy of the anterior cruciate
ligament. Knee Surg Sports Traumatol Arthrosc. 2006 Mar;14(3):204-13. Epub 2005 Oct 19. Review.
PubMed PMID: 16235056.

O’Keefe RJ, Jacobs JJ, Chu CR, Einhorn TA, eds. Orthopaedic Basic Science: Foundations of Clinical
Practice. 4th ed. Rosemont, IL; American Academy of Orthopaedic Surgeons: 2013.

© 2015 American Academy of Orthopaedic Surgeons 2015 Orthopaedic In-Training Examination


260 • American Academy of Orthopaedic Surgeons

Question 166
How does the World Health Organization (WHO) define osteopenia based on the dual-energy x-ray
absorptiometry test?

1. T-score between 0 and 1 standard deviation below the norm


2. T-score between 1 and 2.5 standard deviations below the norm
3. Z-score between 0 and 1 standard deviation below the norm
4. Z-score between 1 and 2.5 standard deviations below the norm
5. Bone mineral density below 2.5 standard deviations of the mean for a healthy 30-year-old

PREFERRED RESPONSE: 2

RECOMMENDED READINGS
Blake GM, Fogelman I. Role of dual-energy X-ray absorptiometry in the diagnosis and treatment of
osteoporosis. J Clin Densitom. 2007 Jan-Mar;10(1):102-10. Epub 2006 Dec 27. Review. PubMed PMID:
17289532.

Templeton K. Secondary osteoporosis. J Am Acad Orthop Surg. 2005 Nov;13(7):475-86. Review. PubMed
PMID: 16272272.

Question 170
Hemophilia A is a genetic disorder that is transmitted by which inheritance pattern?

1. Autosomal dominant
2. Autosomal recessive
3. X-linked recessive
4. X-linked dominant
5. Mitochondrial

PREFERRED RESPONSE: 3

RECOMMENDED READINGS
Vanderhave KL, Caird MS, Hake M, Hensinger RN, Urquhart AG, Silva S, Farley FA. Musculoskeletal
care of the hemophiliac patient. J Am Acad Orthop Surg. 2012 Sep;20(9):553-63. doi: 10.5435/
JAAOS-20-09-553. Review. PubMed PMID: 22941798.

Buckwalter JA, Einhorn TA, Simon SR, eds. Orthopaedic Basic Science: Biology and Biomechanics of the
Musculoskeletal System. 2nd ed. Rosemont, IL: American Academy of Orthopaedic Surgeons; 2000. 

© 2015 American Academy of Orthopaedic Surgeons 2015 Orthopaedic In-Training Examination


SECTION 10: Basic Science • 261

Figure 176a Figure 176b

Figure 176

Question 176
Figures 176a through 176c are the MR images and biopsy specimen of a 42-year-old woman who has a
slow-growing mass in her right knee. Which translocation is associated with this condition?

1. 11:22
2. X:18
3. 12:16
4. 12:22
5. 9:22

PREFERRED RESPONSE: 2

© 2015 American Academy of Orthopaedic Surgeons 2015 Orthopaedic In-Training Examination


262 • American Academy of Orthopaedic Surgeons

RECOMMENDED READINGS
Ladanyi M, Antonescu CR, Leung DH, Woodruff JM, Kawai A, Healey JH, Brennan MF, Bridge JA, Neff
JR, Barr FG, Goldsmith JD, Brooks JS, Goldblum JR, Ali SZ, Shipley J, Cooper CS, Fisher C, Skytting
B, Larsson O. Impact of SYT-SSX fusion type on the clinical behavior of synovial sarcoma: a multi-
institutional retrospective study of 243 patients. Cancer Res. 2002 Jan 1;62(1):135-40. PubMed PMID:
11782370.

Kawai A, Woodruff J, Healey JH, Brennan MF, Antonescu CR, Ladanyi M. SYT-SSX gene fusion as a
determinant of morphology and prognosis in synovial sarcoma. N Engl J Med. 1998 Jan 15;338(3):153-
60. PubMed PMID: 9428816.

Chotel F, Unnithan A, Chandrasekar CR, Parot R, Jeys L, Grimer RJ. Variability in the presentation of
synovial sarcoma in children: a plea for greater awareness. J Bone Joint Surg Br. 2008 Aug;90(8):1090-6.
doi: 10.1302/0301-620X.90B8.19815. PubMed PMID: 18669969.

Question 186
The process of reading DNA information by RNA polymerase to make specific complementary mRNA is
known as

1. replication.
2. recombination.
3. translation.
4. translocation.
5. transcription.

PREFERRED RESPONSE: 5

RECOMMENDED READINGS
Zuscik MJ, Drissi MH, Chen D, Rosier RN. Molecular and cell biology in orthopaedics. In: Einhorn TA,
O’Keefe RJ, Buckwalter JA, eds. Orthopaedic Basic Science: Foundations of Clinical Practice, 3rd ed.
Rosemont, IL: American Academy of Orthopaedic Surgeons; 2007:3-23.

Lieberman JR, ed. AAOS Comprehensive Orthopaedic Review. Rosemont, IL: American Academy of
Orthopaedic Surgeons; 2009:3-13.

© 2015 American Academy of Orthopaedic Surgeons 2015 Orthopaedic In-Training Examination


SECTION 10: Basic Science • 263

Question 192
Achondroplasia is caused by a mutation in the gene encoding for fibroblast growth factor receptor 3
(FGFR3). This genetic mutation leads to

1. accumulation of the protein in the golgi.


2. gain of function of FGFR3 (turned on).
3. loss of function of FGFR3 (turned off).
4. increased production of the FGFR3 protein.
5. decreased production of the FGFR3 protein.

PREFERRED RESPONSE: 2

RECOMMENDED READINGS
Morcuende JA, Alman BA. Skeletal dysplasias, connective tissue diseases, and other genetic disorders.
In: Flynn JM, ed. Orthopaedic Knowledge Update 10. Rosemont, IL: American Academy of Orthopaedic
Surgeons; 2011:797-810.

Sponseller PD, Ain MC. The skeletal dysplasias. In: Morrissy RT, Weinstein SL, eds. Lovell and Winter’s
Pediatric Orthopaedics. Philadelphia, PA: Lippincott Williams & Wilkins; 2006:205-250. 

Question 198
Which factor promotes inflammation-related osteolysis around orthopaedic implants?

1. Interleukin-4 (IL-4)
2. IL-6
3. Osteoprotegerin
4. Interferon (IFN) gamma
5. Transforming growth factor beta

PREFERRED RESPONSE: 2

RECOMMENDED READINGS
Yousef AA, Clohisy JC. The biological response to orthopaedic implants. In: Einhorn TA, O’Keefe RJ,
Buckwalter, JA, eds. Orthopaedic Basic Science: Foundations of Clinical Practice. 3rd ed. Rosemont,
IL: American Academy of Orthopaedic Surgeons; 2007: 365-77.

Talmo CT, Shanbhag AS, Rubash HE. Nonsurgical management of osteolysis: challenges and
opportunities. Clin Orthop Relat Res. 2006 Dec;453:254-64. Review. PubMed PMID: 17016218. 

© 2015 American Academy of Orthopaedic Surgeons 2015 Orthopaedic In-Training Examination


264 • American Academy of Orthopaedic Surgeons

Question 203
Materials that demonstrate stress-strain behaviors that are time dependent are

1. ductile.
2. brittle.
3. isotropic.
4. anisotropic.
5. viscoelastic.

PREFERRED RESPONSE: 5

RECOMMENDED READINGS
Lieberman JR, ed. AAOS Comprehensive Orthopaedic Review. Rosemont, IL: American Academy of
Orthopaedic Surgeons; 2009:21-28.

Lu L, Kaufman KR, Yaszemski MJ. Biomechanics. In: Einhorn TA, O’Keefe RJ, Buckwalter JA, eds.
Orthopaedic Basic Science: Foundations of Clinical Practice, 3rd ed. Rosemont, IL: American Academy
of Orthopaedic Surgeons; 2007:49-64.

Question 207
The son of a man with Huntington disease is informed that he will likely develop a more severe form of
the disease at a younger age than his father. This is an example of

1. anticipation.
2. DNA methylation.
3. histone deactylation.
4. loss of telomere length.
5. lyonization.

PREFERRED RESPONSE: 1

RECOMMENDED READINGS
Cook PC, Sanders JO. Genetic diseases in orthopaedics. In: O’Keefe RJ, Jacobs JJ, Chu CR, Einhorn
TA, eds. Orthopaedic Basic Science: Foundations of Clinical Practice, 4th ed. Rosemont, IL: American
Academy of Orthopaedic Surgeons; 2013: 43-53.

Martorell L, Monckton DG, Sanchez A, Lopez De Munain A, Baiget M. Frequency and stability of
the myotonic dystrophy type 1 premutation. Neurology. 2001 Feb 13;56(3):328-35. PubMed PMID:
11171897.

Kamsteeg EJ, Kress W, Catalli C, Hertz JM, Witsch-Baumgartner M, Buckley MF, van Engelen BG,
Schwartz M, Scheffer H. Best practice guidelines and recommendations on the molecular diagnosis
of myotonic dystrophy types 1 and 2. Eur J Hum Genet. 2012 Dec;20(12):1203-8. doi: 10.1038/
ejhg.2012.108. Epub 2012 May 30. PubMed PMID: 22643181; PubMed Central PMCID: PMC3499739. 

© 2015 American Academy of Orthopaedic Surgeons 2015 Orthopaedic In-Training Examination


SECTION 10: Basic Science • 265

Question 227
An inactivating mutation in the receptor for 1,25 (OH)2 vitamin D3 is associated with which disorder?

1. Paget disease
2. Hypophosphatasia
3. Renal osteodystrophy
4. Familial hypophosphatemic rickets
5. Type II vitamin D-dependent rickets

PREFERRED RESPONSE: 5

RECOMMENDED READINGS
Brinker MR. Basic Science: Bone. In: Miller MD, ed. Review of Orthopaedics. 3rd ed. Philadelphia, PA:
WB Saunders; 2000: 1-39.

Liberman U, Marx S.Vitamin D-dependent rickets. In: Favus MK (ed). Primer on the Metabolic Bone
Diseases and Disorders of Mineral Metabolism, 5th ed. Washington, DC: The American Society for Bone
and Mineral Research; 2003:311-316.

Question 237
A patient comes to you for a second opinion after undergoing surgery for a rotator cuff tear with another
surgeon in your practice. Following your evaluation, you believe that surgeon error resulted in a poor
outcome. What is your responsibility in reporting to the patient that this possible error may have
contributed to the poor outcome?

1. You are lawfully required to disclose this information to the patient.


2. You are lawfully and ethically required to disclose this information to the patient.
3. You are ethically required to disclose this information to the patient.
4. You are ethically required to discuss this information with your partner.
5. No disclosures are required.

PREFERRED RESPONSE: 3

RECOMMENDED READINGS
Bhattacharyya T, Yeon H. "Doctor, was this surgery done wrong?" Ethical issues in providing second
opinions. J Bone Joint Surg Am. 2005 Jan;87(1):223-5. PubMed PMID: 15634836.

American Academy of Orthopaedic Surgeons. Second or Additional Medical Opinions in Orthopaedic


Surgery. http://www.aaos.org/CustomTemplates/Content.aspx?id=22293. Accessed January 2016. 

© 2015 American Academy of Orthopaedic Surgeons 2015 Orthopaedic In-Training Examination


266 • American Academy of Orthopaedic Surgeons

Question 255
A pregnant 28-year-old woman who is seen in the emergency department with a closed ankle fracture
reports that she tripped and fell at home. The patient’s mother tells you that there is reason to suspect
spousal abuse as a cause for the injury. The patient denies any domestic violence. What is your
responsibility in reporting this possible abuse?

1. You should do nothing further because you have no proof of abuse.


2. You should investigate further by asking other family members if spousal abuse is suspected.
3. You are required to communicate this finding with the emergency department staff and
social worker.
4. The patient should be encouraged to seek self-protection and consult with a social worker at
the hospital.
5. To avoid disciplinary action, by law you must report this possible abuse to the authorities.

PREFERRED RESPONSE: 4

RECOMMENDED READINGS
American Academy of Orthopaedic Surgeons: Child Abuse or Maltreatment, Elder Maltreatment, and
Intimate Partner Violence (IPV): The Orthopaedic Surgeon’s Responsibilities in Domestic and Family
Violence. Rosemont, IL: American Academy of Orthopaedic Surgeons. Available at http://www.aaos.org/
CustomTemplates/Content.aspx?id=22286&ssopc=1. Accessed January 2016.

American Medical Association Council on Ethical and Judicial Affairs. Code of medical ethics: Opinion
2.02 – Physicians’ Obligation in Preventing, Identifying, and Treating Violence and Abuse. Available at
http://www.ama-assn.org/ama/pub/physician-resources/medical-ethics/code-medical-ethics/opinion202.
page?. Accessed January 2016.

© 2015 American Academy of Orthopaedic Surgeons 2015 Orthopaedic In-Training Examination


SECTION 10: Basic Science • 267

Question 274
A thicker myelin sheath will affect nerve transmission by

1. decreasing the depolarization rate.


2. increasing the depolarization rate.
3. increasing the relative refractory period.
4. increasing the absolute refractory period.
5. increasing the speed of wave propagation.

PREFERRED RESPONSE: 5

RECOMMENDED READINGS
Jackson WM, Diao E. Peripheral nerves: form and function. In: O’Keefe RJ, Jacobs JJ, Chu CR, Einhorn
TA. eds. Orthopaedic Basic Science: Foundations of Clinical Practice. 4th ed. Rosemont, IL: American
Academy of Orthopaedic Surgeons; 2013;239-251.

Peters A. The effects of normal aging on myelin and nerve fibers: a review. J Neurocytol. 2002 Sep-
Nov;31(8-9):581-93. Review. PubMed PMID: 14501200.

© 2015 American Academy of Orthopaedic Surgeons 2015 Orthopaedic In-Training Examination


© 2015 American Academy of Orthopaedic Surgeons 2015 Orthopaedic In-Training Examination
AMERICAN ACADEMY OF ORTHOPAEDIC SURGEONS

2015 ORTHOPAEDIC IN-TRAINING EXAMINATION SCORE KEY


Question PR Question PR Question PR Question PR Question PR
1 5 56 2 111 3 166 2 221 1
2 3 57 3 112 2 167 3 222 5
3 1 58 5 113 2 168 3 223 2
4 4 59 4 114 2 169 1 224 1
5 4 60 1 115 2 170 3 225 1
6 1 61 2 116 3 171 2 226 5
7 3 62 5 117 5 172 3 227 5
8 3 63 5 118 5 173 1 228 4
9 2 64 3 119 4 174 2 229 3
10 2 65 2 120 4 175 3 230 2
11 4 66 5 121 2 176 2 231 3
12 4 67 2 122 2 177 5 232 1
13 5 68 1 123 2 178 1 233 3
14 1 69 4 124 4 179 5 234 5
15 2 70 5 125 4 180 4 235 3
16 3 71 3 126 1 181 2 236 1
17 4 72 3 127 1 182 3 237 3
18 5 73 4 128 5 183 2 238 5
19 4 74 4 129 5 184 3 239 3
20 2 75 5 130 5 185 1 240 2
21 1 76 3 131 1 186 5 241 1
22 2 77 3 132 2 187 1 242 2
23 5 78 4 133 1 188 5 243 2
24 2 79 5 134 1 189 3 244 2
25 4 80 1 135 5 190 2 245 5
26 5 81 1 136 2 191 4 246 1
27 5 82 5 137 4 192 2 247 3
28 4 83 1 138 4 193 1 248 5
29 3 84 5 139 5 194 2 249 1
30 1 85 4 140 4 195 1 250 3
31 5 86 2 141 1 196 2 251 1
32 3 87 1 142 3 197 4 252 3
33 1 88 2 143 5 198 2 253 5
34 4 89 5 144 3 199 1 254 2
35 4 90 4 145 4 200 2 255 4
36 3 91 4 146 3 201 2 256 5
37 4 92 2 147 3 202 1 257 4
38 5 93 2 148 4 203 5 258 2
39 4 94 5 149 1 204 2 259 3
40 4 95 3 150 5 205 3 260 3
41 3 96 4 151 1 206 2 261 3
42 4 97 4 152 1 207 1 262 2
43 3 98 4 153 3 208 5 263 1
44 4 99 3 154 1 209 5 264 1
45 1 100 5 155 4 210 2 265 2
46 4 101 3 156 2 211 2 266 3
47 2 102 4 157 1 212 5 267 2
48 4 103 4 158 5 213 1 268 1
49 1 104 5 159 1 214 2 269 1
50 4 105 4 160 4 215 1 270 4
51 4 106 5 161 3 216 3 271 1
52 3 107 1 162 3 217 3 272 4
53 3 108 3 163 5 218 5 273 2
54 1 109 4 164 4 219 5 274 5
55 4 110 3 165 5 220 4 275 1

{Excluded Items from Scoring: 47, 57, 103, 111, 140, 161, 200, 209, 216}

© 2015 American Academy of Orthopaedic Surgeons 2015 Orthopaedic In-Training Examination


2015 Orthopaedic In-Training Examination—Question Numbers by Content Domain

(*Excluded from Scoring: 47, 57, 103, 111, 140, 161, 200, 209, 216)

Basic Science (32)


9, 22, 24, 36, 52, 61, 67, 72, 84, 87, 100, 106, 115, 118, 126, 134, 142, 148, 153, 155, 166, 170,
176, 186, 192, 198, 203, 207, 227, 237, 255, 274

Foot and Ankle (24)


1, 16, 31, 46, 54, 69, 80, 93, 105, 117, 130, 147, 158, 179, 193, *200, 211, *216, 222, 232, 245,
251, 260, 268

Hand (20)
8, 15, 28, 39, 48, 58, 65, 75, 88, 96, *103, 133, *140, 175, 228, 246, 256, 263, 267, 271

Hip & Knee Reconstruction (22)


5, 20, 30, 44, 50, 59, 76, 98, 108, 121, 136, 144, 154, 165, 177, 191, 197, 206, 238, 248, 258, 264

Oncology (23)
4, 17, 25, 37, *47, 60, 71, 79, 90, 109, 119, 127, 151, 156, 172, 187, 217, 241, 254, 257, 262,
270, 273

Pediatrics (35)
3, 13, 21, 29, 33, 38, 43, 49, 56, 64, 70, 74, 81, 86, 92, 95, 104, 113, 122, 129, 138, 143, 152,
162, 169, 174, 183, 189, 195, 210, 223, 229, 239, 247, 265

Shoulder and Elbow (23)


12, 26, 34, 41, 53, 66, 85, 89, 101, 116, 131, 149, 167, 178, 185, 202, 213, 215, 221, 234, 242,
253, 272

Spine (23)
7, 19, 27, 35, 45, 55, 68, 77, 94, 110, 124, 141, 163, 180, 194, 199, 205, 214, 220, 225, 233, 243,
249

Sports Medicine (22)


11, 23, 42, 51, 62, 83, 99, 112, 125, 146, 159, 171, 181, 184, 188, 201, *209, 231, 236, 244, 252,
275

Trauma (51)
2, 6, 10, 14, 18, 32, 40, *57, 63, 73, 78, 82, 91, 97, 102, 107, *111, 114, 120, 123, 128, 132, 135,
137, 139, 145, 150, 157, 160, *161, 164, 168, 173, 182, 190, 196, 204, 208, 212, 218, 219, 224,
226, 230, 235, 240, 250, 259, 261, 266, 269

© 2015 American Academy of Orthopaedic Surgeons 2015 Orthopaedic In-Training Examination


© 2015 American Academy of Orthopaedic Surgeons 2015 Orthopaedic In-Training Examination

You might also like